数理化自学丛书 代数 第四册 ==========第1页========== 数理化自学丛书 代 数 第四册 数理化自学丛书编委会数学编写小组编 州千实公大 上海科学技术出版社 ==========第2页========== 数理化自学丛书代数(第四册)数理化自学丛书编委会数学编写小组编上海科学技术出版社出版 (上海路金二%450号) 92长专专在上海发行所发行上海市印别四印刷开本787×10921/32印张7,75字教170,0001966年6月嘴1版1978年2月第2次印期 书号:13119.716定价:0.49元 ==========第3页========== 内容提要 本书是数理化自学丛书中的“代数”第四册。在本丛书“代数”前三册的基础上,讲述初等代数的最后一部分内容。全书共分六章,即:排列和组合、数学归纳法、二项式定理、复数、方程论初步、二阶和三阶行列式。书中对于每一章节的难点和重点,皆予强调;配有丰富的例题和习题,并附提示和答案,以助读者深入理解和加强练习。 本书可供青年工人、在职干部、知识青年自学之用,也可供中等学校青年教师参考。 ==========第4页========== 目 录 C- 量印说明 §3·4二项展开式中系数间 第一章排列和组合…1 的关系73 §1…1排列…1 §3.5(a-b)n的展开式…77 81·2乘法原则……4 §36二项展开式里各项系 §13相异元素不许重复的 数的和…79 排列…7 §3·7二项式定理在近似计 §1…4全排列…10 算中的应用…82 §15加法原则…11 本章提要…85 §16相异元素可以重复的 复习题三…85 排列…17 第四章复数…87 §17组合…20 §4·1数的概念的扩展…87 §18组合数公式…22 §42复数的概念…89 §1.9组合数的两个性质…28 §43复数与平面内点之间 1·10排列、组合综合应用 的对应……94 题……33 §44复数与平面内向量之 *§111概率…35 间的对应…99 本章提要…39 §45复数的加法和减法…107 复习题一…40 §46复数的乘法…113 第二章数学归纳法…42 §4.7复数的除法…119 §2·1归纳推理和演绎推理…42 ⑧48复数的乘方…122 §2…2数学归纳法…45 §49复数的开方…128 §23数学归纳法在证明不 本章提要…134 等式中的应用…53 复习题四…135 本章提要……57 第五章方程论初步…139 复习题二…57 §5·1多项式f(x)的-…些重 第三章二项式定理…59 要性质……139 §3…1杨辉三角形…59 §5.2综合除法…153 内 §3…2二顶式定理…63 §5.3-一元n次方程…159 §33二项展开式的通项公 §5·4实系数一元次方 式…69 程…………168 美 ==========第5页========== 下 $55有理系数一元%次方 §62三阶行列式…203 程…172 §6·3子行列式和代数余子 §56几种特殊类型的高次 式…209 方程的解法…181 §64三元一次方程组…213 本章提要…191 本章提要…221、 复习题五……………193 复习题六…223 第六章二阶和三阶行列式…196 总复习题…226 §61二阶行列式与二元一 习题答案…231 次方程组…196 ● ==========第6页========== F. 第一章排列和组合 排列和组合是初等代数中的一段独特的内容,·是数学里的重要基础知识之一.它对我们解决许多实际问题,以及进 一步学习某些数学知识(如概率、二项式定理、行列式等等),都有着重要的应用. 本章将在阐明排列和组合的意义的基础上,着重学习几种基本的、常用的排列和组合问题的解法。 §11排 列 让我们来看下面这儿个问题 问题1.用红、黄、蓝三种颜色的小旗,按不同的顺序升上旗竿,用以作出信号.在每个信号里,如果要求这三面小旗都要用到,那末单凭这三面小旗可以作出哪几种不同的信号?[解]在这三面小旗中,先取定一面(例如红旗)升上旗竿,这时,第二面旗子就只能在余下的两面中任取一面(要末是黄旗,要末是蓝旗),第三面旗子就只能用剩下的最后一面旗了.这样就可以作出2种不同的信号: 一黄一蓝 红<蓝—黄 如果第一面旗是取定黄旗或者蓝旗,和上面的讨论一样,可以分别作出2种不同的信号: 红一蓝 一红一黄 黄 蓝く 入蓝一红 黄一红 ==========第7页========== 很明显,上面作出的这6种信号是各不相同的,并且,除这6种信号外,不可能再作出其他不同的信号了.由此,我们就找到了这个问题的答案:可以作出上面列举的这6种不同信号. 问题2.由数字1、2、3,可以组成: 半 (1)多少个没有重复数字的二位数? (2)多少个二位数? [解](①)我们可以按照先选十位上数字再选个位上数字的顺序,把各个不同的二位数一起列举出来: 由此可知,总共可组成6个不同的二位数. (2)对于一般的二位数,两个数位上的数字未必不同,因此,当我们选定了一个数字做十位上的数字以后,个位上的数字仍旧可以在这三个数字中任选,所以把这三个数字所组成的二位数一起列举出来便是: 3 由此可知,总共可组成9个不同的二位数 3 上面所提出的这两个问题,有着一个共同的特点,它们都可以看成是:从一些元素(旗子、数字)中,每次取出儿个元素,按照一定的顺序摆成一排的问题 从m个元素中,每次取出%个元素,按照一定的顺序摆成 一排,称为从m个元素里每次取出n个元素的排列.注意根据上面这个排列的定义,所给的这个元素和取出的”个元素,都不要求各不相同。本书只讨论下面这两种排列: 02● ·线 ==========第8页========== (1)从m个各不相同的元素里,每次取出”个各不相同的元素的排列[如上面的问题1和问题2中的(1)都属于这一类型],以后把这类排列简称为相异元素不许重复的排列. (2)从m个各不相同的元素里,每次取出%个元素(可以重复)的排列[如问题2中的(②)就属于这一类型],以后把这类排列简称为相异元素可重复的排列. 在考虑排列问题时,常常把所给的元素顺次编上号码,用符号,ag,…,am来代表.当元素不多时,还可以简单地用字母a,b,C,d,;或者用数字0,1,2,…,9来表示.熟练地解这类符号、字母或者数字的排列问题,可以帮助我们掌握獬排列问题的基本方法,提高解题能力.在下面的例题和习题里,列入这类问题的目的就在于此, 例写出从四个字母a,b,c,d中每次取出2个字母的所有不同排列,并要求: (1)不许重复; (2)可以重复, 这种排列各有几个? 【解](1)ab,ba,ca,da, ac,bc,cb,do, ad,bd,cd,dc, 这样的排列有12种 (2)aa,ba,ca,da, ab,bb,cb,db, ac,bc,ce,dc, ” ad,bd,cd,dd, 这样的排列共有16种 从上面的例子中可以看到,如果两个排列里所含的元素不完全一样,例如ab和ac,就是不同的排列;如果所含的元素完全一样,而排列的顺序不同,例如ab和ba,那末也是不 3● ==========第9页========== 同的排列 习题11 1.由1、2、3、4、5这五个数字所成的二位数一共有几个?其中不含重复数字的有几个? 2.用红色的、黄色的、蓝色的小旗各一面升上旗竿,以作出信号、总共可作出多少种不同的信号? [提示:作信号时,可以只用一面小旗升上旗竿,也可以用二面或者 三面小旗按不同顺序升上旗竿.] §12乘法原则 对排列问题的研究,主要是求出根据已知条件所作出的不同排列的种数.对于一些简单的问题,可以采用上节例题中的方法,把所有不同的排列列举出来,数出种数找到答案。显然,这种方法是很烦的.为了能找寻出一个简单的、直接的求排列种数的方法,下面先来考察一个具体问题. 问题如果从甲地到乙地有2条路可走,乙地到丙地又有3条路可走,试问:从甲地经乙地而到丙地,可以有儿种不同的走法? 手 B By [解1如果用A:、A2表示从甲地到乙地的两条路,用· B1,B2,B3表示从乙地到丙地的这三条路(上图),从图中可 以看出,从甲地经乙地到丙地,有并且只有下面这6种走法: A1—B1,A1-B,A1—B3, A2—B1,Ag-B2,A2B3. 位滨 ==========第10页========== 可以看出,解这个问题需要考虑两个步骤,第一步,先从由甲地到乙地的这两条路中任意进择一条(有2种选法);第二步,再从乙地到丙地这三条路中任意选择一条(有3种选法);而最后计算出来的不同走法的种数6,正就是这两个步骤中每 一步骠的选法种数(2与3)的乘积 对这个具体问题的解,给了我们一个重要的启示:倘使撇开了这里所说的“从甲地到乙地”、“从乙地到丙地”这些具体内容,面把它们一般地看成是要完成一件事的两个步骤,并且把这里所说的“2条路”、“3条路’一般地叙述成“有m1个方法”、“有m2个方法”,这样,就可以作出如下的结论 设完成第一件事有m1个方法,在完成第一件事以后再完成第二件事又有,个方法;那末,依次完成这两件事,就有 N=mm2 个方法. 更一般地,我们还可以作出这样的结论: 设完成第一件事有m1个方法,在完成第一件事以后再完成第.二件事又有2个方法,在完成这二件事以后再完成第三件事又有m3个方法…,在完成了前%一1件事以后再完成第外件事又有m种方法;那末,依次完成这n件事,就有 Y=m1mg3…mw 个方法. 在上面这个式子里,等号右边是乘积的形式,为了突出这 一点,我们把上面这个结论称为乘法原则 应用这个原则,就可以不通过具体作出排列,而求出符合题设条件的所有的不同排列的种数. 5 ==========第11页========== 例如,对§1·1例题中的(①)所作的排列,可以看成是在排好顺序的两个位置 上,从,b,c,d这四个字母里,选取字母去分别占据,因此可以认为这是要依次完成下面两件事: (1)先在a,b,c,d这四个字母中选取1个,去占据第1号位置; (2)再在余下的3个字母(因为排列里不许有重复字母,所以已经占有第1号位置的那个字母不能再选)中选取1个去占据第2号位置 因为,完成第一件事有4个方法,完成第二件事有3个方法,所以完成这两件事共有 N=4.3=12 个方法 类似地,读者可以自己来说明上节例题里(②)所求的排列种数是 N=4.4=16, 习题12 应用乘法原则解下列各题: 1.从a,b,c,d,e这5个字母里,每次取出: (1)2个; (2)3个; (3)4个; (4)5个 不同字母,求所组成的各种不同排列的种数 [解法举例:(1)N2=5.4=20.] 注意本题的四个小题目都是要求排列的种数N,为便于区别,我 1用N2表示选取2个字母的排列种数,N,N,N5便分别表示选取 3,4,5个字母的排列种数. ==========第12页========== 2.用数字1,2,3,4可以组成多少个: (1)二位数? (2)三位数? (3)四位数? 共中不含有重复数字的各有几个? 3.(1)从9块不同颜色的积木里,取出三块排成一横排,共有多少种不同排法? (2)在所有的三位数里,共有几个不含有数字0的?比较一下,(1)和(2)有什么区别. 义 §13相异元素不许重复的排列 在习题1.2第1题里,读者已经计算过:从,b,c,d,e这5个字母里,每次取出2个、3个、4个、5个不同字母,所组成的各种不同排列的种数分别是: Ng=5.4=20, N8=5.4.3=60 N4=543.2=120, N5=5.4.3.21=120 观察上面这些式子,可以看出:所求的非列种数是若干个连续自然数的乘积,乘积中最大的一个因数就是所给元素的个数,乘积中因数的个数等于排列中含有的元素的个数.为了统一起来,有时也把这5个字母里只取出1个,说成是这5个不同字母中取出1个字母的排列.很明显,这样作出的排列的种数是 N1=5. 把上面这个问题推广到一般的情况:就可以得出结论:从沉个不同元素里,每次取出:1个元素的所有排列的种数是m; 2个不同元素的所有排列的种数是m(m一1); ==========第13页========== 3个不同元素的所有排列的种数是m(m-1)(m一2);4个不同元素的所有排列的种数是 m(m-1)(m-2)(m-3); n个(1≤饥≤m)不同元素的所有排列的种数是 m(m-1)(m-2)…[m-(n-1)], 就是 m(m-1)(m-2)…(m-%+1). 上面这种排列是m个各不相同元素中每次取%个各不 相同元素的排列,我们用符号A表示这类排列里所有不同 排列的种数.于是, Aa=m(m-1)(m-2)…(m-n+1). 这个公式称为排列数公式.这里m,n都表示自然数,且 n≤m, 把这公式写成定理形式,即是: 定理从个不同元素里每次取出%个不同元素的所有“排列的种数,等于个连续自然数的乘积,其中最大的一个数是m. 为了以后解排列问题的需要,下面先来熟悉一些关于排列数的运算 客 例1.计算Aio 2A待的值. 【解] 16.15.1416.15.14 2A 2.8.7.651614.15 1 例2.计算4+A特 A 的值. に解】987.6.5+9876=9876(⑤+1)=36. 987 98.7 ==========第14页========== 习题13(1) 1.计算以下各式的值: (1)A8; (2)A1o; (4)3A8 2Ag· 2.计算以下各式的值: (1)4-6; (2)-A90+A0 d 蹈-a; (3) 4A? A娟+A号+A4蟹氵(4) A A8.A 应用上面的公式,可以很简便地解答一些关于相异元素的不许重复的排列问题但在应用这公式时,必须先考察两点: (1)·所给的元素是不是各不相同的? (2)在作出的排列里,元素是不是各不相同的?下面举例来说明这类问题的解法 例3.某铁路线上一共有48个大小车站,铁路局要为这条路线上准备几种不同车票? [解]因为每张车票都标明起点站和终点站的站名,所以同样的两站间就有2种不同的车票.从48个车站的站名中取出两个车站名,分起点站和终点站排起来,所有这种排列的种数即是本题的解,所以这是求在48个不同元素中每次取2个不同元素的所有排列的种数问题。于是,由上面的定理即得,需要准备的车票种数是 A8=48.47=2256 答:要准备2256种不同的车票 注意在解排列的具体问题时,解答中要作简要的说明,不宜只列出一个算式;在以后解组合问题时也一样。 ==========第15页========== 习题13(2) 1,用1、2、3、4、5、6这6个数字,可以组成多少个没有重复数字的 (1)三位数; (②)四位数. 2.有颜色不同的小旗5面,现在要取出3面顺次升入旗竿作出信号,问可以构成多少种不同的信号? 3.有5本不同的书,要分别包上包书纸.现有花色不同的包书纸6张,问有几种不同的包法? 4.有甲、乙、丙、丁、戊5个队进行乒乓球比赛,如果每个队都要与另一队在本队的场子里以及客队的场子里各比赛一次,问这次比赛总共要进行多少场次? §1·4全排列 在公式 Am=m(m-1)(m-2).(m-见+2)(m-%+1)中,如果令见=m,就得 Am=m(m-1)(m-2)…2.1. 这个公式指出:把m个不同的元素全部取出来作排列,所有这样的排列的种数,等于从1开始的m个连续自然数的连乘积. 3 这种排列称为个不同元素的全排列,并且用专门的符号Pm来表示这类排列所有的排列种数,就是 Pm=Am=1.2.…(m-1)m. 为了方便起见,把从1开始的m个自然数的连乘积,用记号m!(读做阶乘)来表示.应用这一记法,m个不同元素的全排列的种数公式就可以写成 Pm=m! ·10 ==========第16页========== 例1.计算8!-61 7:-6!的值. 分析8!=8.7.65.43.2.1=(8.7)×61, 71=7.6543.2.1=7×6!, 所以分子分母中都含有因数6!,可以先约去6!后再计算。 【解 8!-6!=(8.7)×6!-6!55×6!7!-6!7×61一6! 6×6! 56=9 66 大 注(n+1)!=(n+1)2! =(2+1)n.(n-1)! 44t0… 这种变换在计算中很有用,应该熟悉这些计算技巧. 例2.求证Pn+1一Pn=nPa. [证]Pm+1-Pn=(第+1)!一%!=(%+1)%!-n! =(%+1-1)n! =0现!=n…P, 命题得证. 习题14 1.计算: 、 (1)y-923-8P8; P10 Pa (2)·Po-P 2.求证: (1)P8-8P?+7P6=P; (2)16P8=P5-P4. 3.把编上号码的5台车床排成一列,共有几种不同的排法? §15加法原则 在习题1·1里,我们曾经解过下面这个问题: ·11● ==========第17页========== 问题用红色的、黄色的、蓝色的小旗各一面升上旗竿,以作出信号,总共可作出多少种不同的信号?解这个问题时,我们是这样考虑的: 作出的信号可以按照用到的小旗的面数分成三大类,即 (1)只用一面小旗的,这样作出的信号有A3=3种; (2)用二面小旗的,这样作出的信号有A号=6种; (3)三面小旗都用的,这样作出的信号有A=6种 因为上面这三类信号都不相同,并且除此之外不再有其他不同的信号可作,因此总共可作出的不同信号,应该是这三类方法所作出的各种信号的和,由此得 N=3+6+6=15(种). 象乘法原则一样,从这个问题的解答中,可以启发我们作出如下的一般结论: 设完成一件事有%类方法,只要选择任何一类方法中的 一种方法,这件事就可以完成.如果已知其中第一类方法有m种,第二类方法有m2种…,第n类方法有m种,并且这m1+m2十…+mm种方法里,任何两种方法都不相同,那末完成这件事就有 N=m1+m2+…+ma 种方法. 惠 在上面这个式子里,等号右边是和的形式,为了突出这一点,我们把这一结论称为加法原则 在解一些比较复杂的排列问题时,常常要用到加法原则 a 和§1·2里提出的乘法原则,下面举例来说明 例1.用0、1、2、3这4个数字,可以组成多少个没有重复数字的四位数. 4 分析因为四位数的千位上的数字不能是0,为了这一点,我们把 ◆.12· ==========第18页========== 这个四位数分成两步来排:先排千位上的数字,它可以在1、2、3这三个数字中任意选用一个;再排其他三个数位上的,它们可以选用余下的3个数字,因为只有当这两个步骤都完成后这个四位数才能排好,所以要用乘法原则, 【解]于位上数字的排法有A种; 其他三个数位上数字的排法有P3种. 所以,总共的排法有 N=A·P3=36=18(种). 答:可以排成18个不同的四位数. 这个问题还可用另一种方法来解 (1)四个数字0、1、2、3全部取来排列,共有排法P4种. (②)其中0被排在千位上的排列有P3种(这是因为:0 在千位上的排列,实际上就是1、2、3在个、十、百位上的排 列,而后者有P3种).这些排列不能看成是四位数. (③)所以,所求的种数应该是两者的差,即 P4-P8=24-6=18 由此可知,总共可排成18个不同的四位数上面这种考虑,事实上是应用了加法原则, ·,N=%十%g, 其中N表示四个数字的全排列数,%是0被排在千位上的排 、 列数,2是0不在千位上的排列数, .∴.n2=N-n1, 注意对于一些带有特殊条件的排列,如果不合条件的情况计算比较简单,在解题过程中,常常象这一解法一样,先不考虑这个特殊条 件,把所有的排列种数N求出,然后再减去不适合这一特殊条件的排列 种数1,从而得出解答 例2.6个队员排成一列进行操练,其中新队员甲不能站在排首,也不能站在排尾,问有几种不同排法? ◆·130 ==========第19页========== 下面将用两种方法来解这个问题.先运用加法原则来解,可以这 样考虑:倘使对新队员甲的排法没有限制条件,那末共有排法P。种:在 这些排列里,可以分成3类情况: (1)甲在排首的(有P5种排法), (2)甲在排尾的(有P5种排法), (3)甲不在排首或排尾的 前2种情况都不符合题设条件,把总共的排列种数里减去这两种情况的排列种数,即得符合条件的排列种数 【解1]6人的全排列种数是P6.甲在排首或在排尾的 排列种数都是P5.所以,甲不在排首也不在排尾的排列种数 是 P6-2P:=(6-2)P5=4×120=480. 现在再运用乘法原则来解,可以这样考虑:要使甲既不在排首也不 在排尾,可以先让甲在中间4个位置上任意找一个位置站好(有A种 排法),然后,对其余的5人,再在另外5个位置上作全排列(有P。种排 法).这两个步骤依次完成,排列就完成.所以,所求的排列的种数是 A:与P5的积. 【解2]甲可以排在除去首、尾这两个位置以外的4个 位置上,共有A种排法.其余5人可在另外5个位置上排 列,共有P。种排法.所以,总共的排法有 A4·P6=4P5=480(种). 答:有480种不同的排法 从上面这两个例子中可以看到,解排列应用题时,由于思考方法不同,同一问题可以有几种不同的解法,但求出的结果总是一样的.在解题时,建议读者不仅考虑一种解法,同时也考虑一下有没有其他解法.这样做,一方面可以培养解题能力,同时也可以从两种不同解法的结果是否相同来检验作出的解答有没有错误. 14。 ==========第20页========== 民 习题1·5(1) 1.用数字1、2、3、4、5,可以组成多少个没有重复数字的自然数?[提示:组成的自然数里包括一位数,二位数…,五位数.] 2.5件不同的商品,陈列在橱窗内,将它们排成一列 (1)如果某一件商品必须放在中间,问有几种排法? (2)如果某一件商品不放在中间,问有种排法? (3)如果某一件商品不放在中间,也不放在两端,问有几种排法? 3.6件不同的商品,陈列在橱窗内,将它们排成一列. (1)如果甲乙两种商品要分别放在两端,问有几种排法? (2)如果甲乙两种商品都不放在两端,问有几种排法? (3)如果甲乙两种商品要放在相邻的位置,问有几种排法? (4)如果甲乙两种商品,不能放在相邻的位置,问有几种排法?[提示:(3)要使甲乙两种商品放在相邻位置,可以把它们两个先排定(注意这是两者之间的全排列),然后把它们看成一个整体再与其他4件商品作排列.门] 4.在3、5、7、9、11、13这6个数中,任取1个做分子,1个做分母,可以组成多少个不同的分数?其中真分数、假分数各有几个? 下面再举一些带有条件的数字排列问题,研究一下它们的解法 例3.在3000到8000之间,有多少个没有重复数字的、且能被5整除的奇数. 分析适合题意的数,要具有以下的条件: (1)个位上必须是5; (2)千位上必须是3、4、6、7中之一(5已用过,所以不能再用); (3)百位上、十位上可以是除去已排掉的2个数字以外的8个数字中的任何2个. 由比,本题可以按照以上条件分步来排[解]适合题意的数: (1)个位上必须是5,只有1种排法; 15 ==========第21页========== (2)千位上可以是3、4、6、7中之一,共有A种排法; (3)百位和十位上,可以把其余8个数字作排列,共有 A种排法. 所以,适合题意的数共有 A1A娱=4×8×7=224(个). 从这个例子的解答中可以看到,对一个带有条件的排列问题,如果把排列划分成几个步骤来排,那末应该把有条件的位置先排好,再排其余的位置 例4.在3000和8000之间,有多少个没有重复数字的奇数. 分析适合题意的数,要具有以下的条件: (1)个位上必须是1、3、5、7、9中之一; (2)千位上必须是3、4、5、6、7中之一; (3)其他两位上可以是除去已排了的2个数字以外的8个数字中的的任意两个 在条件(1)与(2)中,3、5、7这3个数字重复,因此还不能直接作分步排列.为了避免重复,把干位上的数字再分成两类: (i)千位上是4、6中之一,这时个位上就可以是13、5、7、9中之 言 ()千位上是3、5、7中之一,这时个位上就可以是1、35、7、9这5个数字中除去已排了的1个数字以外的4个数字中的任意一个 意 由此,本题可以先分成两种情况,求出每一情况的所有排列的种数后,再相加而得 [解]适合题意的数,按千位上的数字是偶数或奇数而分成两类 (1)千位上是偶数.它可以在4、6中任取1个,有A经种 排法.这时,个位上有A结种排法,其他两个数位上有种 排法,所以这一类的数有 A2AA层(个) 18 ==========第22页========== 无 (2)千位上是奇数.它可以从3、57和任取1个,有A3 种排法.这时,个位上只有A种排法,其他两个数位上仍有 A种排法,所以这一类的数有 A经·A1A号(个). 由(1)和(2)可知,总共可排成适合题意的数有 AA·A堡+A3·A}A民 =(10+12)×8×7 =1232(个). 注意在解本题时,如果不把千位上数字的情况先划分,从个位上 有A6种排法,干位上有A种排法,其他两个数位上有A:种排法,直接 得出共有 A6·A6A号=25×8×7=1400 种不同的排法,那未就错了.这是因为,这里包括着一类千位上和个位上数字相同的数(如:3453),它们是不符合题意的,在解题时思考必须慎密,既要防止遗漏,也要防止重复, 习题15(2) , 1.用0、1、2、3、4、5这6个数字,可以组成多少个没有重复数字的: (1)三位数? (2)能被5整除的三位数? (3)三位偶数? (4)三位奇数? g、 2.用数字2、3、4、5、6、7组成的没有重复数字的六位数里: (1)能被25整除的有几个? (2)能被6整除的有几个? [提示:能被25整除的数,末两位必须是00、25、50、.75.] 3.求证:用1、2、3、4、5、6这6个数字所组成的没有重复数字的六位数里,1不在第一位、6不在第六位的共有6!-25!+4!个. §1·6相异元素可以重复的排列 在§1·1的问题2的(2)里,曾经计算过:用1、2、3这3 17t ==========第23页========== 个数字所纽成的二位数共有9个,这个结果很容易应用乘法原则来说明.事实上,这个二位数可以分成两步来排.第1步排十位上的,显然有3种排法;第2步排个位上的,因为数字可以重复,所以仍有3种排法.由此得到总共有 3×3=32=9 种排法. 这类问题称为相异元素可以重复的排列问题.应用乘法原则,很容易推出: m个不同元素里,如果每个元素都可以重复选取,那末取%个元素的所有排列的种数是 N=m", 这里m、%都是自然数 在解相异元素可以重复的排列问题时,根据题意正确地判断哪一个数应该作为底数m,哪一个数作为指数化,是解题的一个重要关键。举例说明如下: 例1.(1)3个零件,被分配到4架车床上加工,共有几种不同的分配方法? (2)3个学生争取参加4项公益劳动,共有几种不同的分配方法? [解】(1)因为每一零件都可以分配给4架车床的任一架来加工,都有4种分配方法,所以有 N=4×4×4=43=64 种分配方法. (2)因为每1项公益劳动都可能被3个同学中的任何一个争取到,都有3种分配方法,所以有 W=3X3×3X3=34=81 种分配方法、 ”18p ==========第24页========== 式 从这个例子可以看出,(1)和(2)都是相异元素可以重复的排列问题,但是前者要以车床的架数4作为底数,后者则要以学生人数3作底数.在解题时,首先把题意仔细分析清楚,判断出应该以哪一个为主来考虑分配[例如在(1)中,以零件为主考虑分配的方法;在(②)中,以公益劳动为主考虑分配的方法],直接应用乘法原则来解就可以防止发生错误 因为在相异元素的排列问题里,根据题意选出的元素有的不许重复,有的允许重复,因此,在解题时要正确地判断选出的元素是否可以重复,这是解题时应该加以注意的例2.7个运动员争取参加3项比赛,且每种此赛只派1个运动员参加: (1)如果每人最多只能参加1个项目,有儿种分配方法? (2)如果每人可以参加1项、2项、3项比赛,或者1项比赛也不参加,有几种分配方法? 分析(1)因为每人最多只能参加1个项目,所以参加第1项比赛有7种分配法,参加第2项比赛便只有6种分配法,参加第3项比赛便只有5种分配法.这是7个相异元素中取3个元素的不许重复的排 列问题 (2)因为每个比赛项目都可以让这7人中的任一人参加,都有7种分配方法,所以这是7个元素中取3个元素允许重复的排列问题。[解](1)分配方法有=7×6×6=210种; (2)分配方法有7×7×7=343种. 习题1·6 1.有7个体操运动员,同时参加4项体操比赛,分项冠军的获得者有几种能? 2.某城市的电话号码原来有5个数码,以后改成了6个数码.这样改变以后,可以增加多少个用户(如果规定电话号码中第1个数字不 419◆ ==========第25页========== 用0)? 3.用0、1、2、…、9这10个数字,可以组成多少个不同的三位数?在这些三位数中 (1)没有重复数字的有几个? (2)三个数字都重复的有几个? (3)只有两个重复数字的有几个? 4.有5本不同的书,准备送给3个小朋友: (1)如果每人只能得1本,有几种送法? (2)如果这5本书都要送完,但不限定每人都要得到,有几种送法? 5.有2种不同的书,每种有p本,现在从这些书中取书,证明不同的取书方法共有(p+1)”一1种. §17组.合 让我们来看下面的问题: 问题飞行在北京—一上海一一广州航空线上的民航飞机: (1)要准备多少种不同的飞机票? (2)有几种不同的飞机票价? 这个问题里的(1)是3个相异元素取2个元素不许重复 的排列问题:从A经=6可知共有6种不同的飞机票,具体地 说,就是 〔起点站)北京北京上海上海广州广州〔终点站)上海广州洲.北京·广州北京上海至于这个问题里的(2),性质与(1)有些不同.因为飞机票的种数和起点站、终点站有关,从北京到上海和上海到北京应当准备两种飞机票;但是,飞机票的票价只与起点站到终点站间的距离有关,从北京到上海和从上海到北京的飞机票价 ·20 ==========第26页========== 是相同的.由此,从(①)求出的结果可以看出,不同的飞机票价只有6÷2=3种.具体地说,就是 北京与广州间的票价,北京与上海问的票价,广州与上海间的票价. 象上面(2)这类问题,称为3个不同元素中取2个不同元素的组合问题. 一般地,在m个不同元素中,每次取出%个元素,不管怎样的顺序并成一组,称为m个不同元素里每次取出几个不同元素的组合① 从上面的例子中可以看出,m个不同元素中每次取%个不同元素的排列与组合之间,有以下的主要区别: 在排列中,要考虑元素间的先后顺序,所以在两个排列里,即使元素完全相同,只要这些元素间的先后顺序不同,就要看成是不同的排列 在组合中,不考虑元素间的先后顺序,所以在两个组合里,只要元荼完全相同,就是同一种组合 抓住了“有没有顺序关系”这一点,就可以正确地判断被考虑的问题是排列问题还是组合问题 正因为组合问题是可以不考虑取出元素的先后顺序的,所以要作出具体的组合,可把所给的元素编上号码,规定小的号码在前、大的号码在:后,再象§1·1作出具体的排列那样,把所有的组合无遗漏无重复地写出来 例写出在a,b,c,d这4个字母中每次取2个字母的所有组合. ①对于组合题,象排列问题一样,也有元素可以重复选取的组合和不尽相异元系的组合等等,本不研究这类问题。 ·21· ==========第27页========== [解]我们按a-b-g一d这一顺序来考虑, c-d. 由此可知,共有6种不同的组合,即 ab,ac,ad,be,bd,cd, 习题17 1.(1)写出1、2、3、4、5这5个数字中每次取3个不同数字的所有不同的组合?这样的组合一共有几个? (2)在1、2、3、4、5这5个数字中取出3个不同数字,可以作成几个三位数? (3)从(1)与(②)的结果,可以发现:5个不同元素中每次取出3个元素所有组合的种数与5个不同元素中每次取出3个不同元素所有排列的种数之间有怎样的关系? 2.判断下面这些问题里所指的是排列呢?还是组合? (1)小组里有甲、乙、丙、丁、戊5个组员,在假期里约定每两人要:()互通一封信,问他们总共要写多少封信?()通一次电话,问他们总共要通几次电话? (②)平面内有A,B,C,D,E这5个点,其中无三点共线,问: (1)过这5点中任意两点,可作多少条不同的直线?()以其中的一点为端点,并过另一点的射线有几条? 3、在上题中,把每一小题的结果计算出来(如果是排列问题,可以直接应用公式来算,如果是组合问题,先把所有的组合写出,再数一数有多少个组合).由此可以发现:5个相同元素中每次取出2个不同元素的所有组合的种数与所有排列的种数间有怎样的关系? §1-8组合数公式 在§1·7已经计算过:3个不同元素中每次取出2个不 ·22· ==========第28页========== 同元素的所有组合的种数,就等于3个不同元素中每次取2个不同元素的所有排列的种数除以2,这里“2'就是2个不同元素的全排列数2!, 读者在习题1·7里也曾计算过:5个不同元素中每次取出3个不同元素的所有组合的种数,就等于5个不同元素里每次取3个不同元素的所有排列的种数除以6,这里“6”就是3个不同元素的全排列数3!. 现在我们来证明:m个不同元素中每次取出%个不同元素的所有组合的种数与所有排列的种数之间,都具有这样的关系 定理从m个不同元素里每次取出%个不同元素的所有组合的种数,等于从个不同元素里每次取出n个不同元素的所有排列的种数除以%个元素的全排列数. 从m个不同元素里每次取出%个不同元素的所有组合的 种数,通常用记号C”来表示.所以,证明上面的定理,也就 ·只要证明下面这个等式成立: C-A P 这里m、见都是自然数,且≤m. 【证]从m个不同元素里每次取出%个元素的排列,可以分成两个步骤来作出:第1步先从m个不同元素里取出m个元素;第2步再把这%个元素进行不同的全排列.根据假设,m个不同元素里每次取出%个不同元素的所 有组合的种数是C,而%个元素的全非列数是P.所以,根 据乘法原则,可知它们的积 OmP 应该等于m个不同元素里每次取出见个不同元素的所有排列 ·23e ==========第29页========== 的种数Am.这就是说, CnPn=Amt .Cm=A P 定理得证 上面这个等式称为组合数公式.这个公式可表成: Ca-盘-mm-122m-n+少1.2.3.…n 这就是说,要计算C的值,只需写出一个分式,使它的分子 是m个连续自然数的积,其中最大的一个数是m,分母是从1开始的%个连续自然数的积 应用上面导出的C的计算公式,就可以方便地解一些 简单的组合问题 例1.平面上有12个点,其中无3点在一直线上.问: (1)这些点可以确定多少条不同的直线? (2)以这些点里的任意3个点作为三角形的顶点,可以作出多少个不同的三角形? [解](1)因为每2点可以确定1条直线,所以所求的直线条数就是12个不同元素里每次取出2个不同元素的所有组合的种数,即 C悦=12x11 1×2 =66. (②)因为每3个点可以确定一个三角形,所以所求的三角形个数就是12个不同元素里每次取出3个不同元素的所 有组合的种数,即C2=12×11×10=220. 1×2×3 答:(1)可以确定66条直线; (2)可以作220个不同的三角形。 。240 ==========第30页========== 文 习题18(1) 1.计算: (1)C; (2)C; (3)C; (4)C: (5)C8; (6)C8: (7)C; (8)Cg. 比较上面(1)与(2),(3)与(4),(5)与(6),(7)与(8)求出的结果, 可以发现组合数C”有怎样的性质? 2.证明: (1)C8+C8=C8; (2).C3+C8=C10, 3.求适合下列等式的自然数n: (1)C=28; (2)C4=A8 4、一次篮球比赛共有8个球队参加,比赛采取单循环制: (1)这次比赛一共要进行多少场次? (2)这次比赛冠军和亚军的获得者有多少种可能情况? 5.空间有10个点,其中无4点在同一平面上:()过每3个点作一平面,一共可作多少个平面? (2)以其中的4个点为顶点的四面体一共有多少个? 象解排列问题一样,解某些条件比较复杂的组合问题,可以应用乘法原则或者加法原则,把它归结为若干个简单的组合问题来解。 例2.小组里有男同学5人、女同学4人,现在要推选男、女同学各2人,组成一个爱国卫生宣传小组,共有多少种选法? 分析这个小组的组成,可以分成两个步骤来完成.第一步,先从 4 5个男同学中选出2人;第2步,再从4个女同学中选出2人.然后并成一组.因为,这两个步骤都完成后小组才能组成,所以要用乘法原则 [解]2个男同学的选法有C种; J 2个女同学的选法有C种, '.小组组成的方法有 ·25· ==========第31页========== C号.C=10×6=60(种). 答:有60种选法来组成这个小组, 例3.小组里有组员9人,其中的2人分别担任正倒小组长.从这9人里,欲派出5人去参加公益劳动,并要求派出的5人中至少有一位组长,共有几种不同的派法? 分析因为派出的5人中至少要有一位组长,所以适合题意的派法有下面这两种情况: (1)只有一位组长在内,另外1人都是组员; (2)有二位组长在内,另外3人是组员, 根据加法原则,只要先分别求出(1)、(2)两种选法,再相加即可. [解)考虑如下两种情况: (1)2个组长中派出1人、7个组员中派出4人,共有派法 CC1=2×35=70(种). (2)2个组长都派去、另外再派3个组员,共有派法 C号.0=1×35=35(种). 所以,总共有派法 70+36=105(种). 答:共有105种不同的派法. 注这问题也可以用另一种方法来解.因为,如果没有至少有一 组长的条件限制,那末显然有C8种派法;中间除去派出的5人都是组 员(派法有C种)这种派法外,余下的派法中就至少有1位组长在内. 所以,只需求C与C的差即可,由此得共有派法 C8-C9=126-21=105(种). 例4.平面上有10个点,其中除有4个点在同一条直线上以外,不再有3点共线。经过这些点,可以确定多少条直线? 分析如果这10个点中没有任3个点在一直线上,那末可以确定 ·26·. ,心 ==========第32页========== 人 C条直线.现在既有4个点在同一直线上,确定的直线就不到C。条, 减少的条数应该就是:这4点不在同一直线上时可以确定的直线条数 (C条)与实际能确定的直线的条数(1条)之差(例如:当点A,B,C,D 这4个点共线时,AB,AC,AD,BC,BD,CD这6条直线事实上只是 1条直线) 【解](1)10个点中如果没有任3点共线时,可以确定 直线C0条 (2)在同一直线上的这4个点,如果也是无3点共线时, 可以确定直线C条、现在,这C条直线只能算做1条 从(1)和(2)可知,经过这10个点可以确定的直线只有 0o-(C4-1)=45-(6-1)=40(条). 答:可以确定40条直线 注解这个问题也可以用另一种解法.我们先把这10个点所确定的直线进行分类 设A,B,C,D这4个点在同一直线1上,E,F,G,H,工,M(图 中只画出其中两点)这6个点在直线?外.由图可以看出,它们所确定的直线有以下3类: (1)由A,B,C,D这4点所确 定的直线只有1条. (②)由A,B,C,D这4点中的1 点,以及不在1上的6点中的1点,所 确定的直线有C4C号±24条 (3)由不在1上的6点中的2点,所确定的直线有C=15条. 所以,总共可以确定直线 1+24+15=40(条). 从例3和例4可以看出:一个条件比较复杂的组合问题,常常可以用不同的方法来解.一般地说,当应用加法原则来解题时,如果适合条件的组合种数容易计算,那末就先把适合条件的组合分成若干类(要注意:不遗漏不重复),计算出各类 ●27◆ ·· ==========第33页========== 中组合种数,再求它们的和;如果不符合条件的组合种数容易计算,那术只需先不考虑条件,算出所有组合的种数,然后减去不适合条件的组合的种数 习题18(2) 1.甲、乙、丙、丁、戊、己这6人中,选出3人出席一次会议, (1)如果甲、乙两人必须在内,有几种选法? (2)如果甲、乙两人都不在内,有几种选法? (3)如果甲、乙两人中有1人且只有1人在内,有几种选法? (4)如果甲、乙两人中至少有1人在内,有几种选法? (5)如果甲、乙两人不能同时在内,有几种选法? [提示:(5)有下面这些情况:甲在内而乙不在内,甲不在内而乙在内,甲乙都不在内.] 女2.在1、2、·、9这9个连续图然数中,任意取出两个数来: (1)积是奇数的取法有几种? (2)积是偶数的取法有几种(用两种方法来解) (3)和是奇数的取法有几种 (④)和是偶数的取法有几种?(用两种方法来解) 3.证明:凸n边形有与n(n-3)条对角线. 4.平面上有10个点,其中除有4个点在同一直线上外,不再有3点共线.以这些点中的每3个点为顶点,可以作出多少个不同的三角形? 5.平面内有12个点,其中有4个点在同一平面内,此外不再有4点共面 (1)这些点可以确定多少个平面? (②)以这些点里的每4个点为顶点的四面体有几个? §19组合数的两个性质 1。组合数的第一个性质在习题18(1)的第1题里, .28 ==========第34页========== 之 我们曾看到: C9=C片,C=C9,Cg=C%,C路=08. 现在来证明:对于一般的组合数Cm和C”,都具有这样的性质,就是 Cm=Cm, 这里m、%是自然数,且m>%.[证]因为 Cm=mm-1)(m-%+1) 1…2…% 又注意到m>%,即m-%是自然数;所以,把上面这个分式的分子和分母都乘以 (m-%)(m-%-1)…3.2.1, 这时,分子就是 m(m-1)(m-2)…(m-n+1)(m-n) ×(m-%-1)…3.2.1=m!, 分母就是 (1.23…n)[1.23…(m-n-1)(m-m)]=n!(m-n)!. 由此即得 C%- m! n!(m-见)!3 这里m、n是自然数,且m>%. 在上式中,用m-%代替见,即得 m! m! C-(mn)![m-(m-n)]!(m-2)!%!·由此可知 COm- 这就证得了这一性质, .●28· ==========第35页========== 注组合数的这一性质,也可根据组合的意义而得到解释.事实上,在m个不同元素中,每当取出%个元素作组合时,余下的m一个元素也可看作是一组合,因此取出%个元素的一切组合的种数也就等于余下的m一n个元素的一切组合的种数,这就说明了C=Cm”, 在上面证明过程中,导出的等式 Ch- m! n!(m-n)! 是组合数Cm的又一表达式,它在证明有关组合数的关系式时很有用 为了使这公式当%=m时也能适用,我们规定0!=1.这时,因为Cm=1,那末1= m! m! m!(m-m)!m0!,可见规定 0!=1是合理的 为了使上面所证得的这一组合性质对于n=m时也成立,通常还规定C9=1,这时,因为Cm=1,那末Cm=1= Cm-"=C9,可见规定C=1也是合理的. 注在m个不同元素里,如果1个元素也不取出,显然只有1种可 能选法,这个事实就可以用符号C来说明. 例1.解方程C8=Cs2 分析要从上面的等式中求出x,只需列出一个关于x的整式方 程。这里等号两边的两个组合数符号的下指标都是18,所以有两种可能 (①)它们的上指标相同.由此可得心=x一2,但是这个方程无解. (②)把C8改用与它等值的符号C8-2来代替,这时从C8=C9 可得 18-x=g-2, 从中即可解出x的值.另一方面,如把C2代以C8红-2)=C8·,则从C=C-*即有x=20-父,由此也可解出x,只是和上面的结果是同样的。 ·.30,m .. ==========第36页========== 解]因为Cs=C8,代入原方程Cs=C2,即得 C18-*=Cs2. ∴.18-花=心-2, ..心=10 将它代入原方程验算…下是适合的,所以原方程的解是c=10. 注意解未知数含在组合数符号中的方程时,必须注意两点: (1)求到的未知数的值,必须使代入组合数符号后有意义. (2)题中给出的组合数符号虽是Cm,不要忽视了如果改用与C”等值的符号C%n时的情况. 2。组合数的第二个性质在习题18(1)的第2题里,还曾看到: Cg+C=C8,C8+C号=Co. 把这一事实推广到一般的情况,就得到组合数的第二个性质: Cm=Cm-1十Ch-. m! 【证]因为.C=n!(m=n!,所以 Om-1+Cm1 (m-1)! (m-1)! [(m-1)n (a-1)![(m1)-(-11! n+(m-1)! =(m-1)I·(m-p+(n-1n n!(m-)!n!(m-2)! =(m-1)(m-n)+=m·(m-1! n!(m-m)! n!(m-见)! m! =n:(m-0)!1 ∴.Cm=Cm-1十Cm二1. 31· 。输 ==========第37页========== 注组合数的这个性质,也可以直接根据组合的意义利用加法原则来证明.事实上,从m个不同元素里每次取出%个不同的元素的组合,可以分成两类: (1)不含某一指定元素的,这就相当于从一1个不同元素中每次取出n个不同元素的组合,所有这样的组合的种数是C-1; (2)一定含某一指定元素的.这就相当于从一1个不同元素中每次取出”一1个不同元素的组合,所有这样的组合的种数是Cm. 由此,根据加法原则,得 Cn-1十Cm-1=Cn. 、例2.求证Cm+1+C-1+2Cm=C+ 【证]应用上面的组合数的第2个性质,我们有 C+1+Cm1+2Cm-00+01+Cm+Cm =Cmt1+Cm+Om+O1 -Cm+O+1 =Cn+. 例3.求证C班+Cm+1十C册+g十…十Cm+k-1=Cm。 【证]由上面的组合数的第2个性质,有 Cm-O, C1十Cm+1=Cm+拉, C十C册+2=C轻, Cm杜-1十C州+k-1=C+无. 将以上各式分别相加,得 Cm+C1十C+1十C技十Cm+8十…十C转-1十C孤+k-1=C,1+C2十C班格+…十C+-1+Cm+.经过整理,抵消同类项后即得 Cm+C别+1十Cm+9十…+C+k-1 =C老。 、324 ==========第38页========== 习题19 1.计算: (1)C88; (2) :·2.解方程: (1)C8=C结”; (2)C器=C器1. 3,求证Cm+2=Cm+2Cm1+Cm2 4.求证Cm-1+Cm-2+Cn-3十…+Cn+1+C=Cm+1, 5.求证Ca=n+CH. m+1 6.求证C9n+Cn+1+C%+2+Cn+8十…+C+-1=Cm+,[提示:利用组合数的第1个性质和例3的结论.] §110排列、组合综合应用题 下面我们再来看几个需要综合运用关于排列、组合知识来解的应用题 例1.有不同的书6本, (1)平均分给甲、乙两人,有几种分法? (2)平均分成两堆,有几种分法? [解](1)不妨让甲先任意取3本,余下的就给乙,所以共有分法 6×5×4=20(种). 1×2×3 (2)对于某3本书,分给甲或者分给乙要算成两种不同分法,但是分成两堆只能算做同一种分法,所以所求的分法应该把(1)的分法再除以2,由此得共有分法 ·33· 斋 ==========第39页========== C一20=10(种). 22 答:(1)有20种分法;(2)有10种分法 注意平均分组而不必考虑组的先后时,可以先假定按照组的顺序来分,然后再除以组数的全排列, 例2.将4个男孩子和4个女孩子分成两组,进行混合双打乒乓赛,不同的搭配方法有几种? 【解】先把4个男孩子平均分成两组,有分法罗-3 种. 把这4个男孩子看作各占据1个位置,再把4个女孩子对这4个位置进行搭配,这只需把4个女孩子作各种不同排 列,排法有P4种 这两个步骤都完成后,搭配才能完成,所以共有搭配方法 C.P4=3×24=72(种). 2 注意想一想,上面这个解法是怎样进行分析的?除掉这一解法外,还有什么不同的解法? 习题110 1.有不同的书6本,分给甲、乙、丙3人,如果要使 (1)甲得3本,乙得2本,丙得1本; (2)一人得3本,-·人得2本,一人得1本; (3)每人各得2本,各有几种分法? 2.有不同的书6本,如果要分成三堆,使 (1)一堆有3本,…堆有2本,堆有1本 (2)每堆各有2本; (3)两堆各有1本,各有几种分法? ·34●. ==========第40页========== *3.把4个男同志和4个女同志平均分成4个小组,到4辆公共汽车里参加售票劳动.如果同样两人在不同的公共汽车上服务算做不同的情况: (1)有几种不同的分配方法? (2)每个小组必须是一个另同志和一个女同志,有几种不同的分配方法? (3)男同志和女同志分别分组,有几种不同的分配方法? [提示:(3)先把男女同志各分成两组,再对这4个小组作排列.] *§111概 率 排列、组合知识的重要应用之一,是求某一事件出现的概率问题.概率论是数学里一个独特的分支,它在生产实际中以及科学技术研究中都有广泛的应用.这一节将简单介绍一下什么叫做概率,以及应.用排列组合知识来求某一事件出现的概率的方法. 我们先来看下面的问题. 问题从1、2、3、4、5、6、7、8、9这9个数字中,任意取出两个数字, (1)有几种取法? (2)其中和是奇数或者偶数的取法各有几种? (3)和是奇数或者偶数的取法各占全部取法的几分之几? 很明显,问题中的(1)是求9个不同元素中每次取出2个不同元素的所有组合的种数,所以共有取法 0房-1” 9×8=36(种). 问题中的(2),曾在习题1·9(②)里解过,和是奇数的取法有 CgC1=20种, 和是偶数的取法有 C8-CC1=36-20=16(种). 由此容易算出: 和是奇数的取法占全部取法的,就是?,20 ·35· ==========第41页========== 和是偶数的取法占全部取法的品,就是手· 从上面的计算结果可以看出,取出来的两个数的和是奇数的可能性要大一些.为了说明这一事实,我们引用概率这一概念.我们说:从 1,2,,9这9个数字中,任意取出2个数字,和是奇数的概率是日和不是奇数的概率是手。 在这个问题里,因为这两个数是任意取出的,所以取出的两个数或是(1,2),或是(1,3)等等,其可能性是一样的;并且,取出了(1,2)就不能取出(1,3)。这样的两种取法,称为等可能的、两两互斥的基本事件(或者试验结果).很明显,在这个问题里,一共有36种基本事件,其中有20种基本事件适合于“和是奇数”这一事件的出现.所谓“和是奇数”的概率,事实上也就是指适合于“和是奇数”这一事件出现的基本事件的种数与总共基本事件的种数这两者之比. 一般的,可以给出如下的定义:设在某种条件下进行试验,一共有 N个等可能的、两两互斥的试验结果(基本事件),其中有k个是适合于 事件A的出现的。那末,A的概率是急,记作 24)=京 例如,在上面的问题里,把“和是奇数”看成是事件A,“和是偶数” 看成是另一事件B,那末有 P(A=号,5 PB告 如果问题里只要研究某一种事件出现的概率或者不出现的概率,常常用字母p来表示这一事件出现的概率,字母q表示这一事件不出,现的概率。 例如,上面这个问题里,对于“和是奇数”的事件来说 m品 很明显,p与之间存在着以下的关系: 2+91, 就是 q=1…p ·38· ==========第42页========== 理解了上面所讲的关于概率的意义,就可以应用前面学过的知识来解一些简单的概率问题 例1·箱子里有100个零件,其中混进了2个次品,现在顺手取出5个零件来检验,求次品恰巧都被检到的概率. 【解】在100个零件里任取出5个的取法有C种,取出的5个 零件中恰巧有2件次品的取法有CC种。所以,次品恰巧被检到的 概率是 98×97×96 p=iC 1×2×3 C5oo 100×99×98×97×96 1×2×3×4×5 201 9900495 习题1·11(1) 1.在例1中,求取出的5个零件中: (1)只有1个是次品的概率; (2)至少有1个次品的概率. 2.在50根纤维中,有16根长度超过30毫米,从这些纤维里任意取出一根,求: (1)这根的长度超过30毫米的概率;(②)这根的长度等于或不足30毫米的概率, 3.有12齿和18齿的齿轮衔接在一起旋转,其中各有一齿磨损,现准备进行检修,求拆下时: ()恰巧两个磨损的齿衔接在一起的概率;(②)衔接的两齿中至少有一个是磨损的齿的概率. 4.有热水瓶60只作为处理品出售,其中48只是二等品,其余是 三等品.现在从这些热水瓶里任意取出两只,求: (1)这两只热水瓶都是等品的概率; (2)这两只热水瓶中二等品三等品各有一只的概率。 ==========第43页========== > 下面再来看一些条件比较复杂的概率问题的解法 例2.一盒螺钉20个中有16个是合用的,另一盒螺母20个中有15个是合用的.现在从两盒中各取一个螺钉和螺母,求两个都合用的概率. 分析按照题意,取用时有两个步骤,每一步骤都成功才能达到目的.因此,只需先算出每一步骤成功的概率,再应用乘法原则求它们的积 16 [解1取出的螺钉是合用的概率:1=20· 取出的螺母是合用的概率: 20· 于是,两个都合用的概率: 16、153 p=21P=20×20=5· 例3,有20个零件,其中有12个是合用的.现在每次取出一件来检验,求前两次所检验的零件: (1)都是合用的概率; (2)第一个不合用,第二个才合用的概率 12 【解】()第一次检到的零件是合用的概率:1=0 11 第二次检到的零件是合用的概率:一9,(想一想, 这是为什么?) 所以,所求的概率是 121133 p=・2=20×1995・ 8 (2)第一次检到的零件不合用的概率:1=1一”1=20·第二次检到的零件是合用的概率:=号.〈 (想一想:这是 为什么?) 所以,所求的概率是 81224 卫=917=20×19-95· ==========第44页========== -礼 习题111(2) 、 1.一个事件发生的概率已勿是子, (①)这个事件不发生的概率是多少? (2)在连续两次试验中,这个事件都发生的概率是多少? (3)在连续两次试验中,这个事件第一次不发生而第二次发生的概率是多少? (4)在连续两次试验中,这个事件至少发生一次的概率是多少? 2.制造一种零件要经过三道工序,第一道工序出废品的概率是 0.5%,第二道工序出废品的概率是1%,第三道工序出废品的概率是 1.5%.如果这三道工序的生产情况是互不影响的,那末这三道工序完成后制造出的一个零件是合格品的概率是多少? 3.一个气象站天气预报的正确性达95%,求三次预报中:(①)只有两次是正确的概率; (2)至少有两次是正确的概率. 4.一个工人同时看管5部机器,在1小时内每部机器需要照顾的 概率为,求在1小时内: (1)没有一部机器需要照顾的概率; (2)至少有4部机器需要照顾的概率。 本章提要 1.基本原则 (1)乘法原则一一事的完成可分n个步慰每一步骤各有方法m1,m2,…,mn种,完成此事的不同方法种数是 N=m1'm2…mn. (2)加法原则一一一事完成可有”类不饲方法,每一类中又分别有,m,…,mn种不同方法,完成此事的不同方法种数是 , N=m+m十…+mn。 9· ==========第45页========== 小、 2.基本公式 (1)m个不同元素中取”个元素的排列种数: 不许重复时:An=m(m一1)…(m-n+1);特例:Am=Pm=mI;可以重复时:N=m”, (2)m个不同元素中取%个不同元素的组合种数: C-、4 m! 卫Pn n!(m-n)! (3)组合数的两个性质: Cn=Om-n Cn=Cn-1十Cm-. (④)简单事件的概率P(A)=奈, 复习题一 1.求证P1+2P2+3Pg+…+nPn=(n+1)l一1. 2.(1)已知C+3=C+1+Ch+1+C-2,求m; (2)已知C:C%=44:3,求; (3)已知C=C2,求C; (4)已知Cm1=Cm+,求证m=1+2+3+…+n. 3.用数字0、1、2、3、4、5组成没有重复数字的数, 东 (1)能够组成多少个自然数? (2)能够组成多少个六位的奇数? (3)能够组成多少个是25的倍数的四位数? (4)能够组成多少个比201345大的数? 4.8个人排成一排,如果: (1)其中某2个人要排在一起; (2)其中某2个人不排在一起; (3)其中某4个人要排在一起、另外4个人也要排在一起问有多少种不同排法? 5,用1、2、3、4、5、6六个数字, g40。 ==========第46页========== 义 (①)如果数字不允许重复; (2)如果数字允许重复,可作成多少个五位的不同奇数? *6,某人过去射击的成绩,每射5次总有4次可以射中目标。根据这一成绩,求: (①)射击3次全部射中目标的概率? (②)射击3次中有2次,并且只有2次射中目标的概率?(③)射击3次中至少射中3次的概串? p 스 ==========第47页========== 第二章数学归纳法 数学归纳法是数学中的一种非常重要的证明方法,许多命题的证明都要用到这种方法.本章将在说明数学归纳法的意义的基础上,着重学习数学归纳法的应用. §2·1归纳推理和演绎推理 在数学里,经常要使用推理方法,从一些已知正确的判断,作出新的判断. 例如,在研究等比数列的通项公式时,曾进行过这样的推理: 设a1,ag,a3,…,n,…是一个公比为q的等比数列(a1≠0,q≠0),那末,根据等比数列的定义,知道 =g,.ag=419; as 3=g,.43=a2g =a1g2; W 04二q, %4=a3g=a1q; a3 观察上面这些等式,可以看到:等比数列第二项起的各个项,都可以写成第1项1和公比g的幂的乘积;并且,在这个积里,q的幂的指数就等于这一项的项数减去1,就是 ag=a1q2-1,a8=1g3-1,4=q4-1,…. ==========第48页========== 之 据此可以归纳出,这个等比数列的第项一定是 am=agm-1(%=2,3,4,…). 在这个公式里,如果令%=1,那末可得 a1=a1g2-1=a1g°-1 所以1也可以应用这个公式来表示.由此可以得出定理: 第1项是a,公比是g的等比数列的通项公式是 an=a1g"-1, 式中a1≠0,g≠0,%为自然数. 这里,就是从等比数列的定义出发,通过推理的方法,推出了一个新的判断一等比数列通项公式 又如,在研究一元二次方程的根的性质时,我们知道,对于实系数的方程 ax24bg+C=0, 产 当它的判别式b2-4c<0时没有实数根.而方程 G2+心十1=0 的判别式 b3-4ac=1-4=-3<0 因此可以作出结论:方程x+心十1=0没有实数根。这里也是应用了推理的方法. 考察上面所举的两例子的推理过程,可看出两者有着不同的特点.在前面这个例子里,首先是考察一些特殊的事例,然后分析它们共同具有的特征,作出了一般的结论.象这种由特殊到一般的推理方法,通常称为归纳推理,或者归纳法。在后面这个例子里,从一般的实系数一元二次方程的根所具有的性质,推出了特殊的实系数一元二次方程x+c十1=0的根所具有的性质。象这种从一般到特殊的推理方法,通常称为演绎推理,或者演绎法, ·48● ==========第49页========== 应该注意,应用演绎推理时,只要在推理过程中不发生错误,那末从已知正确的判断所作出的新的判断,也总是正确的、但是,应用归纳推理,却不是这样。 我们来看下面的例子 设f(c)=2+心十11.现在取心=1,2,…,9等自然数值来计算f()的值.得 f(1)=1+1+11=13, f(2)=22+2+11=17, f(3)=32+3+11=23, f(4)=42+4+11=31, f(6)=6+5+11=41, f(6)=62+6+11=53, f(7)=72+7+11=67, f(8)=82+8+11=83, f(9)=92+9+11=101 可以看出,上面所算出的这些值都是质数.因此,可以得出一个结论: 当c是1到9的自然数时,函数f(心)=x2+十11的值总是质数. 很明显,这个结论是正确的 但是,是不是也可以象开始时所举的那个例子一样,归纳出一个更加一般的结论:“当x是任意自然数的时候,(心)一x2+十11的值总是质数”呢? 只要取:=10来验算一下, f(10)=10+10+11-121=11, 它就不是一个质数,所以这个结论是错误的 从这个例子可以看到,应用归纳推理的时候,如能把所有 ==========第50页========== 的情况都列举出来,由此得出的结论可以保证一定是正确的.这种证法通常叫做列举法(穷举法).但是对于只从一些个别的情况所归纳出来的一般的结论(这种推理称为不完全归纳法)就不一定正确。 习题2◆1 1.无穷数列的前5项是:,a4(4-l,4=素-8,5,4102a1=1号,=1,a4=名,4=,s=0.3 观察这些已给出的项的特点,写出一个用项数”来表示它的通项,的公式. [解法举例:(1)a2,a,a,s的分母都比项数大1,而分子是项数 的2倍并且4=1-号-也具有这样的特点,所以这个数列的 通项是a= 2n n+7] 2.考察下面这两个数列所给出的各项与项数间有怎样的关系?由此写出它们的通项公式: (1)1,4,9,16,5,…g (2),13,37.81151 2’3’4’5)6,… §22数学归纳法 上节曾指出,应用不完全归纳法,考察一些特殊情况所归纳出来的一般结论,是不一定正确的.因此,它仅仅提出了一种合理的设想,这个设想是否能够成立,还需要作进一步的证明 例如,在自然数列里,我们来考察从第一个奇数开始的各 ·456 ==========第51页========== 个连续奇数的和.注意到: 1=1=12, 》 1十3=4=22, 1+3+5=9=32, 1+3+5+7=16=42, 1+3+6+7+9=25=52, 发现了一个重要的事实:自然数列里前1个,2个,:,5个连续奇数的和,恰巧都等于这个和里加数个数的平方.很自然会作出这样的一般结论: 自然数列里前n个连续奇数的和等于 但是,这里毕竟只考察了少数几种情况,发现有这样的规律,这个结论是不是对于任意的自然数%都正确呢?这就必须作进一步的证明. 怎样证明这个结论确实是正确的呢?企图用一一验算的办法,显然是不行的.但是,如果能够证得“当和式里连续奇数的个数是某一个自然数(例如当%=)时这个结论正确,可以推出对和式里连续奇数的个数再增加1个(例如,当=+1)时也一定正确”这一事实,这时,就可以应用递推的方法,从这个结论对于m=1正确,而推出对于n=2也正确;对于见=2正确,推出对于%=3也正确;这样顺次地推下去,显然,这个结论对于所有的自然数儿都正确. 我们就采用这样的思想方法来证明上面这个结论.因为任何一个奇数都可以表示成2%一1(m是自然数)的形式,所以欲证明上面这个结论,也就是要证明等式 1+3+5+…+(2m-1)=n2 (1) 对于所有的自然数%都成立. [证]当m=1时,有1=1,所以式(1)成立. 046· ==========第52页========== 假定当见=时等式(1)成立,即 1+3+5+…十(2-1)=2 (2) 成立 那末,当%=飞+1时,式(1)的左边是1+3+5+…+[2(k+1)-1] =[1+3+5+…+(2k-1)门+[2(+1)-1]=2+(2k+1) [应用式(2)] =(十1)2 而这时式(1)的右边也是(飞+1)2,所以式(1)当%=+1时: 左边=右边. 这就是说,式(1)当%=飞+1时也成立 这样,因为已验证了等式(1)当%=1时成立,可以顺次地推出当%=2,3,4,5,…时都成立,所以式(1)对于所有的自然数m都成立. 上面证明中所采用的方法,称为数学归纳法.从证明中可以看出,应用数学归纳法来证明一个命题,有下面这两个步骤: 1°先证明当命题中%取第一个自然数值a(例如n=1,或者%=2,等等)时,这个论断是正确的. 2°假定命题中见取某一自然数值k时这个论断正确;在这基础上证明命题中%取后一个自然数值飞+1时,这个论断也正确 在证实了这两步之后,就可以作出结论:“命题对于从a开始的所有自然数%都正确.” 在实际解题时的叙述可以比上面证明中的叙述再简化一些 例1.证明 ·47 ·2避 ==========第53页========== 1+2+22+…+2n-1=2"-1, (3) [证]1°当n=1时,左边=1,右边=22-1=1,式(3)显然成立, 2°假定当肌=时式(3)成立,即 1+2+22+…+2-1=2-1, (4) 那末,由 (1+2+22十…+2k-1)+2k+1)-1=2*-1+2k [应用式(4)] =2.2%-1 2k+1一1 可知,式(3)当=+1时也成立. 根据1°和2°,这就证得了式(3)对于所有的自然数%都成立。 例2,证明 1-3+6-7+…+(-1)"-1(2m-1)=(-1)"1.%.(6) 【证】1°当%=1时,式(6)的左边=1,右边=(-1)°.1=1.所以式(5)显然成立. 2°假定式(5)当=k时成立,即 1-3+6-7+…+(-1)-1(2k-1)=(-1)k-1.k.(6)那末,当%=+1时, 左边=[1-3+5-7+…+(-1)-1(2k-1)] +(-1)k+1)-1[2(k+1)-1]=(-1)k-1.k+(-1)(2k+1) [应用式(4)] =(-1)*[(2k+1)-] =(-1)*(k+1), 右边=(-1)+1)1(+1)=(-1)*(张+1), 。左边=右边 ·48 ==========第54页========== 所以当n=层+1时式(5)也成立. 根据1°和2°,可知式(⑤)对于所有的自然数%都成立.注1.上面例1和例2在第2步的证明中,叙述方法虽然不同,但精神是一样的.一般来说,如果待证式子的右边比较简单,可以采用例1的叙述方法;待证式子的右边比较复杂,那末可以采用例2的叙述方法. 2.在例1和例2的证明里,在第2步都先提出了“假定待证式子当%=k时成立”,这通常称为归纳法假定,下一步在证明当》=k+1时待证式子也成立中必须用到这个假定. 习题22(1) 应用数学归纳法,证明以下各题: 1.1+2+3+…+n=n(+1)2 2.1+3+9++3m-1=3-1 2 3.12+2.3+3.4+…+n(%+1)=3(n+1)(m+2). 4.1.2.3+2.3.4+3.4.5+…+n(n+1)(m+2) -号m+1a+2)6a+3). 5.1-2+4-8+16-…+(-1)m-2-1=(-1)m-12”+ 6.式+능+ + a -1)(2n+)・1 2 例8.证明 18+28+…+n3=(1+2+…+n)2 (7) [证】根据等差数列的前n项的和的公式,可知 1+2+3+十n=n(%+1)2 所以,本题只需证明 49· …落 ==========第55页========== 1+243*++-[a(a生2]. (8) 十 1°当%=1时,式(8)显然成立.2°假定当%=时,式(⑧)成立,即 1+2+3++=[2], (9) 那末,当2=+1时,式(8)的 左边=(13+23+33十…+3)+(k+1)8 -[++ [应用式(9)] =专2[+4+1刃 -[++2], 有边=[+》+2, ‘.左边=右边 所以当%=十1时式(8)成立. 根据1°和2°,可知式(8)对于所有的自然数%都成立, 从而可知式(T)对于所有的自然数见也都成立. 例4.当%≥2时,证明(a1千g十…+an)a =(ai+a+…+a)+2(a1u2十i1g+… +a1an十ga8+a4十…十a2an十…+am-1an).(10)[证】1°当%=2时,左边=(a1+a)8=1+2a1a+a吃,右边=a1+贻+2a1a2,所以式(10)成立.2°假定当%=k时式(10)成立,即 ·50· ==========第56页========== 《 (1+ag+…+ax)3 2 =(a1+a5+…+)+2(1a2+a13+… +a1ak+asaa十asa4+…+asae十…+ak-iax),(11) 那末,当凯=+1时,式(10)的 左边=[(a十ag+…十ak)+ak+1]2 =(a1+ag+…+ax)2+2ak+1(a1+ag十… 十ak-1十k)十a绿+1 ={(a1+a+…+绿)+2(aa2十1g+…+aak 十agag十a2a4+…+agak+…+ak-1ak)}+2a1ak+1十2aek+1十…+2a%-a%+1+2akak+1+a呢+1 [应用式(11)门 =(a+a吃+…+a2+眠+i)+2(a1ag十aa3十…+auk+a1☑%+1+a2ag+ag&4十…+aga%十a2k+1+… +Hax-1ax+ak-iak+1+akax+1), 这恰巧与式(10)的右边相同,所以当见=+1时式(10)也成立. 根据1°和2°,可知式(10)对所有大于1的自然数%都成立 说明1.在这个例子里,题目上指定n≥2,所以第1步要从=2开始检验 2.式(10)右边第2个括号里的 1a2+3+…+c41gn十a2ag+十d2d4+…+2am+…+n-1any就是1,2,Qg,…,an这n个相异元素中每次取2个相乘而作出的各种可能组合的和. *从上面这些例子里可以看出,用数学归纳法来证明一个命题时,在作出最后结论前,总是有着这样的格式: 1°当n=a时,命题成立; ·51◆ ==========第57页========== 2°假定当n=时,命题成立,即…,证明当=+1时,…,命题也成立 正 这里,第1步只要直接进行检验,第2步则要根据题目的具体内容,利用归纳法假定来进行推导 必须注意,上面这两个步骤中,第1个步骤是递推的基础,第2个步骤是递推的依据,缺少任一步骤都是不行的.不然就会得出错误的结论. 例如,知果不考虑递推的依据,只验证了当x=1时,甚至x=2,3.…,9时, f(x)=x2+x十11 的值都是质数,就作出:对于所有的自然数x,(x)的值是质数的结论就错误了. 又如,在例1里,如果不先验证:当%二1时, 1+3+5+…+(2m-1)=%2+1 这一等式是否正确,直接考虑第2个步骤: 假定这个等式当=k时成立,就是1+3+5+…+(2-1)=2+1,那末容易推出 1+3+5+…+(2k-1)+(2%+1) =(e2+1)+2k+1 =(k+1)2+1, 这个等式在=+1时也成立.从而会作出错误的结论:“自然数列里前n个连续奇数的和等于22+1”. 习题22(2) 用数学归纳法,证明: 1.首项是a1、公差是d的等差数列: (1)通项是an=h+(-1)d; (2)前n项的和是Sn=a1+%(0-1)d: 2.首项是(a1卡0)、公比是q(q卡1)的等比数列: (1)通项是an=412-1; ●52● ==========第58页========== (2)前%项的和是Sn=a1(1-g”) 1-q 3.自然数列里,前n个自然数的平方和: 1 n=6(%+1)(2%+1). 122 4.如果一个数列的通项是2以-)2十那末这个数列前n项的和是+号 5.自然数列里前N个连续奇数的平方和是寻n(2+1)(2-1.§2·3数学归纳法在证明不等式中的应用 在证明某些对于所有自然数(或者,从某一个自然数起的所有自然数)都能成立的不等式中,也常常应用数学归纳法, 例1,求证 が>() (1) 这里a>0,b>0,%是自然数. 【】1°当n1时,显然有告色-生,这时式 中等号成立 当%=2时,式(1)的左、右端分别为: 左ど,右过-(2) 这时,因为 a8+b 2 -(2告广-0425.+2g+4 4 -8-2a6+62-(a-b)2≥0, 4 2.sど() ·53· ==========第59页========== 所以式(1)当=2时也成立. 2°假设式(1)在n=k(k≥2)时成立,即 t (.2 (2) 因为题设>0,>0,所以a十0>0,在式(2)的两边同乘以 2 ,得 ()告)=(生2), 就是 "+如+(巴9)”. (3) 4 ∵aコ+b_aき++aが+ba+*·1 4 ak+コ1-ba++1-ab 4 -(a-b)a*-(a-8)6k 4 =(a-b)(a*-b), (4) 4 这里,因为a,b都是正数,所以 当a>b时,有a*>b,a-b,a-b都是正数;当u0, 4 又,当a=b时,显然有a-b)a-b-0.由此可知 4 "(ど)()=()" ◆54· ==========第60页========== 这就是说,当n=+1时,式(①)也成立. 根据1°和2°,即可断言:原式对于任意的自然数%都成立 注意1、第1步的证明虽然比较简单,但是也应注意,如果只验证2=1时的情况,只能说明式(1)的等号成立,所以还必须验证n¥2时的情况 2.筠2步的证明中,如直接比较+”士1与()的大2 小,是比较困难的,所以采用了在承认(①)式的前提下,两边同乘以一个 正数“生(注意:在这一步中必须证明“士”>0),这样,不等式的右边变皮(色士护)“就与所要证的一致,留下来只张证男41b1 不 小于()(生)就可以了.由此可以看出,应用数学归纳法证 明问题时,要善于应用归纳法假定,作合理的推导.同时,在叙述上也不宜拘泥于一格, 3,对于这个例题,读者还可考虑一下: (1)如果a>0,b>0,a*b,n为自然数,那末"+b(色”将有怎样的关系? (2)如果题设条件改为a<0,b<0,n为自然数,这个不等式是否仍能成立?如果缺掉了a>0,b>0这一条件呢? 例2.求证:当%是1,或是不小于5的自然数时,总有 2">2. (5) 分析第2步证明中,需要从(归纳法假定)2>(>5)推出 2+1>(k+1)2 从归纳法假定可知2*+1>2k2,因此,要证明不等式2+1>(k+1)3成立,只需证明>(k+1)2,即 k>2k+1, 即 >2+君 ·55· ==========第61页========== 而当&≥5时,2+是<3,所以>8+名显然成立 【证】1°当%=1时,左边=2,右边=1,左边>右边,式 (5)成立.当%=5时,左边=2=32,右边=52=25,左边>右边,式(⑤)也成立. 2°假设式(5)当=k(聚≥5)时成立,即 2>k2(k≥5). (G) 两边各乘以2,得 2+1>2k2. (7) 从已知条件≥5,可准得2+是<3,从西有 B>2+是 两边同乘以,得 k2>2k+1, 两边再同加上2,得 22>(飞+1)2. (8) 由式(7)和(8)得2+1>(居+1)2 这就是说:当%=形+1(飞≥5)时,式(5)也成立. 根据1°和2°,可以断言:当%=1,或者m是不小于5的自然数时,式(5)是恒成立的 注意想一想:在上面的证明中,从归纳法假定2*>2推出2*+1>2识,只需加上一个条件≥3(这条件在证明>2+无时要用到就够了.由此,能不能把这个不等式成立的范围扩大到对于一切不小于3的自然数都成立? 习题23 1.用数学归纳法,证明: 。56 ==========第62页========== 么 (1)(1+h)”>1+h(h>-1,h≠0,h是大于1的白然数); (2)2”>n(%是自然数). 2.(1)证明:当k≥4时,3>32+3+1(是自然数); (2)要使不等式 2n>n3 成立,可取哪些自然数的值? [提示:=1是显然成立的,除此之外,应先通过直接检验,着取哪一个最小的自然数值才能使这个不等式成立,然后应用数学归纳法证明%取比这个自然数大的一切自然数都能使这个不等式成立.] 本章提要 应用数学归纳法证题的一般步骤 1°先验证n取第一个自然数值a时命题成立(这是递推的基础);2°再作出归纳法假定:“设n=(飞≥α)时命题成立”,然后利用这个假定以证明:“当%=k+1时命题也成立”(这是递推的根据) 证实了这两点,就可作出结论:“对于从a开始的所有自然数”,命题都成立”(此即是在1°的基础上利用2°的结论进行递推). 复习题二 1.用数学归纳法,证明: (1)1+x+2+…+x”=1-xn+1 1x (x≠1); (2)ユ+3-+6++(1)- (n +1)(n+2); 2 6 11 n(a+3) (3)1.23+2.34++n0+1m+2)=4m+)0+2) (④1-票+1)++(-)2e-1)8+1D2 nl =(-1)n(c-1)(x-2)…(c-) 2! 2.用数学归纳法,证明: ·57· ==========第63页========== 》 ()如果一个数列的首项是,并且从第二项起的每一项都等于前 一项除以这一项的项数,那末它的第m项一定是号; (2)如果一个数列j的第1项是1=1;并且从第二项起,相邻两项间有下面的关系: an=3a-1+2(n≥2), 那未这个数列的通项公式一定是 an=3”+3m-1-2 2 3.要使不等式 2m>2m+1 成立,m可取哪些自然数的值? ◆58· ==========第64页========== 和.中,雾得 第三章二项式定理 在本丛书代数第一册里,曾经讲过二项式a十b的平方公式和立方公式.本章将导出二项式a+b的任何正整数次幂的公式一二项式定理,并研究二项展开式的一些性质以及它们的应用. §3·1杨辉三角形 我们已经知道: (a+b)1=a+b: (a+b)2=a2+2ab+b2,(a+b)3=a3+3a2b+3ab3+b3 对于+b的其他正整数次幂,是不是也可以不通过乘法运算,直接把它的展开式写出来呢? 不妨再进行一些计算,得到a+b的4、5、6次幂是: 人 (a+b)4=a4+4a3b+6a62+4ab3+b4; (a+b)5=a3+5a4b+10a3b'+10a2b3+5ab4+b5:(a+b)6=a°+6a5b+15a4b2+20a3b3+15a2b4+6ab5+b8现在来观察这些展开式里各项的字母因式,从中发现一些特点. 如把展开式中单独由一个字母α(或者b)的幂构成的项,看作是这个字母的幂与另一个字母的零次幂的积(例如,把4看成Q4b°,b4看成ab),那末,可以看出,这些展开式里各项 ·59。 ==========第65页========== 的字母因式有两个重要的特点: 1°展开式的每一项中,字母a与b的幂指数的和,恰巧都等于左边二项式a十b的幂指数 2°展开式里,字母a的幂指数是逐项减小的,第一项中的指数与左边二项式a十b的幂指数相同,以后便逐项依次减少1,直到0为止;字母的幂指数是逐项增大的,第一项中的指数是0,以后便逐项依次增加1,直到与左边的二项式的幂指数相同为止. 把这两个特点概括起来说,也就是:二项式a十b的%次幂(这里%=1,2,…,6)的展开式是按照字母a的降幂而排列的、关于字母a和b的%次齐次式. 进一步来考察,展开式里各项系数间的关系,看有什么特殊的组成规律.为此,把它们的系数分别列成下表的形式: (a+b)1 11 (a+b)2 121 VV (a+b)3 1331 VVV (a+b)4 14641 VVV V (a十b)5 15101051 VVVVV (a+b)8 1615201561 从这表可以看出,二项式a十b的各次幂的首末两项的系数都是1,而中间各项的系数恰巧都等于它的肩上两个数(就是上 一行中位于它的上方的两个数)的和.例如,(a+b)4的展开式中各项的系数:首末两项都是1,从第二项起的中间各项系数4、6、4,顺次是它的肩上两数1与3、3与3、3与1的和.倘使继续把(a十b)?、(a十b)s的展开式写出,可以发现它 ·60 ==========第66页========== 本 们的展开式也同样具有上面所说的这些特点.例如 (a+b)?=a7+7ab+21ab2+35a4b3 +35a3b±+21a265+7ab8+b7; (a+b)8=a8+8a7b+28az2+56ab3 +70a4b4+56a3b5+28a2b6+8ab7+a8 早在1261年,我国宋朝数学家杨辉所著的“详解九章算术”一书里就已经出现了上述形式的表.由于这个表的外形很象一个三角形(如果上面再加上a+b的零次幂的系数1的话),所以把它称为杨辉三角形 应用杨辉三角形,可以写出a+b的任意正整数次幂的展开式,方法是: 先根据二项式a+b的幂的指数,写出次数与它相同的关于字母a和b的齐次式; 再根据杨辉三角形中相应这一行的各数,顺次写出展开式中各项的系数. 注杨辉曾经指出,他这个方法出于“释锁算术”,并且说,我国古代数学家贾宪已经用过它.所以,我国发现这个表不迟于十一世纪.在欧洲,认为这个表是法国数学家巴斯加(1623~1662)首先发现的,把它称作“巴斯加三角形”,其实比我国要迟500年左右. 例1.写出(2c-3g)5的展开式. 分析把2x看成是a,-3y看成是b,代入(+b)°的展开式,然后把它化简即可. [解](2x-3y)5=[(2c)+(-3则)]5 =(2)ǒ+5(2c)4(-3g则) +10(2ac)8(-3gy)2+10(2c)2(-3y)8+5(2x)(-3y)4+(-3y)=32a-2404则+720xy2-1080x2y8+810c-243y°. 、 心活游 ==========第67页========== 例2.求(3a+号)的展开式中a4的系数.分析只要把(a-》的展开式写到含有α4的一项,那末这项 的系数即是所求. [解] (3a+号)°-8a+63@)(层)+15(3a(爱》°+, 所以,所求4的系数是 15×3×(层'-15×81×青-540. 习题31 1.写出: (1)(4+b)°的展开式的前5项;(②)(a+b)10的展开式的前6项。 2.应用杨辉三角形,展开: (1)(2ac-y); (2)(2ax+y)5. 比较这两个展开式里的各同类项的系数,可以看出它们之间有怎样的关系? 3.(1)求(2一x)7的展开式中3的系数; (②)求(e+》的展开式中的嘴数项。 4.根据杨辉三角形,指出: (1)(a+b)3,(a+b),(a+b)7的展开式中系数最大的项分别是第几项? (2)(a+b)2,(a+b)4,(a+b)6,(a+b)8的展开式中系数最大的项分别是第几项? 从上面的观察中,可以发现:在(+b)的展开式中,系数最大的项的项数k与间有怎样的关系? ·62· ==========第68页========== 5.根据杨辉三角形,计算(a+b)2,(a+b)3,(a+b)4,(a+b)5的展开式中各项的系数的和是什么?由此可以发现,(a+b)”的展开式中各项系数的和与指数”间有怎样的关系? §3·2二项式定理 应用杨辉三角形来确定(α+b)”的展开式中各项的系数,首先要知道(a十b)-1的展开式中各项的系数.为此,又得先知道(@十)"-2的展开式中各项的系数;这样逐步倒推.上去.例如,要确定(a+b)14的展开式中各项的系数,在第60页这个表的基础上,还必须继续顺次写出(a十b)7,(a十b)8,…,(a+b)18,(a+b)14的展开式中各项的系数.自然,这样做是比较麻烦的. 现在进一步来研究展开式中各项的系数与指数%间有怎样的关系,从而找出一个能够直接写出(α+b)”的展开式中各项的系数的法则.为此,先来研究下面这个间题. 问题求(a+b1)(a+b)(+b)…(a+bm)的展开式对这个问题,可以利用第二章学过的知识,用数学归纳法来解. 首先,直接进行计算,求出见=2、3、4时的展开式: 心 (a+61)(a+62) =a2+(01+b2)a+0102i (a+61)(a+62)(a+63) =a3+(b1+b2+b3)a2+(b1bg+b1b3+bb3)a+b1bb3;(a+61)(a+62)(a-+63)(a+64)=a4+(b1+bg+b:+b4)a3 +(0102+6103+6164+6283+6204+8304)a2+(016283+018204+016304+020304)a+01030304. ·63· ==========第69页========== 可以看出,展开式有下面的特点: 展开式是字母a按降幂排列的多项式,的最高项的次数与乘积中因式的个数相同; 展开式中最高项的系数是1,后一项的系数是因式中各个常数项b1,b2,b,…的代数和;再后一项是因式中各个常数项b1,bg,b3,…中每次取2个作出的所有不同的积的代数和;最后一项是各个因式中各个常数项的积。根据上面这两个特点,可以归纳出一个结论:(a+b1)(a+b2)(a+bg)…(a+b)=a"+(b1+b2+b3+…+b)a"-1+(b1bg+b1b3+…+b1bn +b2b3+…+bbn+…+b-1bn)a4-a +b1bsb3…bu. (1) 进一步,应用数学归纳法,证明这个结论对任意大于1的自然数n都成立. 1°当m=2时,从上面的计算可知式(1)是成立的;2°假设当%=飞时式(1)成立,即 (a+b1)(a+b2)(a+b3)…(a+bk-1)(a+be)=a品+(b1+bg+b3十…+bk-1+bx)a-1+(b1bg+b1b3+…+b1bk +bgbg十…+bbe+…+bk-1bx)ak-3 十… +b1b2b3…bk-1bk. (2) 为了书写简单起见,设 b1+bg+bg十…+bx-1+b%=$1, b1bg十b1b3+…+b:bk十bgb3+…+bzb+…+bk-1b%=, "64· ==========第70页========== b1b:b3…be-1bk=Sk.式(2)可以写成: (a+b1)(a+bg)(a+b3)…(a+bk-1)(a+bk)=a十81a-1十8eak-8+…+8k. (3) 当%=k+1时,可以推得: (a+61)(a+b2)(a+83)..(a+bx-1)(a+6x)(a+6x+1)=(a十8a*-1+seak8+…+8x)(a+bk+1)=ak1+(s1+0K+1)a+(8+6x+181)a-1+…+bk+1Se. (4) 但是 s1+bk+1=b1+bg+b3+十bk十bk+1, 它就是b1,b2,b,…,bk+1这形十1个常数的代数和. +8x+11=(0103+0183++0108 +bεb3十…+b2be十…+bk-1b)+bk+1(b1+bg+…+bw)=8108+6108++010x+010x+1+beb3+…+bb%+bgbk+1 十… +016x+1, 上 它就是b,b2,b,…,bk+1这尼十1个常数中每两个乘积的代数和. ◆◆●· 最后可推得 bk+1S6=b1bbg…bebk+, 它就是b1,b2,b3,…,bk+1这k十1个常数的积.因此,式(4)可以写成: 65◆ ==========第71页========== (a+B1)(a+62)(a+68)...(a+6x)(a+6p+1)=aド+1+(b1+b+b3+…+b+1) ak +(b1bg+b1bg十…+b1bk+1 +bb3+…+bgbe+1十…+bxbx+1)ak-1 十… +b1b2b3…be+1. 这也就表明,当=+1时,式(1)也是成立的 这样,根据1°和2°,就可以断言:式(1)对于任意大于1的自然数m都是成立的. 例1.计算(+1)(+2)(+3)(x+4)(十5).[解]所求的积是x的5次式,其中:心的系数=1, x4的系数=1+2+3+4+5-15,心8的系数=1.2+13+14+1.5 +23+24+2.5 +3.4+3.5 +4.5 =2+9+24+50=85, x的系数=12.3+1.24+125+1.34+1.3.5+1.4.5 +2.34+2.3.5+2.45 +34.5 =6+8+10+36+45+120=225, w的系数=1.234+1235+12.45+13.45 +234.5 =24+30+40+180=274, 常数项=1.2345=120,所以,所求的积是68, ==========第72页========== 5+15c4+85w3+225x2+274+120 习题3·2(1) 应用上面所得出的公式,直接写出下列各题的结果: 1.(x+2)(x+3)(x+4). 2.(x-1)(x-2)(x-3). 3.(c+1)(x+2)(c-3)(-4). 4.(-2)(x+3)(x+1)(-1). 有了计算二项式乘积的公式,就容易推出二项式a+b的n次幂的公式.为此,只需把(1)式中的b1,b2,b3,…,bn都改成b.这时,(1)式的左边就可以写成(a+b)"的形式,而右边各项的系数就有以下的规律: a"的系数仍旧是1; a-1的系数b1十b2+…十b是%个b的和,所以Q-1的系数是nb(也就是Cb); an-2的系数b1b2+b1b3+…十b-1bn的各项都变成b2,它的项数等于个元素中每次取2个元素的所有组合的种数 C,所以a”-3的系数是Cb2, 同理,a-8的系数是C3;a4的系数是Cb4;;a-k的系数是Cb;;最后一项b1bg…bn变成了b. 这样,原来的等式就变成 (a+b)"-am+Cnam-1b+Cia"-262 +…+C%an-k.bk十…+b 这个公式通常称为二项式定理.右边的式子称为(十b)的二项展开式 为了写法上的一致,用C9和Cm来分别表示@和b”的系数1,这时上面的公式就可以写成 ==========第73页========== (a+b)"=C9a"+Can-1b+Ca"-2b2+… +C%a”-kb+…十Cb” 这里的C%,C,O%,…,C%,…,C%称为二项展开式的系数, 或称为二项系数. 例2.写出(1十心)15的展开式的前4项. ャ [解](1十x)15=1+C1+Cs2+C3+… =1+15x+105x2+455a3+…, 所以,展开式的前4项是1+15ax+105w2+455x3. 的8.写出(+、)》 的展开式。 I解1(@+》 -(》 -是2a+10 -是[(2)+032a)+2✉)r +C(2)8+0영(2x)2+0(2c)+C] -04e+6-32x+15,16o +20.8x3+154x2+6.2c+1)=(64m8+192m5+240a4+160r3 +60x2+12w+1) =643+192+240x+160+6++ c3. 说明1。在熟练以后;中间步骤可以省去一些. 2.对于这类问题,当幂指数较小时,可以应用杨辉三角形直接把各项系数写出. 。6自· ==========第74页========== 半 习题32(2) 1.应用二项式定理,展开: 四(答+》: (2)(a+/b)9; (3)(+知5; (4)(c-3)6. [提示:x3=x+(一3).] 2.应用二项式定理,写出: (1)(1+2c)1”的展开式的前三项; (2)(5+8) 3\6的展开式的前四项。 3,化简: (1)(2x-1)5+(2ac+1)5; (2(+2)-(-)。 §3·3二项展开式的通项公式 上节导出了二项式定理: (a+b)m=Ooam+Cia-1b+02am-26 +…+Cha"-畅6+…十Cb, 可以看出,二项展开式具有以下特点: 它是按照字母a的降幂排列的、关于字母a、b的%次完全齐次式 展开式里一共有%+1项.展开式里各项的系数顺次是 08,CA,O%,…,0%,…,C%; 其中C是第+1项(k=0,1,2,…,肌)的系数. 展开式的第k+1项是 ·6日 ==========第75页========== T1+1-Ck an-kble 这个公式称为二项展升式的通项公式.利用这公式可以直接写出展开式中某一指定的项 例1.求(心+a)12的展开式中的第4项和倒数第4项.[解】这里,通项公式是 Tk+1=C1c12-ka*. 于是 T4=T3+1=C72c12-3,a3 12.11.109.a8 1.2.3 =220a3x8, 又,展开式中共有12+1=13项,其倒数第4项也就是第10项,由此得 T10=T9+1=C112-8.a9=C3aa9=220ax3.注可以看到,在(x+)12的展开式中,第4项与倒数第4项的系数是相同的.想-想:在这个展开式里,第1、2、3、5、6项与倒数第1、2、3、5、6项的系数是不是也顺次分别相同?为什么? 例2.求(答+是)的展开式里中间一项的系数 【解】这里儿=8,展开式共有9项.所以中间的一项是第5项.今 2-.-c()(2) 8.7.6524 1.2.3434 70.1624=1120 81 81 由此可知,中间一项的系数是18 81 8.7.6.524 注意按照题意,求展开式的中间一项2:3:·多的系数, ==========第76页========== 年 所以,这一项里所有关于心的数字系数部分都要计算进去,不要误解成 只求二项系数C. 习题33(1) 1.(1)求(+√b)12的展开式里的第4项和第9项; (2)求(2xc+3y)0的展开式里的第3项和倒数第3项. 2.(1)求(x+a)2的展开式里的中间的一项; ②号+}的展开式里的中间两项. 3.求(x+“)9的展开式里第2项与倒数第2项;第3项与倒数第3项;第4项与倒数第4项的数字系数. 比较一下所求得的数字系数是否两两相等: 应用二项展开式的通项公式,还可以解一些比较复杂的问题,下面举例来说明、 的展开式里心3的系数.这个展 开式里有没有不含心的项?如果有,把这一项求出.分析这里并没有直接告诉我们所求的是哪一项,我们把k作为未知数,根据题意,并应用通项公式 16()3》, 导出一个关于飞的方程.这样,解这个方程,求出k(k=0,1,2,…,11)代入上式即得. 【解]这里,二项展开式的第飞+1项是 -0院(”气3》' 3k33-5 =C-FR谷6 ·714 ==========第77页========== 首先,令35形=3,得3-5h=18,即得=3.所以, 6 所求含有3的项是第4项,这一项的系数是 ..89 11.10,927 1.2,365536 445565536 ・ 其次,令23-5肠 6 =0,得33-5=0,这个方程没行整数 解.所以,展开式里没有不含心的项。 例4。在(@+号)”的展开式里,已知其第4项与第6项 的系数相等,求展开式里不含a的项 分析这里,先要根据题中第一个条件确定的值,然后就可仿照上题确定展开式里不含a的项究竟是哪一项,从而求出结果. [解]这里红-0堡a(合》°=C8a2m-8, To-Cira2n-(°=C8a24-10. 根据题设条件:T4与T6的系数相同,得 C经n=C8. 但 C经n=C3州-8, 故得 C2n=C经H-3. .∴.5=2m-3, %=4. 由此知,所给的式子就是(a+是)°.它的第k+1项是 +1=0a-(&)》” =0ga3-% 。72· ==========第78页========== 平 欲求不含a的项,只须令8一2k=0,即得飞=4,代入上式,得 Ts-C.a=8765=70. 123◆4 所以,展开式里不含a的项是T0. 习题33(2) ” 1.在(c+a)1的展开式里,求: (1)含有a8的项; (2)含有x3的项, 3.在(+)》的展开式里,有没有 ; (1)不含盆的项? (2)含有x8的项? 如果有,把这个项求出来. 3.求(3x+° 18的展开式里的常数项. 4.已知(a+)”的展开式里的第3项含有a,求n. §34二项展开式中系数间的关系仔细观察一下,第60页的杨辉三角形中的这些二项系数,可以发现这些系数间存在着一些特殊的关系,如: (1)与两端等距离的一对系数分别相等; (②)当二项式的幂指数是偶数时,处在中间位置的一个系数最大;是奇数时,处在中间位置的两个相同的系数最大.现在来证明,不论%是什么自然数,(α十b)"的二项展开式的系数间都有这样的关系. (1)在二项展开式里,和两端等距离的两项的系数相等.[证]二项展开式里,各项的系数顺次是 C8,C%,C%,…,C%,…,Ca-2,C州-1,C. ·73· ==========第79页========== 这里,Cg与C,C与C%-1,C%与Cg-2,…分别是与两端等距离的两项的系数 .·C9=1,C%=1,∴.C%=C%. 根据组合的性质 Onm 可以得出 0h=C州-1,C4=0%-2,…. 这样,命题就得到了证明. (②)当m是偶数时,二项展开式中第罗+1项的系数有最大值;当n是奇数时,二项展开式中第士项的系数和第 n+1 2十1项的系数有相同的最大值.[证]二项展开式中各项的系数顺次是 C%=1,C%=%, C=n(1),Cg=n%-1)(-2) 1.2 12.3 C4=%(n-1)(n-2)(n-3) 12.3.4 这里,从前一项的系数到后一项的系数,分子是乘以逐渐减小1的数(如%,n一1,n一2,…),分母是乘以逐渐增大1的数(如1,2,3,…).因此,一个二项展开式各项的系数,开始时总是逐渐增大(当分子上的乘数大于分母上的乘数时),但到了一定项(当分子上的乘数小于分母上的乘数时),就要逐渐减小.因为和两端等距离的两项系数分别相等,所以系数最大的项必定在中间.由此可知: 当%为偶数时,展开式共有+1项(%十1为奇数),中间 74 ==========第80页========== ·· 的一项是第罗+1项,这一项具有最大的系数。 当%为奇数时,展开式共有%+1项(%+1为偶数),中间 项有两个,就是第士项与第n十1+1项,2 这两项具有相 同的最大的系数. 上面的证明中,指出二项展开式中相邻两项的系数间有着一个特定的关系: Ch+1=%(m-1)(m-2)…(m-k+1(m-) 12.3…k(k+1) 限一在C, k+1 这里,一恰巧是第十1项中a的幂指数.由此可知,二项展开式的系数还具有下面这一性质: (3)展开式里,从第2项起的系数,等于它的前一项的系数乘以这一项中a的幂指数、再除以前一项的项数 根据上面这些性质,在展开(a+b)”时,就可以不必去逐 个计算二项系数C%,C,C,…的值,而直接把这些系数求 出. 例知,要展开(a十b)11,根据(3),只需先把它的前6项写出,得 (a+b)11=:a1+11a1b+55a62+165a8b3 1×1)(11×9)(65×号)+330a7b4+462a65+….(165×)(330×号) 由此,再根据(1),就可把后面的6项写出,得 (a+b)11=a11+11a10b+55g62+165a8b3 +330a7b4+462ab5+462a5b6+330a4b7+165a3b8+55a269+11ab10+b11, ·75· ==========第81页========== 例1.展开(2x-°. 分析把2看成a,-号看成b.就可以应用(a+b)°的二项展开式来展开 [解1(2z-日)° -[2)+(-是g] (2x)+-8(2)'(-공) +282)(-是)°+56(2m(-是)°+702(-克+56(2)(-号)°+22)(-豆)+8(2a(-是)》+(-공 =256x8-512x7y+448axy2-224xgr +70y-14y+7时 ッ+2563. 注意这里,最后求到的展开式里,与两端等距离的两项的两个系数并不相等,中间一项的系数也不是最大系数,请读者想一想,这是为什么? 例2.应用二项式定理,展开(1+心十心)4. 分析把1十x看成a,x2看成b,就可以应用(a+b)的二项展开式展开. ·7B4 ==========第82页========== 年 [解](1+x+x2)4=[(1+)24 米 =(1+)4+4(1+c)3.2+6(1十c)2(a2)3+4(1+c)・(a3)3+(x3)4=1+4c+6ar2+4a3+a4 +4w2+12a3+124+4c +64+12ax5+6ax8 +4c6+4x7+c8 -1+4c+10x2+16c3+19ax+16ax5+10w8+4x7+w8 习题34 1,求二项展开式里系数最大的项(系数只需用组合数符号表示, 。 不必算出): (1)(a+b)10, (2)(1+x)17. 2,应用二项展开式系数间的关系,展开 の(+学) (2)(+ 3.应用二项式定理,展开 (1)(x2+x+2)5; (2)(x2+2x+1)5. [提示:(2)c2+2ax+1=(+1)2.] 4.(a+b)m的展开式里如果第5项的系数与第13项的系数相等,求展开式里系数最大的项(系数用组合符号表示,不必算出)。[提示:先确定”的值.] §35(a-)的展开式 在§3.2已经看到,凡是可以化成(a+b)"形式的式子,都可以应用二项式定理来展开.为了应用上的方便,现在来导出(a-b)"的展开式.显然,所求的展开式,只需在(a+b)"的展开式里,用一五来代替b就可以了. 77 ==========第83页========== 因为 (a-b)=[a+(-b)]" 乳 =C0a"+Cia"-1(-B)+Ca2(--6)2+...+Chan-(一b)6+… +Cn-2a2(-b)n-2+Cg-1a(-b)n-1+C%(-b)n -Coan-Cian-16-40Ran-262+... 十(-1)kCw"-6+…+(-1)"Cmb”, 这样,就得到了(a一飞)"的二项展开式 (a-b)"-=C0an-Cia"-b+C%a1-2b2+… +(-1)C所a"-kb+…+(-1)"Cgb". 它的通项公式是 Tk+1=(-1)C%a"-b花 很明显的,对于(一b)"的二项展开式,除去各项是正负相间以外,其他都和(a+b)”的二项展开式相同. 例1.求(a+√b)5+(a-√b)"的展开式[解](a+Ni)5+(a-√b)5 =[Cga+Cga4(√b)+C%3(√b)?+Cga2(√b)3+C结a(/万)4+C(√b)][OSa-Oia(B)+Ca8(6)2 -Cga2(√五)8+Ca(√b)4-Cg(Nb)]=209a+2号a3(√万)2+2Ca(/i)4=2a+20a3b+10ab2. 例2。求(号2)展开式里系数敬大的 分析(α一b)”的展开式里,因各项的系数除掉符号外都与(a十n的展开式里相应的项的系数完全相同,所以仍可应用§3·4里的(②),先求出系数绝对值最大的项,再选择其中系数是正数的一个。 ·78· ==========第84页========== [解]展开式里,系数绝对值最大的项有两个,就是T? 与Is,其中T7的系数是正数,所以所求的是展开式里的第7项.即 a-(-1)%-・()(2)-1718e. 习题35 1.展开: (1)(x-2)8; @(-)》. 2.写出: (1)(√3-/2)6展开式里的第5项; (-”展开式里系数绝对值最大的项, 3.求(1++2)5.(1一)5展开式里的中间的项.[提示:(1+就+x2)(1-x)=1-x3. §36二项展开式里各项系数的和 在第60页的杨辉三角形里,把每一横行里的各个系数相加,可以发现二项展开式各项系数的和,有一个重要的特点:(a+b)1 1+1=2=22, (a+)3 1+2+1=4=22, (a+b)8 1+3+3+1=8=28, (a+b)4 1+4+6+4+1=16=24, 由此,可以归纳出 (+b)"展开式里各项系数的和等于2".对于这个命题,只需在公式 ◆79● ==========第85页========== ◆ (a+6)"=C8a"+Chan-1b+C2a-2b2+… +C所a”-b+…+Cbn 里,令a=b=1,即得证明. 在二项展开式各项系数的和的公式 C州+C0+C%+…+C%+…+C=2m 里,两边都减去C,并用1来代替C9,即得 C%+C+…+C%+…+C州=2"-1. 这个公式指出:在%个相异元素里,每次取1个、2个、…飞个、·4…、%个元素的所有组合种数的和,等于2的见次幂减去1,因此它通常称为组合总数公式. 例1.求证(a+)”的展开式中,各奇数项系数的和与各偶数项系数的和相等,并且这个和就是2-1, 分析这就是要证明C%+C%+C4+…=C%+C+C%+….为此,只 需证明它们的差等于零: C9-C%+C%-C%+C4-C9+…=0. 这里,左边的式子恰巧是(α一b)”展开式里各项系数的和.所以,在(a-b)r的展开式里,令a=b=1即可证得. [证]在公式 (g-b)"=C0a"-C%a-1b+Ca"-2i2 -Ca"-3b3+C4a-4b4-C7an-5b5+… 里,令a=b=1,得 C9-CA+0g-0%+04-0%+…二0. (1) 把所有含“一”号的各项移到等式右边,即得 C9+C%+CA+…=C%+C%+C%+…, (2) 这就证得了展开式中各奇数项的和等于各偶数项的和.又, C9+C%+C%+C路+CH+C%+CR+…=2. (3) (1)+(3),得 。80◆ ==========第86页========== 是 2(0+2+염+…·)=2, .C%+C份+C+…=罗=2-12 (4) 从(2)和(4),得 C9+C8+C4+…=CH+C%+C%+…=2m-1. 命题得证. 应用(a+b)"展开式里各项系数的和的公式以及例1的结论,还可以证明一些关于组合数的恒等式,举例如下: 例2.求证当%是偶数时,总有 1+20%+C%+20%+CH+2Cm+…+20%-1+0n=32n-1. [证】左边=(O9+CA+C%+…十C) +(C+C%+C%+…+C%-1). 已知 C9+O%+C%+…+Cg=2n,(组合总数公式) C%+C+C克+…+Cg-1=2-1,(例1) ..左边=2m+2m-1 =(2+1)2m-1 =3.2-1 .左边=右边. 习题36 1.某单位有7篇革命故事,现在要从中取出1篇或几篇编入黑板报里,共有几种不同的取法? 2.有1分币、2分币、5分币各一枚,1角票、2角票、5角票、1元票、2元票、5元票各一张,可以组成多少种不同的币值? 3.求证:当n为奇数时,总有 2+C%+2C%+Ca+…+Cm-1+2Cm≈32m-1。 ·81· ==========第87页========== §3·7二项式定理在近似计算中的应用 应用二项式定理,可以比较简捷地计算某些数的正整数次幂的近似值,使它达到一定的精确度 例1.计算(1.009)5的近似值(精确到0.001).[解](1.009)5=(1+0.009)5 =1+5×0.009+10×(0.009)2+10×(0.009)8+… =1+0.045+0.00081 +0.00000729+· 这里可以看出,如把第二项以后的各项一律略去,而前二项之和作为(1.009)的近似值,那末它与真值间的误差不会超过 0.001.因此 (1.009)5≈1+0.045=1.045. 例2.计算(0.991)5的近似值(精确到0.001).[解】(0.991)5=(1-0.009)5 =1-5×0.009+10×0.000081 -10×0.000000729+…=1-0.045+0.00081 -0.00000729+… 这里可以看出,如果略去从第三项开始的以后各项,而把前二项的代数和作为(0.991)5的近似值,它的误差不会超过 0.001,因此 (0.991)5≈1-0.045=0.955. 从上面这两个例子可以看到:如果a的绝对值比起1来是很小、且不太大的时候,应用近似公式 ·82· ==========第88页========== 辛 (1+a)"≈1+na(a>0) 或 (1-a)"≈1-na(a>0) 可以计算出(1士a)”的近似值,使它具有一定的精确度. 但是,有时也会通到这样的问题:应用上面的公式求近似值时,它的误差超出了指定的范围,这时就需要在二项展开式里再继续取一项或儿项来计算. 例3.计算(1,009)0的近似值(精确到0.001).[解1(1.009)0=(1+0.009)0 =1+20×0.009+190×(0.009)3+1140×(0.009)3+… =1+0.18+0.01539 +0.00083106+…, 可以看出,要使所求的近似值的误差不超过0.001,需要取展开式中前三项的和,就是 (1.009)0≈1+0.18+0.01539≈1.195例4.计算(0.991)0的近似值(精确到0.001).[解](0.991)20=(1-0.009)0 =1-20×0.009+190×(0.009)3 -1140×(0.009)3+… =1-0.18+0.01539 -0.00083106+· 象例3一样,这里要使所求的近似值能精确到0.001,应该取展开式中前三项的代数和,就是 (0.991)0≈1-0.18+0.01639≈0.836注应用近似公式 (1土a)"≈1士na(a>0) ·88·, ==========第89页========== 计算到的近似值,它的误差接近于%(1),而用近似公式 1.2 (1±a)ア~1士na-+0(n1) 1·2 (a>0) 计算到的近似值,它的误差接近于nn-1)-2)a,当a很小且n不 1.2.3 过分大时,这两个误差都已很小,在实际应用上已足够保证应有的精确度了. 上面的例子,都是求底数接近于1的%次幂的近似值,现在再来举一个求其他一些数的%次幂的例子. 例5.计算(4.003)c的近似值(精确到0.01).[解](4.003)8=(4+0.003)6 =48+6×45×0.003 +15×44×(0.003)3+…=4096+18.432+0.03456+…≈4114.47, 注倘使4.003是一个近似数,按照近似计算法则,(4.003)°的近似值只能取4个有效数字,这时就有 (4.003)6=(4+0.003)° ≈46+6×45×0.003 ≈4096+18.4 ≈4114, 习题3.7 应用近似公式(1士a)”≈1士na(a>0),计算下列各题: 1.(1.02)6. 2.(0.998)8 3.(0.999)1o 4.(1.0006)5 5.(1.003)10, 6.(0.9997)1 应用二项式定理,计算下列各题的近似值,使其值的误差不超过指定的范围: ·84◆ ==========第90页========== 7.(3.002)6,误差不超过0.001. 年 8.(1.002)16,误差不超过0.001. 9.(5.001)5,误差不超过1. 10.(3.998),误差不超过0.1. 本章提要 1.二项式定理 a+6)*=00an+Cian-16+02an-262+... +Can-6+…+Cbn. 2.二项展开式的性质 (1)通项公式 Tk+1=C你an-b(=0,1,2,…,n); (②)系数最大的项 当物为偶数时:第号+1项, 当m为奇数时:第”项和第”+1项2 (3)各项系数的和 C9+C头+C%+…+C%+…+Cm=2m 3.(a-b)n的展开式 (a-6)n=0ga2-Cian-16+02an-282-... +(-1)*Can-6+…+(-1)nCbn A 通项公式: Tk+1=(-1)C%an-畅(k=0,1,2,…,n). 复习题三 1.展开: (1)(√+a); (-》 2.化简: (1)(2+3c4-(2i-3r; ◆85· -n ==========第91页========== (2)(1+Va)5+(1-√a)5. 3.应用二项式定理,展开: (1)(1+x-x2)5: (2)(1-2+x2)6. 4.(2+)”展开式里第4项的系数与第13项的系数相等,求民 开式里不含x的项 5.(x立-心)"展开式里第5项的系数与第3项的系数的比是 7:2,求展开式里含有心项的系数.“6.求: (1)(1-2c)(1+3ax)4展开式里的前三项; (2)(1+x+x2)(1-x)10展开式里含有x4的项; (3)(1+2c一3x2)6展开式里含有心的项的系数; (④)((。-1+是)厂展开式里不含的项 从 7,求证: (四(-)展开式里的常数项是(-2135(2m-1少;n! 文 (2)(1+020展开式里系数最大的项是135…(②m-12(2x)”. m! 8.求: (1)(1+如)(1+x2)(1+3)…(1+xn)展开式里各项系数的和; (2)(1+)(1+x2)(1+)8.(1+xn)”展开式里各项系数的和. 9.应用二项式定理,证明: (1)555+9能被8整除; (2)(n+1)n-1能被n2整除; (3)990-1能被1000整除; (4)8910+87能被88整除. [解法举例:(1)555+9=(56-1)6+9 =(565-C655654+C%55658-…+C8酷.56-1)+9 =56[5654-C55569+C85562-…+C88]+8, 这里最后一式的两个项都能被8整除,所以原式能被8整除.] 10.用二项式定理,求8910除以88所得的余数. 11.计算下列各式的值,精确到0.01: (1)1.04; (2)1.9985, ·86● ==========第92页========== 平 第四章复数 数的概念的扩展,是代数里重要内容之一,它和代数里的其他内容(如代数式、函数、方程等等)的研究有着密切的联系.到目前为止,我们所讲到的数还只限于实数;代数式、方程等等,还只限于在实数范围里进行研究.为了进一步研究这些内容,同时也为了今后能更有效地掌握数的知识、处理有关实际问题,本章将把数的概念作再一次的扩展,引进关于复数的知识。 §41数的概念的扩展 在小学算术里学过自然数、零、正分数的基础上,“代数”第一册和第二册已经把数的概念作了两次扩展.到目前为止,读者已学过的数的系统,可以归结成下表: 正整数(自然数) 「正有理数正分数 有理数零 实数 负整数 负有理数负分数 无理数正无理数 负无理数 我们知道,在实数范围里,加法、减法、乘法、除法(除数不能是零)和乘方这五种运算总是可以进行的,但是乘方的逆运算—一开方一却还不是永远可以进行的。例如,在实数范 ·87· ==========第93页========== 围里,负数就不能开平方 回顾一下,过去对于数的概念的儿次扩展,都是从解决实际问题的需要提出的.这说明了,数的概念的每一次扩展,都是为了适应人类生产活动中实际的需要.但是,从数学运算的角度来看,这种儒要也正反映在要解决“逆运算可进行”这 一问题。例如,为了使加法的逆运算一减法一一永远可以进行,就需要引进负数;使乘法的逆运算一除法(除数不是零)一永远可以进行,就需要引进分数;使正数的乘方的逆运算一正数的开方—水远可以进行,就需要引进无理数。新数的引进,解决了逆运算可进行的问题,数的范围也随着扩大了;由此,也就可能更有效地解决更多的实际问题这里还需要注意,数的范围的每一次扩大,都是在原有的数的基础上增添了一种新的数而构成的.因此,在扩大了的数的范围里,原有的数所原来具有的运算意义和运算性质,仍旧要保持下来 为了要使乘方的逆运算永远可以进行,需要把实数的范围再加以扩大.下面各节将根据上面所说的原则来扩展数的概念. 习题41 1.方程4x=3在整数范围里有没有解?要使这个方程有解,需要引进怎样的数? 2.方程4x+3=0在正有理数范围里有没有解?要使这个方程有解,需要引进怎祥的数? 3.方程如2=2在有理数范围里有没有解?要使这个方程有解,需要引进怎样的数? 4.在实数范围里,方程+1=0是否有解? 5.在什么条件下,实数系数的一元二次方程a心+bx+c=0才有实数根? ·88◆ ==========第94页========== 半 §4·2复数的概念 我们早已知道,在实数范围里,负数不能开平方.最简单的一个例子就是一1的平方根没有意义, 为了使开方运算永远能够进行,首先就得引进一个新的数,用它来确定一1的平方根的意义. 本节就从解决这个问题着手来引进新的数. 1.虚数单位要确定一1的平方根的意义,也就是要引进一种新的数,使方程 G2=一1 (1) 有确定的解 我们用符号“⑦”来表示这样的新数,并且规定,数i具有下面的两条性质:1°2=-1; 2°¢与实数在一起,可以按照实数的运算法则进行运算.数⑦称为虚数单位. 根据上面的规定,可以知道: (-)2=2=-1, 所以一⑦也是方程x2+1=0的一个根.由此就解决原来提出的问题,可以说:在扩大了的数的范围里,方程 x2+1=0 有两个根i与一;或者说,一1有两个平方根与一元.根据上面的规定,容易推出虚数单位的一个重要性质: 计=; 2=1; 3=.=-:29=(2)2=(-1)2-1. 一般的,对于任意整数见,有 ·89· ==========第95页========== i4n=(⑦4)n=1"=1; +1=进排.店=: 4n+3=4n.=一1; 4n+3=24.3=一元, 这个性质通常称为⑦的周期性. 例1,计算: 元-1,克2,公-3,公4。 【解】1=1=分i 龙-9=11 23、 =ー1; 一1 8-3.1i =0: 411. 注解这个问题,也可应用上面的性质,把1变形成+8来计算,具体解法请读者自己完成。 例2.计算: 60.的.…28, 这里%是自然数. [解】ii、3.…8n=21+2+3++8n) 1+2+3+…+8m-6m(8+1)=4n(8m+1), 2 ,∴.原式=24(8m+2)=(⑦”)(8n+1)=18m+1=1, 习·题42(1) 1.计算2,49,9,含812、计算: (1)7+202+808-27; 90·・ ==========第96页========== 章 (2)还,+1.冰+2.冰+8(k是整数)} (3)i3.…22-1)(飞是整数). [提示:(3)分为奇数或偶数两种情况来讨论.]2。纯虚数我们来解下面的方程: x2+2=0. 这个方程也就是 x2=-2. 很明显,它在实数范围里没有解. 但是,根据上面所规定的虚数单位i的意义,容易算出: (√2i)2=(√2i)(√2i)=22=ー2,(-√2)2=(√2)2=-2. 这样,把数的范围扩大以后,这个方程就有两个根: 1=√2i,.c2=-√2元. 一般的,姐果a是一个正实数,那末√a也是一个正实数,且有 (土√a)2=0=a×(-1)=-a. 因此,把数的范围扩大以后,负数一的平方根就有两个值: √ai和-√ai. 形如b元(这里b是一个不等于零的实数)的数称为纯虚数 3。虚数我们再来解下面这个方程: 心2-2x十2=0, 这里,判别式 b3-4ac=4-8=-4<0. 这个方程在实数范围里没有解. 应用配方的方法,心2一2+2=(c一1)2+1,原方程可以变形成 ·91· ==========第97页========== (c-1)2+1=0, 就是 (-1)3=-1, 由此可得 花-1=士元. 根据的意义,两边都加上1,得 化=1士元. 这样,在扩大了的数的范围里,方程心2-2+2=0也有两个根: 花1=1+i,2=1一元 形如g+bi(这里a,b都是实数,且b≠0)的数称为虚数.4。复数在代数第二册里当引进了无理数以后,为了把这种新的数和原有的有理数用统一的名称来称呼,引进了名词实数”,并且说“有理数和无理数总称实数” 同样,在引进了虚数以后,也需要把这种新的数和原有的实数用统一的名称来称呼.我们引进以下的定义: 如果、b都表示实数,那末形如a+bi的数称为复数;a称为复数的实部,b称为复数的虚部,b称为虚部的系数 当b=0时,复数a+i就表示实数a,即 a+Oi--a. 特别,当a=b=0时,复数a+b元就表示数0,即 0+02=0. 当b≠0的时候,复数a+bi是一个虚数.这时,如果a=0,那末它就表示纯虚数b元,即 O+bi-bi. 根据上面这样的规定,可以把复数a+bt按照a,b是否为零分类如下:· ●92· ==========第98页========== 形 实数(a+0i=a) 人 复数a+bi 纯虚数(0+bi=bi,b卡0) 虚数(a+bi,b≠0) 非纯虚数(a+bi,a≠0,b≠0) 注意1.如果4+b2表示一个虚数,必须加上≠0这一条件. 2.实数有有理数、无理数或正数、负数的区分(见第87页的表),对于虚数来说,就没有这样的区分 3.为了方便,有时也用一个单独的字母,例如8,来表示一个复数这时,如果要进一步指出它的实部和虚部是什么,就说:“复数=a+b”. , 4.如果没有特别的说明,本章中的字母a,b都表示实数, 3计을151+- 【解]原式=2√/2i+√2i-3√2 =(2√2+√2-3√2) =0%=0. 例4.解方程x4-2-6=0 【解]4-x2-6=0. .(+2)(x2-3)=0. 由2-3=0,得G=士√3. 由x2+2=0,得心2=-2,w=士√2,由此即得,原方程有4个根:士√3,士√2 人 习题42(2) 1.~写出下列这些复数的实部、虚部和虚部系数: 护 (1)1+; (2)-V3i (3)-√2; (4)0, 2.已知复数的实部和虚部的系数,写出这个复数来: (1)实部是一√2,虚部的系数是1; (②实部是0,嘘部的系数是-Y; ·93· ==========第99页========== (3)实部是3,虚部的系数是0; (4)实部是1,虚部的系数是-1. 3.回答下面的问题: (1)3i是不是正数,-3i是不是负数? (2)√5i是不是无理数? (3)01是不是虚数? 4.计算: (1)2i+5i-9i;(2)· +~ 3.5 (3)W2i-/②7i+√3i; (4)/-8i+V8i-8i, 5.解方程: (1)x2+5=0; (2)4x2+9=0; (3)x4-16=0; (4)x4+x2-20=0, §43复数与平面内点之间的对应 在上一节,从负数开平方的需要出发,逐步引进了复数的概念.但是,这样引进的新数,究竟有什么具体意义呢? 为了解决这个问题,需要象过去用数轴上的点与实数间的关系来说明实数的儿何意义一样,利用平面内的点与复数间的关系来说明复数的几何意义, 1.复数平面在代数第二册里,我们已经学过:任何一个实数α,都可以用给定的数轴上唯一的一个点来表示;反过来,给定的数轴上任何一个点,都唯一地表示一个实数.这就是说,数轴上的点和实数间可以建立一一对应的关系 在代数第三册学习函数和它的图象时,我们还曾学过,用两条互相垂直的数轴心和y(它们的交点是O)构成一个直角坐标系;那末,一对排定了顺序的实数(a,),就可以用这个直角坐标系里唯一的点M(@,b)来表示;反过来,在这个直角 。94。 ==========第100页========== 坐标系里的每一个点M(a,b),也都唯一地表示着一对排定了顺序的实数(α,b).这就是说,平面直角坐标系里的点,和 一对排定了顺序的实数之间,可以建立一一对应的关系 从复数a+bi的定义可以看出,复数也是由一对排定了顺序的实数a和b构成的,这里实部a和虚部系数b就分别相当于点M(a,b)的横坐标和纵坐标.所以,在引进了复数以后,就可以用一个复数名=g十bi来表示平面直 ・:a+ 角坐标系里的点M(a,b);反过来,对于任何一个复数名=a+b成,也就可以用平面直角坐标系里以a做横坐标,b做纵坐标的点M(a,b)来表示它 图41 (图41).这也就是说,平面直角坐标系里的点,和复数之间,可以建立起一一对应的关系 很明显,对于用这种方法来表示复数:=a+bi,当b=0时,对应的点都在轴上;当=0时,对应的点都在则轴上.这也就是说,表示实数的点都在¢轴上,表示纯虚数的点都在y轴上.· 这种用来表示复数的平面,称为复数平面.其中心轴称 人 为实轴,y轴称为虚轴 2。复数的相等与不等在学习实数时,我们知道,利用数轴上的点的位置关系,可以规定它们所对应的实数间的相等和不等的关系.就是: 设实数a和a在数轴上所对应的点是A和A',那末, 当点A和点A'重合时,a='; 2 当点A和点A'不重合时,a≠a',这时,点A在点A'的右边时,4>a; 。95● ==========第101页========== 点A在点A'的左边时,a0, 或<0, 这样,在不等式两边同乘以,根据不等式的性质应有 2>0, 或 2>0, 就是 -1>0,或-1>0, 这就引起了矛盾.这就表明,既不能>0,又不能<0,即:与0这两个数之间不能规定大小关系. 例求适合于方程 (3c+2则)+(3ac-y)i=13-2i 的实数心和y的值 分析根据复数相等的意义,使等号两边两个复数的实部与虚部系数分别相等,就可以得到一个关于x,y的方程组,解这个方程组即得这个问题的解 [解】已知化和则是实数,所以3a+2则和3c一y都是实数,那末, (3+2y)+(3x-y)元=13-2元,3x+2y=13, ● 3x-y=-2, 解这个方程组,得 「花=1, ly=5. a=a' 注意应用a+bi=a+'i的关系导出 时,必须注意,a, b,b都应该全是实数 3。共轭复数考察下面这两个复数: 1=4+3i,z2=4-3i. 这是实部相等、而虚部的系数互为相反的数的两个复数.把这两个复数在平面内表示出来,可以看到,它们所对应的点对 ·97 ==========第103页========== 称于心轴(图43). 我们把关于c轴为对称的两个 1-4+36 点所对应的两个复数称为共轭复 3 数.这就是说: -1384 如果两个复数的实部相等、虚部系数是相反的数,那末这两个复 3 2=4-3 数为共轭复数 图43 如用字母:来表示复数a+bi, 那末它的共轭复数a一bi就可以用符号z来表示,就是 如果之=a十b%,那末=a-bi 因为0的相反数仍旧是0,所以当虚部系数是0时,这个复数的共轭复数就是它本身,这也就是说:实数与它本身共轭 注当b≠0时,复数?和它的共轭复数都是虚数,所以称它们为共轭虚数. 习题43 1.在复平面内,作出表示下列各个复数的点: (1)1; (2)-1; (3)0; (4); (5)-i; (6)1+; D (7)-1-; (8)1-; (9)-1+2. 2.写出图中各点所表示的复数(方格的每边等于单位长), 3.指出上题中各对互为共轭的复数 4.求适合于下列方程的实数:和y的值: (第2题) ·98· ==========第104页========== 多 (1)(3x-4)4(4y+5)i=0;[提示:0=0+0.] (2)(3x-4y)+(4x-3y)i=7i; (3)(x+y)+xyi=一5-24i; (4)(x2-2y2)+2ryi=6i-8. 5.实数m取哪些值时,复数(m2-3m-4)+(m2-5m-6)i是 (1)实数; (2)纯虚数; (3)零. §44复数与平面内向量之间的对应 1。向量在图41里,如把原点0与点M连结起来, 并且把原点O看做是这条线段的起点,点M看做这条线段的 终点(图·4),就得到一条有方向 之 的线段,这样的一条线段,称为一个 向量@,记做OM.线段0M的长 M:a+bi 度(就是向量OM的长度)?指出 b 了向量O亚的值的大小,∠心OM的 值日指出了向量OM的方向. 当点M与点O重合时,线段 图44 OM退缩成为一点,称这样的向量OM为零向量.零向星没 有确定的方向. 根据这样的规定,平面内的点,和向量之间,就可以建立 起一一对应的关系.即把点M对应于向量O立, 在§43已经讲过,平面内的点与复数之间,可以建立起 一一对应的关系。因此,通过平面内的点作为媒介,可以建立 ①方向相同的等长线设,不管它们的起点在哪里,都认为是相等的向量这里所指的则是限定以原点为起点的向量,明确些说,应该称为位置向避。本书中所讲的向量都是位置向量,所以全简称为向量。 ==========第105页========== 起复数与平面内问量间的一一对应关系: 复数8=a+bi对应点M(a,b)对应向量0应. w 例1.作出与下列各复数对应的向量: y (1)1=-2+; (2)2=-√3元. M1:-2+8 [解](1)作出点M1(-2, 1),联OM1,即得与复数:对应 -2 0 的向量OM1. (2)作出点M2(0,-√3), 联OM2,即得与复数对应的向 M2:-W3 =2 量OM2. 图45 2。复数的模数如果向量 O立与复数2=a+bi相对应(图46),向量OM的长,就称为复数:的模数.容易看出,这里 r=va2+b2 M:a+bi 如果b=0,那末艺=a+b%是 一个实数,这时它的模数等于√a+0,也就是{a|.因此,复数的模数也称为复数的绝对值,并且 图46 也用在这个复数的两旁加上两条竖线来表示.就是 |g=a+bi=r=√a2+b. 例如,例1里的复数和22的模数分别是 1=-2+=√(-2)+1=√/5,2=|-√3i!=W02+(-/3)=√/3,例2.求证两个共轭复数的绝对值相等 [证]·设名=a十b,那末它的共轭复数就是z=a-bi◆100● 鑫 ==========第106页========== 于是 2=a十bl=/a2+ =laーbi=√a2+(-b)=√a2+63 이=| *例3.求满足下列条件的复数名所对应的点的轨迹: (1)z=4; (2)x<4. 分析·复数:的绝对值,从几何意义来说,就是这个复数所对应的 点M与原点O间的距离.因此,本题就是求与原点的距离等于4和小 于4的点的轨迹。 9 【解1设复数:所对应 的点是M. =4 (1)=4的儿何意义 2F [< 是:点M与原点O的距离永 远等于4个单位长度.由此43-2-1.0128 -1 可知,点M的轨迹是以原点 2 为中心、4个单位长度为半径的圆 (2)|2<4的儿何意义 图47 是:·点M与原点O的距离永远小于4个单位长度.由此可 知,.点M的轨迹是以原点为中心、4个单位长度为半径的圆 的内部。 习题44(1) 1.作出下列各复数所对应的向量: (1)3+4; (2)3-48; (3)-3+4; (4)"一3-4 (5)5i; (6)-5; (7)5; (8)-5. 0101-● ==========第107页========== 2.证明:上题中的8个复数所对应的点在同一圆上,[提示:证明这些点和原点的距离都相等,] 头 3.已知复数 1=-√3+i, 2=2, zg=√2-√2i, 34=-26, (1)求作这些复数所对应的向量; (2)求这些复数的绝对值; (3)求这些复数的共轭复数,并作出它们所对应的向量; (4)这4个复数以及它们的共轭复数所对应的点,是不是在同一圆上?为什么? *4.求满足条件1≤|z1≤2的复数名所对应的点的轨迹. 3。复数的辐角在图48里,表示向量OM的方向的 这个角6,称为这个向量所对应的复数:的辐角.这就是说:c轴的正方向与向量OM所夹的角,称为向量OM所对应的复数2的辐角 很明显,不等于零的复数:的辐角有无数多个值,这些值相差2m的整数倍.把其中适合于 0≤8<2m 的辐角8的值称做辐角的主值 知道了一个复数的辐角的主值,那末,在这个主值上加上2π(飞是任意整数),就得到了这个复数的辐角的一般值.例 如,复数6的辐角的主值是受,它的一般值是受+2m,这里 飞是任意的整数. 利用三角学中已经学过的知识,可以知道:要确定一个复数名=a+b(2中0)的辐角8,只需利用公式: ,8 Co8gng 其中r=√/2+b, 0102· ==========第108页========== 必须注意,当z=十b8=0时,它所对应的向量O亚没有确定的方向,所以0这个数没有确定的辐角.例4.求下列复数的辐角的主值: (1)1+; (2)-1+2; (3)-1-i; (4)1- ()2; (6)√2; (7)-√2; (8)-√/2元 [解]容易看出,这8个复数的模数都等于√2。设这8个复数的辐角的主值分别是01,02,9,…,88。 1 0891=/21 1 sin1=-2 可以看出,8:的终边在第I象限,所以0<8<受.再从这两 方程之一即可求得 4· 同理,可以求得 元,8g5 Cos bo-2=1, 0 sin 65=- 可以看出,8的终边在c轴的正方向O上,由此得。一0,同理可得 e=匹,61=元, 2. ·108◆ ==========第109页========== 说明这里如果只写出一个方程 c0991=-1 那末因为余波函数的值是的在0到2=间的角奶有两个,即置和血,所以还不能确定日的值.但是,先从这两个方程确定日1的终边 位置以后,那末就只需应用其中一个方程求出91,这时所得的值也必适合另一个方程, 从这例子不难看出,根据a、b的符号就可确定复数=a+b2的辐角的主值所在的范围,如下表所示: a 6 辐角终边所在象限 辐角的主值范围 0<0<音 II <0、b>0,求 (1)a; ●(2)-a; (3)bi (4)-bi 的辐角的主值. 2.求下列各复数的辐角的主值: 四号+V: (2)1-1√3i; 2-2 (3)-是+V3 ④-是V3. 3.求下列各复数以及它们的共轭复数的辐角的主值(查表): (1)-4-V3i; (2)5+12i, 4。复数的三角函数式从公式 cos6=a sin6b 容易看出,复数名=a+bi的实部a和虚部系数b,可以分别用关于模数?和辐角8的表达式 a=r cos0,b=r sin 来表示.所以,复数z=a+b%也就可以应用关于?和0的表达式 z=a+bi=r cos 0+ir sin0 =r(cos 0+i sin 0) 来表示 式子r(cosB+isin)称为复数名的三角函数式;与之区别,式子a+bi称为复数?的代数式. 很明显,知道了一个复数的代数式,只需应用第102页里 ·105• …2之 ==========第111页========== 的公式来确定它的模数和辐角(通常只需写出辐角的主值),就可以把它化成三角函数式. 例5.把下列复数改用三角函数式表示: (1)-√3+: (2)-1. [獬](1)这里?=√(-√3)2+12=2. て CosB=-V√ 2 in0=5, 0的主值是名 4-V8+i-2(osgx+8n吾s入 (2)这里=1,0的主值是匹. 22=-1=cos +i sin 例6.求复数4(c0s驾-is如号)的模数与辐角.分析这里4(osg-ii血智)还不是r(cos9+isin9)的形式,所以不能直接看出它的辐角是什么.为此,先把它化成三角函数式 【解】.· sm(-爱)--sing, .4(os晋-isin容)=4[cos(-)+ism(-g)小. 由此可知,这个复数的模数是4,辐角是2m一g(这里居是整数). 注,这里因为并不要求求出辐角的主值,所以上面用负角的三角 ·106· ==========第112页========== 函数来解就比较简便。倘使要求求福角的主值,那末只需在-答中令-1,即得主值是号. 一般的,对于形如(cosB一isin6)的复数,都可以用这种方法化成复数的三角函数式r[cos(一)+iin(-)门. 习题4·4(3) 1.化下列各复数为三角函数式,并且作出和它们对应的向量: (1)4+3; (2)-5-12i; (3)3V3-3i; (4)-5+5; (5)-5; (6)13; (7)6i; (8)-4i, 2.化下列各复数为代数式,并且作出和它们对应的向置: (4(aos晋+iia) (2)4(cos o+isin ) 3 (④)4os共s+in若). 3.已知a=a+bi=r(cos6+iin),用复数的三角函数式表示它的共轭复数它. *4.化下列各复数为三角函数式,然后求出它们的模数和辐角主值:(2(cosg-iing)5 2)-3(os空+iin¥} 5 (3)√3(os15°+igin165)方(4-cos号m+iain §45复数的加法和减法 我们已经知道,一个复数可以用复数的代数式a+bi或者复数的三角函数式r(cosB+ginB)来表示;并且知道,复数z就对应着平面内的一个点M(,b),或者一个向量 OM,由此使复数得到了几何解释。 ·107· ==========第113页========== 在这基础上,现在就可以来建立复数的各种运算的法则,并且对复数的各种运算作出几何的解释.本节先研究复数的加法和减法.· 1。复数加法和减法的法则复数的加法和减法,可以按照多项式的加法和减法的法则来进行.就是:实部和实部相加减,虚部和虚部相加减.用式子表示即是: (a1+b1)士(42+b)=(a1士a2)+(b1士b)i.例1.计算: (1)(1-)+(2-)+(3-)+(4-); (2)(シ-8+2√2i)~(√2i-2). [解1(1)原式=(1-)+(2+)+(3-)+(4+) =(1+2+3+4)+(-1+1-1+1)元=10, (2) 原式=(-2+2√2)+(2-√/2)=(-2+2)十(2N√2-2)i=/2i 注意复数的和,或者差,仍旧是复数.在特殊情况下,这种复数可能就是实数或者纯虚数 例2.计算1与2的和,已知 4=2(co罗+iim号),( 중+sn). 하】-(号+)-1+/1 2=4(+是)-2N8+2城, .+2=(1+2√3)+(2+√3)元. ·108◆ ==========第114页========== 说明复数用三角函数式表示时,欲求它们的和或差,一般要先把它们改用代数式表示,这样可使计算比较简单.题中如无其他要求,求到的和或差,可以不必再化成复数的三角函数式. 例3.设(x+2y%)+(y-3ac)-(6-5)=0,求实数x、y的值. 分析先把左边化简成a+bi的形式.从a+bi=0得a=0,b=0即可列出关于x,y的方程组,从而求出x,y的实数值.[解1.·(c+2yi)+(y-3ac)-(6-5)=0, .∴.(G+y-5)+(2则-3+5)i=0.既然心,y均为实数,所以,根据复数相等的规定,有 x+y-5=0, 2y-3c+5=0, 解这个方程组,得 龙=3,y=2. 习题45(1) 1.计算 与 (层++1-号)-(侵+ (2)(-√2+√3)+(V3-V2) -[(V3-W2)+(V3+V√2)]; (3)[(a+b)+(a-b)]-[(a-b)-(a+b)](a,b均为实数); (4)(2x+3yi)-(3ac-2yi)+(y-2ri)-3ai(c,y均为实数); (5)(1-37)+(2+49)-(3-58). 2.计算: (1)(cos210°+igin210)+(cos120°+iin120); (2)3(cos60°+isin60)-2(cos180°+iin180) +(+) ·108· ==========第115页========== 3,求实数心和y的值: (1)(2+5m)-3-(14i+3x-5y)=0; (2)(x+)+(y-y)-(-3+2)=0; ((+)+(-+)+(-)-0 4.求证:两个共轭虚数的和是实数,差是纯虚数*5.设名是复数,解下列关于?的方程: (1)22+2-6i=6-3i; (2)z一8=1+21, [提示:设2=x十yi,代入方程,从而解出x和y.] *2。复数加、减法的几何解释利用复数与向量间的对应关系,对复数的加法和减法的意义,不难作出几何的解释. 想必读者已经知道,物理学里象力、速度这些量,都是向量(也称做矢量);对于两个向量,可以应用平行四边形法则进行加法运算.例如,图 48中1和F2表示作用于同一 点O的两个力;用表示这两个力的 有向线段0☑1,0M作为邻边,画 出平行四边形,那末通过这个作用 F 点O的对角线O亚就表示这两个 力的合力F,这就是说: 图48 0立=0M+0M2 现在,我们来证明:如果向量O丛1与OM所对应的复数分别是: 1=a1+b1i,22=a十b28, 那末这两个向量的和OM所对应的复数z,正就是上面所规定的复数1与的和(a1+b1)+(a+b2). [证】OM1MM?是平行四边形 E (图49),点M1的坐标是(a1,b1),点 M2的坐标是(a2,b2). 作x轴的垂线M1N1,M2N2和 4M1 g MN,并且作M1S⊥MN.容易证明, 这时有 Na △MSM≌△ON,M2, 四边形MNWS是矩形。 图49 ●110· ==========第116页========== 一 于是, ON=ON1+N.N=ON1+M1S=ON1+ON2=a1+a2, NM=NS+SM=NM1+N2M2=01+62 由此得M的坐标是(a1+a2,b1+b2).从而可知,向量OM对应复数 (a1-a2)+(b1-bg)i. 这样,复数的加法就得到了几何解释, 之 如果先作出与对应的向量OM1, 与2对应的向量OM2,然后,以OM1为 M1(a1,b1 对角线,以OM?为一边,作平行四边形 0M2M1M(图410),容易证明0☑就 对应着1与的差 M2(an a) (a1-a2)+(b1-b2)i, 0 这样,复数的减法也就得到了几何的解释。 图410 理解了复数加减法的几何解释,就可以应用复数这一工具来解一些实际问题. 例4.两个力都等于6公斤,作用于一点,并成120°角.求它们的 M 合力. 分析根据力的合成法则,这个问题可以用图解法解出,解法如下: 7 作☐OM1MM2,使 120° 0M1=0M2=6厘米, 0 .6公斤 ∠M10M2=120°, 图411 量出OM的长度,得6厘米.量出 ∠MOM的大小,得60°.由此可知,所求的合力是6公斤,它与这两个 力都成60°角. 从上面的解法中可以看出,如果把作用点O作为坐标系的原点, OM1所在直线作为x轴,那末这两个力就可以分别用复数 f:=6(cos0°+igin0),f2:2=6(c0s120°+2sin120) 来表示.由此可知,只要求出1与2的和名,那末,g的模数就表示合 0111· 、 ==========第117页========== 力f的大小;名的辐角就表示力f1与合力f间的夹角,从而确定了合力∫的方向.这样,就得到下面的复数解法. 【解]设力f1与f2分别对应着复数 1=6(c0s0°+2sin0)=6, 22=6(cos120°+iin120°)=-3+3V/3i, 那末它们的合力所对应的复数就是 2=为+2=3+3V3i, 可以求得 12|=V9+27=6; g9=3Y3-V5,.0=60. 3 由此可知,合力f是6公斤,且与力f1成60°角. 注为了叙述上的简使,“力f1对应着复数1=6(cos0°+isin0)”也可以直接用 f1=6(cos0°+iin0) 来表示。 例5.有作用于一点的两个力f1和f2,已知它们的大小分别是北 14.14公斤和20.00公斤,方向分别是北45°东、南60°东,求它们的合力. 【解】取作用点为原点,从西 450 东向东方向作为x轴的正方向(图 0 60° 412).则 f1=14.14(cog45°+isin45) 南 10(+) 图412 =10+10i, f2=20.00[cos(-30)+iin(-30)]=10V√3-10i. .f=f1+f2=10(1+√3)-0i. .f=10(1+√3)≈27.32, 0=0°, 由此可知,合力的大小为27.32公斤,它的方向为正东. 0112" ==========第118页========== *习题4·5(2) 1.用几何方法作下列加减法: (1)(4+5)+(2+3); (2)(4+5)-(2+3); (3)(cos210°+iin210°)+2(cos120°+isin120°); (4)2(cos120°+isin120)-3(cos135°+sin135), 2.设有平行四边形ABCD,它的三个顶点A、B、D可以分别用复 数0+0i、2+01、1+i来表示,试考虑下述问题:(①)画出这个平行四边形; (②)用复数来表示它的另一顶点C; (3)求对角线AC的长; (4)求对角线BD的长. [提示:把Bi平行移动到A应的位置,则B方=A应=A-店] 3.在第I象限里有一个锐角是30°的直角三角形,它的直角顶点 和30°角的顶点可以分别用复数 1=V3+0i,物=2√③+i 来表示.求表示第三个顶点的复数 4.有两力1=30.00公斤、f2=40.00公斤作用于一点,它们的夹角是60°,求合力的大小. §4·6复数的乘法 1。复数乘法的法则象复数的加减法一样,复数的乘法也可以按照多项式乘法的法则来进行,在所得的结果中把换成一1,并且把实数和纯虚数分别合并.用式子来表示,就是 (@1+bii)(a2+b2i)-(a:@:-b10s)+(a1ba+a201)i.例1.计算(1+)(4-)(2+3). [解1(1+)(4-)(2+3)=(1+)(4+)(2+3) =(3+5)(2+3)=9+19%. ·113· ==========第119页========== 来 例2.验证 心4+4=(就-1-)(0-1+)(筋+1-)(+1+). I [证](x-1-)(x-1+)(c+1-)(x+1+) =[(-1)3-打[(x+1)2-]=(x2-2c+2)(c2+2心+2)=c4+4, 《 原式得证 例3.设心、y是实数,解方程 (1+2)(c+yi)+(2g-2ac)=-5+38.[解]原方程可化为 (x-2则)+(2x+y)i+(2-2x)=-5+3i, 即 x+y%=-5+36, 因为心、y是实数,按复数相等的规定,有 心=-5,·y=3. 习题46(1) 1.计算: (1)(-5+6i)(-3); (2)(-3-4)(2+3); (3)(0.1+0.3)(-0.1-0.4); ④(+-日). 2.已知a>b>0,计算: (1)(a+Vi)(a-V石i)(-a+V)(-a-√石); (2)Vb-a(√a-b-V√b-a).[提示:√b-a=Va-bi.] 3.(1)证明(a+bi)(a-bi)=a2+b2; (②)应用上面这个公式,分解下列各式为一次因式的积:(i)x2+4, (iⅱ)x4-a4(a为实数), (iii)c2+2x+2, (iv)x+x+1, 0114◆ ==========第120页========== 手 [解法举例:(ii)x2+2x+2=(x2+2心+1)+1=(+1)2-2 =(x+1+)(x+1-).] 4.已知心、y均为实数,解下列各方程: (1)(x+y)i-2+4=(c-y)(1+); (2)(c+1)+(y-3)i=(1+)(5+3). 2。复数三角函数式的乘法运算我们来计算复数 a=r1(co901+元sin01),zg=ra(cos0g+元sin0g) 的积. 根据上面所述的复数乘法法则,容易算出:212=rir2(cos 01+isin 0)(cos 02+isin 03)=riT2(cos 01 cos 0a-sin 01 sin 03) +(cos 01 sin 02+sin 01cos 02)i]. 根据三角学中的知识,知道: Cos 01 cos 02-sin 01 sin 02=c0s(01+02),cos 01 sin 02+sin 01 cos 02=sin (01+02), 代入上式,即得 122=r1r2[cos(01+02)+8sin(01+0g)]. 这就是说,两复数相乘时,积的模数等于因数模数的积,而积的辐角等于因数的辐角的和. 从这个法则可以看出,在复数相乘时把它们表示成三角函数式后进行计算就比较简便例4.已知 %=r1(c0801+元gin01),22=r2(cos 0s+isin 03),2g=r3(cos 0:++isin 03), 求名12228. [解】.·名2=r1rg[c0g(01+0a)+iin(01+8g)门, ·115● ==========第121页========== 来 ,.12g=r1r2[c0s(01十02)+元sin(01+0g)] .rs(cos 03+isin03) =r1r23[cos(01+02+0a)中isin(01+0g+0g)],由例4容易看出,对于n个复数 =r1(C0s01+元sin01),z8=r2(cos 02+isin 02), 名=rn(c0s0a+元sin日n) 的积,可以用下式直接写出: 212g…2m=r1rg…rn[C0s(01+0g+…+0m) +isin(01+02+…+0n)门. 例5.计算 3(cog18°+sin18°).2(c0g54°+元sin54) ,5(co8108°+isin108°). [解] 3(cos18°+isin18°).2(c0g54°+元sin54°)5(cos108°+元sin108) =30[cos(18°+54°+108°)+2sin(18°+54°+108)]=30(cos180°+isin180)=-30. 例6.化简(cos38-sin30)(cos28-元sin28).分析这里的两个因式,都还不是复数的三角函数式,所以要先把它们化成复数的三角函数式,才能应用上面的法则. [解] (cos 30-isin 30)(cos 20-i sin 20) =[cos(-39)+isin(-38)][cos(-20)+isin(-28)]=cos(-30-29)+元sin(-38-28)=cos(-58)+元sin(-58). ●116● ==========第122页========== 华 习~题46(2) 1.计算: 少Va(eos宽+isin受)V3(osg+isin) ②)(eoeg+iin)-2(cosg-isin晋) (8)(1-(-+)o(-)-ism(높- (4)2(cos12°+isin12)3(cos78°+1sin78) 1 ·(cos45°+isin45). 2.证明: (1)3(cos75°-isin75)(cos15°-isim15)=-3i; (2)(cos 30+i sin 30)(cos 20-isin 20) =(cos 50+i sin 50)(cos 40-isin 40). *3。复数乘法的几何解释根据复数三角函数式的乘法法则,不难作出复数乘法的几何解释. 设复数1=r1(cos01十isin01),2=r(cos02+iin92)分别与向量0碰1,0应2对应.它们的积 z=12=r1r2[co3(01+92)+iin(01+e)] 与向量0M对应(图413). M 01+2 Me 81 图4.13 ·117、 ==========第123页========== 半 从图中可以看出,复数1乘以复数2的几何意义就是:把1所对 应的向量OM1先按照2的辐角日2旋转一个角2,再把OM1的长度 按照2的模数2改变r?倍,所得到的一个新的向量O亚,即是复数21·22所对应的向量. 例7.用几何的方法做下面的乘法: 2(cos60°+isin60)3(cos45°+isin45), y 6 5 4 3 M2 2 459 60450 3-2-10 ☒414 并用复数的代数式表示求得的结果 [解]作出复数2(cos60°+isin60)与复数3(cos45°+isin45) 的向量01与0M2,将向量0M旋转45°的角并且伸长3倍,得到新 的向量0M,它所对应的复数-1.6+5,8即为所求的积. *习题46(3) 下列各题中,作出复数1与2以及它们的积名所对应的向量: 1.1=2(cos30°+isin30),2=3(c0s60°+iin60°). 2.1=22(cos15°+iin15),22=V2(cos 1200-i sin 120). 3.=1-, 22=2(cos 750+isin 75). 4.1=2+1, 22=4+32, ◆118 ==========第124页========== R §4·7复数的除法 在习题46(1)的第3题,曾经证明过: (a+b)(a-b)=a2+b2 这就是说,两个共轭复数的积是一个实数.根据这一性质,就可以应用复数乘法的法测来做复数的除法 设1=a1十b1i,2=ag+bi,并设2≠0,那末 2丝=1+b18=(a1+b1)(a2-bsi) 222+b2 (a2+b=)(ae-b2i) agaztbbza361-a363i 略+b略 a略+略 这就是说:两个复数相除(除数不为零),可以先把它们写成分式的形式,然后将分子分母同乘以分母的共轭复数,并且把结果化简 例1.计算8+3-5)-55+3元 5+76・ [解 原式-18-34+30+38-52-(5+3)67) (4-3)(4+3) (6+7(6-7) -76+50i+8i+6-286-8702 25 74 =3+2%+-32+6-4+5% s4+10% 如果复数用三角函数式来表示,那未计算它们的商就非常方便. 设1=r1(cos01+sin61), z=r3(C0s02十sin6),+0. ·1190 n的耐 ==========第125页========== 那末 2ri(cos 0tisin 0) r2(cos0。+isin0) =r1(cos0i+元sin61)(cos0。-克sin0g) r2 (coS 02+isin 02)(cos 02-i sin 02) "(cos0;cos0s+sin 01 sin 0) cos2 02+sin2 02 +i(sin6cose,-os8hsin62] cos202+sin203 =m1[cos(01-0g)+元sin(61-0g)]. 这就是说:两个用三角函数式表示的复数相除,商的模数等于被除数的模数除以除数的模数,商的辐角等于被除数的辐角减去除数的辐角.例2.计算 12(o至+ism7)-6(eos2+sin入 [解1 12(cas7平+i6m7)÷6(cos+im)1,[s()+-sn(주-)c+ain ) 例3.求证 (1-√3)(c0s0+isin9)(1-i)(cos 0-isin 0) -√2[o(20-晋)+isim(26-)小. ·120· ==========第126页========== 子转 [证] 2(cos sin 左边= 5)(c0s9+8sin0八) √2(cas7至+isim)[os(-6)+isin(-9]2os(8+5)+i(g+5)l vcos(-+isn(g-0】 Vz[s(9+ir-+0) +sim(9+-至+9)] =√2[0s(26-品)+8in(26-)】-右边. 注从上面所述的复数三角函数式的除法运算法则 ri(cos 01+i gin 01) M 72(coS 02+i gin 02) 1 01 =r1[cos(01-f2)+isin(91-2)], 0 81-8g (2 读者可以利用图4·15,自己来解释复数除法的几何意义, 图415 习题4.7 1.做下列除法: (1) (2)3뉴 (3)1+2元 2-45 ·121。 ==========第127页========== 发 (④)√3(es150+isim150) V√3(cos225°+isin225) 《 -i (5)/2(c0s120°+isin120)· 2.计算: (1)√5+V3iW3+W/5;, √/5-√3i√3-W5i 9+3/2i (2)(3+W2)(1+i) 3.设之是复数,解方程: (4-多)=27-9 ②)g-1)=(1+ 4.设u,v是复数,解方程组 (1-i)u+(2+)v=7, (3+2)u-(2-3i)=13i. *5.(1)求证 (2)设x,y都是实数,求 x2-y2+2ay1 yW②+Vc4+i §48复数的乘方 把复数a+b%看成是一个二项式,那末就可以应用§32的二项式定理来计算复数的乘方.这里须注意,在所得的展开式里,把元的幂都化简,并且把实数和纯虚数分别合并. 例1.计算: (1)(2+3); 122· ==========第128页========== 丝 【解] (1)(2+3)4=24+4(23)(3) +6(22)(3)2+4(2)(3)3+(3)4=16+96元+21622+2163+8124=16+96%-216-216i+81 =-119-120%. 小、 ((+)-(동-+安[v8)+8(8)%+15(W3)g +20(/3)3+15(/3)24+6√32+2]=是[(27-135+45-1)+(4-60+6)Wg订4(-64)=-1. 例2.化简 (1+2)2_(2-)2 1-% 1+么… 【解]原式=一3+463-42 1-2 1+⑦ (-8+4(+) =(-8+4)1-动(1+2 =-3+4% 习题48(1) 1.计算: (1)(3-2); (2)(2+): (3)(-1-)-8 (4) 2+2i)¥ •123◆ ==========第129页========== 2。化简: (1)(1+2)5+(1-22)5; (2)(1+22)2-(1-)8 (3+2)3-(2+)3 3.计算: 1)(》; (2)(뉴) [提示:1+-+2-.] 1-元一1-2 4,设-14,--1百i求证 (1)w=w8=1; (2)w1=w2; (3)ω号=w1; (4)1+w1+ω1=1+w2+ω3=0. 注从(1)可以看出,w1和w2都是x3=1的根.本题的结论,在以后解题时很有用。 在§46里,曾谈到复数用三角函数式表示时做乘法就非常简便乘方是乘法的一种特殊运算,所以也就容易想到:把复数先用三角函数式表示,再来做复数的乘方,同样非常简便。 设名=?(c0s9+sin9),现在来计算2的%次幂 (r cos 0+isin 0)". 在§4·7例4的后面,曾经有过下面这个乘法公式:212n=r1(cos 01+isin 01) r2(cos 02+i sin 02)...mn (cos 0n+isin 0n)=m1rg…rn[c0s(01+02+…+0n) +isin(01+02+…+0n)]. 在这个公式里,令 Y1=T2=…=rn=”, 01=02=…=0n=9, 这时,就有 1=2=·=2=之。 ·124● ==========第130页========== 业 从而有 z"=[r(cos 0+i sin 0)]"=r(cos ne+isin ng)这里%是自然数.这就是说,复数的%次幂(%是正整数)的模数,等于这个复数的模数的”次幂,它的辐角等于这个复数的辐角的%倍 这一法则称为棣美弗定理。例3.计算(1+√3)0 【解】(1+√sw=2(o晋+i血爱)】门” 3 -io4(--) =-512一512√3元. 注本题如果应用二项式定理来计算,显然运算过程就要繁复得多 习题48(2) 1.用棣美弗定理计算: (1)[3(cos18°+isin18)]5; (2)[3(as空-in)]门; (3)(2+2)8; ω(-)。 2.计算: の(V-(-+)?(2σ- . ())2+(-2; 1- 1+2 ·125· ==========第131页========== (④(-1+V3015+(-1-V3)5 (1i)20 (1+)20 解法带例:(令-子+=0,则(-音+,)”=6”-o0au-1-V3.2 [习题48(1)第4题]. 原式=(W3i-1)-1-√3i2 (-1+√-1ーV3 =号1+3)-2.] 下面再举几个应用棣美弗定理来解题的例子, *例4.已知”是自然数,且(1+)”是实数.试问”的最小值是什么?这个实数是什么? 分析应用棣美弗定理,把(1+)”用复数的三角函数式表出,令虚部系数等于0,即可求出的最小值.[解】:1+i=V(cos要+iin ·.(1+)=(V2)(os+isin") 4 要使(1+)”为实数,必须并且只须in如-0. 2=4(k是任意整数). 要使n是最小的自然数,只须使k=1. .∴.n=4. 这时, (1+)”=(1+)=(W2)4(cosr+iinr)=-4 注如果要使(1+)”是正实数,应该怎样解?要使(1+)”是纯虚数呢? *例5,用sin8和cos0的代数式来表示sin56和cos58.分析根据棣美弗定理,有 (cos 0+isin )5=(cos 50+isin 50), •126· ==========第132页========== 为 美 之 应用二项式定理把左边的式子展开,并化简,这样就可以根据复数相等的意义,求得cos58和sin59的表达式.[解】设z=cos日+isin巳,则 =cos 50+isin 50=(cos0+isin 0)5 < =cos5 0+5 cos4 0.sin 0.+10 cos3 g sin29.2+10cos20,sin80.23+5cos9sin40,4+sin50.5 -(cos5 0-10 cos3 0 sin20+5 cos0 sin40)+(5 cos4 0 sin 0-10 cos20 sin3 0+-sin5 0)i. ★ 'cos 50=cos5 0-10 cos8 0 sin20+5 cos 0 sins0;sin 50=sin5 0-10 cos20 sin3 0+5 cos40 sin 0. 注应用公式sin2日=1-cos20、cos20=1-gin0,还可用cos6的有理代数式来表示cos58,用in9的有理代数式来表示in56.具体的推导留给读者. *习题48(3) 人 1.设n是正整数,求证: (1)(cos日+isinθ)-n=cos(-n8)+iin(-n8); (2)(cos -i sin 0)"=cas no-i sin no .V3 2.求最小的正整数”,使 是实数,并求这个实数 + 的值, 3.设(告》=1,求最小的正整数n. 提示: (1+)2 一二+可=名=i,所以本题不必用棣美弗 1-i 定理. 4.设1+)2”+1-)2”=2,求最小的正整数n. 1-元 1+ 木 5.用棣美弗定理和复数相等的条件,证明: (1)cos 20=cos2 0-sin20,sin 20=2 sin 0 cos0; (2)cos 30=cos3 0-3 cos 0 sin20,sin 30=3 cos2 gin 0-sin8 0. 上 ·127· ==========第133页========== X *6,运用棣美弗定理、复数相等的条件,以及二项式定理,试求ind和cos8用sin0,cos日表示的公式. §49.复数的开方 象实数开方的意义一样,如果复数w的%次幂等于另-- 个复数之,即0=名,那末U就称为的乳次方根.求复数2 的m次方根的运算,称为把复数z开n次方. 例如:因为=-1、(一)=一1,所以和-都是一1的二次方根; 又如:因为 1(1+y-1.(1y-, 所以1-1十W8元和一17。都是1的三次方根. 2 这里可以看到,在复数范围里,负数也可以开偶次方,正数的奇次方根也不只有唯一的值. 本节将根据开方是乘方的逆运算这一关系,利用棣美弗定理来导出复数开方的一般法则.由此说明:任何一个复数都可以开%次方,并且,任一非零复数的%次方根都有个值. 1。复数开方的法则先来计算一个具体的题目.求复数名=1十√3龙的平方根. 为了能应用棣美弗定理,先把这个复数写成三角函数式: 名-2(c0智+isin雪),并且设所求的平方根是 w=p(cos o+isin o), 那末,根据方根的意义:w=2,就有 0128● ==========第134页========== Lp(coisi)2coin, 就是 p(o0s2p+isin2p)=2(c0s管+isin)因为,两个复数相等时,它们的模数必须相等,而辐角可以相差2m的整数倍,所以 p2=2, 2p=3+2hr(飞是整数) 由此可知, 0=W√2 (因为p是模数,所以只取算术根), lo=6+,这样,即得所求的平方根是 w=√[o(晋1r)+isin(g+km)小 这里k是整数, 为了确定所求的平方根究竟有几个,不妨试取的一些值代入计算.例如: 当=0时,得 -√(e풍+sm)-号√동+号4 当=1时,得 二 √(금+)=-νб-6 正弦函数、余弦函数都是以2x为周期的函数,所以,徜使 K 取的其他值士2,士4,士6,…代入,求得的结果都与o相同;取的其他值土1,士3,士5,…代入,求得的结果都与w1相同.由此可知,1+√3元的平方根有两个值,并且只有 ·129· ==========第135页========== 两个值,它们可以用式子 V2[cos(答+m)+8sim(g+km)月(k=0,1)来表示 一般地,设复数=r(cos0+元sim9)(≠0)的次方根是w=p(c0sp+isin).那末,从w=2(≠0),可得 p"(cos no-i sin no)=r(cos 0+isin ) 由此,根据复数相等的意义,有 p"=r, p=0+2kπ(为整数), 从面有 p=Wr, 0+2k元 n ,∴.U%=rcog8+2kπ isin0+2k7 m 这里?表示的%次算术根,k只需取0,1,2,…,%-1这%个值.(想一想,这是为什么?) 为了方便起见,采用符号z来表示复数2的各个肌次方根,这样就有 z=yr[cos9+2h元+8sin8+2kr1 2 (k=0,1,2,…,-1). 这就是说,复数的%次方根有%个值,它们的模数都等于这个复数的%次算术根,而它们的辐角分别等于这个复数的辐角与2c的0,1,2,…,%-1倍的和的%分之,. 注意在实数范围内和在复数范围内,记号一的意义有所不同.例如:在实数范围里,√1只表示1的算术根1,在复数范围里√1 ◆130◆ ==========第136页========== 重 就表示1的两个平方根士1.又如:在实数范围里,-工只表示实数 -1,但在复数范围里,一工就表示-1的三个立方根一1,-ω,-w2,为了防止混淆,以后如要用方根符号来表示实数a的”个”次方根,宁可把a写成a+Oi的形式,表示成/a+0i,而仍旧把符号a(当它有意义时)来表示唯一的实数值. 例1.计算√-1十飞, 【解】-1+i-V区(s+im孕x.3 -√-1+i-√√z(s+in)(o(2+)+s(k+음)】[ (a+용1)-+sn(kr+)] 由此得 w=z(ew管g+8血骨以wz(cs号w+isi血若知)11 11 例2.在复数范围里,解方程5+1-0。[解]+1=0, 5=-1, .'.5=c0s元+i sin w. 于是 上 k=C082kr+元+isin2k死+元 5 5 =c0 공:+。+*4n(2+1) (k=0,1,2,3,4), 由此得 ·131· ==========第137页========== X @o cos 3 3 1=cos亏x+8sin亏, cg=c0sr+isin元=-1, 7 7 g=0os6五+isi万元, 9 9 4=008方π+isin6匹. 习题49(1) 1.计算: ()d(cas号r+iin号+(②)y8o0-n60. 2.计算: (1)-i; (2)√/2-√2i; (3)-1+√3i; (4)/-3+3i. 3.(1)求一1的三个立方根;(②)求-64的四个四次方根. 4.设a为正实数,应用复数开方的法则,解下列关于复数x的方程: (1)x3+a8=0; (2)x3-a3=0; (3)x4+w4=0; (4)x4-a4=0. 5.设a、b、c都是实数,且b2-4ac<0.解方程 ax2+bx+c=0(a+0). *2。复数开方的几何解释根据复数开方的法则,不难作出复数开方的几何解释 设z≈r(cos日+iin0)的n个n次方根是 w(oosi sin 其中k=0,1,2,…,n一1.并设w在平面内对应的点是Mk.那末,容易看出: ·132· ==========第138页========== 上 文 ()这些点与原点的距离都等于?.因此,这些点在以原点为中心、?为半径的圆周上; (2)任意相邻两点(例如Mo与M1,M1与M2,…)间的夹角都等于2.因此,只要先作出对应于的向量0证,把它沿逆时针方向 旋转角,就得对应于的向量O成,再旋转角2,就得对应于 2x u2的向量O成2,最后,把对应于0n-2的向量O立n-2旋转角m, 就得对应于Un-1的向量OMn-1.由此,从几何观点来看,复数名=r(cos日+isin)的n个%次方根所对应的点,恰巧是以原点为中心, M ?为半径的圆的一个内接正n多边形的顶点,其中第一个顶点(点 M)所对应的向量(向量OMo)与x M3 轴的正方向O间的夹角是】日(以 O元为始边). M M 例如,图4·16是当n=7时的情况. 图416 例3,根据复数开方的几何意义,试在复数平面内作出表示方程 一1=0的五个根的点. [解1'x=1=cos0+isin0,∴、x=/cos0+i8in0 M? 所以,它的五个根对应的点是一个以 坐标系原点O为中心、其外接圆半径 是1的正五边形的顶点Mo,M, 1 M2,M3,M4.先作出顶点 M。=cos0+iin0; 然后把向量OM。沿逆时针方向旋转 2M4 72°角,得到向量OM1,这就得到了 顶点M1;又把向量OM1旋转72°角, 图417 从而得到顶点M2;依次同样进行下去,即可得顶点山g,M4. ·133· ==========第139页========== 单 *习题49(2) 1,在复数平面内,作出表示:1)1的六次方根; (2)1的八次方根 的各点 2.求方程4+1=0的复数解,并证明这些解所对应的点构成一个正方形. 本章提要 1,虚数单位t 性质:(1)=-1; (②)。与实数在一起可以按照通常四则运算的法则进行四则运算.推论:i的整数次幂具有周期性: n+1=元,n+2=一1,4n+8=一i,4n+4=1. 2.复数 定义:形如a+bi的数(a、b都是实数).几何意义: (1)表示平面上的点M(a,b);(2)表示平面上的向量O应. 3.复数的代数式与三角函数式的互化 =a+bi=r(cos 0+i sin 0) r—模数(绝对值) r=lz={a+b2l=√a2+b 0一辐角 fcos0=a 确定辐角的公式: sin0=6 辐角的主值区间: 0≤0<2r 辐角的一般表示式: 0=2km+0(k是整数)。 4.复数的相等 ●134● ==========第140页========== 「a1=2, (1)a1+b1i=2+b2i←-→161=62. (②r1(c0s0a+iin0r1)=r(cos6+isin,)←→{91=2kx+.∫r1=Y2) 6.共轭复数a=a+bi和z=a-bi互为共轭复数. 6.复数的运算 (1)加法(a+)+(a2+b)=(a1+a2)+(b1+b2)i. (2)减法(a+b1i)-(a+bi)=(a1-ag)+(b1-b2)i, (3)乘法 (i)(a1+b1)(a2+b2i)=(aa2-b1bg)+(bg+ab2)i,(ii)r(cos01+isin 01).r2(cos02+isin02) =r1r2[cos(01+02)+isin(91+02)]. (4)除法 (i)a4+b1i=(+b1(ag-bgi】(a%+bgi≠0). ag+bzi a经+b (i)(cos+i sin=r1[cos(01-0)+isin(01-)] r(cos 02+i sin 02) (r2≠0). (5)乘方 (i)应用二项式定理.()应用棣美弗定理 [r(cos0+i sin 0)]"=rm(cos ne+isin n 0). (6)开方 r(cos日+isin6)=/7(cos2kr+8+i6in2%元+0 (k=0,1,2,…,n-1)。 复习题四 】,计算: (1)2次+4千论+5++6+次+7(是自然数); (2)1+含+2+28+…+255 2.设1=a1+bi、2=ag十bi是两个复数,在什么条件下有: ·135· ==========第141页========== f (1)1+2是实数?是纯虚数?(②)1一是实数?是纯虚数? 地 (3)1·2是实数?是纯虚数? (4)红是实数?是纯虚数?(这里2≠0.) 22 (5)号是实数?是纯虚数? 3,已知下列这些命题都是成立的,写出它们的逆命题.这些逆命题是否仍成立? (1)模数、辐角分别相等的两个复数一定相等; (2)共轭复数的模数相等 [提示:要否定一个命题成立,只需举一个例子来说明就可以了,例如,命题“两个偶数的和是偶数”的逆命题是“两个加数的和是偶数,则这两个加数都是偶数”,只要举出1+3=4一例,因4是偶数,但1和3都不是偶数,就足以说明这个逆命题不成立.] 4.(1)已知名=(cos0+isin),求证它的共轭复数?可以写成r(cos0-isin)的形武. (2)如果复数&的辐角的主值是a(0≤a<2x),怎样用a的代数式来表示复数8的辐角?表示8的辐角的主值? 5.求适合下列各式的实数和y: (1)(1+22)x+(3-102)y=5-62; (2)2+%+2-3i=y2+y8+9-2; (3)2x2-5x+2+(y2+y-2)i=0: 5 (④+2 6,化简: (1)(1-√3)[cos(p-39)+isin(p-39)门 X(cos29+iin29)9(cos29-isin2p): cos(e-o)+isin(e-p) (cos+isin) (2) (cs-in受) 0188· ==========第142页========== 生 7.计算: (1)(-1+√3)8-(-1-√3); (2)(-1+V②)4(-1-V②i)4 (V2+)4(V2-)4 8,计算: (1)(ェー1-√2i)(c-1+√2i)(x+1-√2i)(x+1+√2i); (2)(x-√2+i)(+√2+:)(x+√2-i)(x-√2-i). 9.已知m是一个不等于零的实数,且{m|<1.求证 V1+m+V1-m8-√/1-m+V1+m2 V1+m-v1-mi /1-m-V1+m含 的值是-个实数. 10.设w=-1+√3i 2一,求证: (1)(1-w+w2)(1+w-w2)=4; (2)(1-w)(1-w2)(1-w)(1-w8)=9; (3)a8+b3=(a+b)(aw+bw2)(aw2+bw); 长 (4)a8+88+c3-3abc=(a+b+c)(a+b+c2)(a+62+aw)[提示:可以利用ω的性质(见习题48(1)第4题): w3=1,1+ω+w2=0.] 11.利用√x2-y2士2cy=士(x+yi),计算: (1)√V-3+4i; (2)√7+V15i; (3)√5-√1ii; (4)√-7-√W15i. 2.设e)- ++1,求 (1)f(2+3), (2)f(2-3i) 的值。[提示:f(2)表示复数:的函数.在右边这个分式中,令8=2+3, 化简后即得f(2+3)的值.] *13.求作关于:的二次方程,使它的两根是: (1)1+和1Y正;②)和. 2 2 14.设1、和是不等于零的复数,用几何方法证明 ·187◆ ==========第143页========== }21}+}22}≥1+22}≥1-; 并回答下列问题: (1)这个二重不等式的两个等号能否同时成立? (2)1}+2≥1+22|中,什么情况下等号成立? (3)1+21一21中,什么情况下等号成立? *15.求证{1+2?+1-212=2z12+2到2,并作出几何解释、 16.已知甲数的平方等于乙数,且乙数的平方等于甲数,求这两个数. 17.设名的共轭虚数是?,解下列方程: (1)名+8引=2+; (2)22=8 [提示:.设=+(y+0),则名=一2i,] 61380 ==========第144页========== 生 第五章方程论初步 在本丛书“代数”第二册,曾在实数范围里探讨了一元一…次、二次方程,以及某些可化为一元一次、二次方程的高次方程的解法。本章将在学过复数的基础上,研究关于一元方程的一些初步理论和某些特殊形式的一元高次方程的解法 研究一元方程的理论和解法,需要用到一元多项式的一些重要性质,所以,本章将从复数范围内的一元多项式的一些重要性质谈起, §5·1多项式f(c)的一些重要性质 形如 aoc十a1c-1+a-8+…+a-1+am 的代数式,称为心的多项式.式中a0,a1,a2,…,w-1,am都是复数,%是自然数 如果ao≠0,这多项式便称为x的n次多项式.为了说 法上的方便,通常把不等于0的常数称为的O次多项式;把 常数0称为零多项式.零多项式没有次数.本章凡提到的%次多项式,都指只含有一个变量的正整数次的多项式 很明显,多项式 a0"+1"-1+an-2x8-8+…+am-1心十n 可以看作是心的一·个函数.为了方便起见,今后将用函数符号f(w)来表示x的多项式,并且用f(a)来表示=w时多项 ·139· ==========第145页========== 式f()的值. 下面将研究%次多项式f(心)的一些重要性质 1。多项式f(c)被x-a除,所得的余数我们先来看下面的问题: 问题已知f(x)=2x2-5w+1, (1)求f(2); (2)求f()除以心~2所得的余数 很明显,问题中的(①),只需把2代替f()中的心,进行计算,得 f(2)m222-5.2+1=-1, 问题中的(②),只需直接做除法,从 22-6x+1 心-2 2x2-4x 2-1 ー+1 t -花+2 -1 得余数是-1. 这里,可以看到,f(x)除以x一2,所得的余数恰巧是(②).现在来研究:一般的多项式f()除以心一a,所得的余数是不是总有这样的性质 我们知道,在除法里,被除式、除式、商式和余式之间,存在着这样的关系: 被除式=除式×商式十余式, 其中商式的次数应该等于被除式的次数减去除式的次数,余式的次数至少比除式的次数小1. 由此,可以得出:任何一个%次多项式(),总可以表示成以下的形式: ·1400 ==========第146页========== f()=(x-a)f1(x)+r, (1) 这里f1()是%一1次多项式,?是一个常数. 等式(1)是一个恒等式,不论x取什么值它总能成立.特别是,当=a时应该也成立,所以 f(a)=(a-a)f1(a)+r. (2) 对于等式(2)的右边第一项的值,不论f1(@)的值是什么,它总等于0,由此可知 f(a)=r. (3) 把(3)代入(1),就得到恒等式 f()=(w-a)f1()+f(). (4) 从(4),当心≠a时,可以得出: f() (5) 化-a f()+@ B-a… 这就告诉我们: 定理1(余数定理).%次多项式f(w)除以心一a,所得的余数等于f(). 应用余数定理,就可以不做除法而直接求出多项式∫()除以一a所得的余数.这样,有时可使计算比较方便。 例1.已知f()=8x3-9,求: (1)f(c)除以心- 六 骨的余藏(2寸a)除以+是的余数. [解]根据余数定理,f(c)除以一a的余数r等于f(a). 0这里a-司 .=时(侵)=8(侵)°-9-1-9=-8 0141◆ ==========第147页========== ②)这里a-司, ∴y=f(-2)=8(-号)-9=-1-9=-10.例2.飞是什么数的时候,多项式 f(x)=x3+ka2-20x-4 除以+2的余数是0. 【解]根据余数定理,f(心)除以心+2的余数是: f(-2)=(-2)3+k(-2)2-20(-2)-4 =4k+28. 今余数为0,所以 4k+28=0, .k=-7。 习题5·1(1) 1.不做除法,求下列各题中的余数: (1)(3x3-7x2+4x+40)÷(x-2); (2)((x4-23+x+15)÷(x-3); (3)(3,x8+5x2-5.x+1)÷(c+1)片 (④(-+2)÷(@+) 2.根据下列条件,求多项式(x)中字母系数飞的值: (1)f(x)=c3-5x2+13x+k除以x-2,所得的余数是3; (2)f(x)=2x3+3x2+x-1除以x+3,所得的余数是-4. 3.f(x)=6x3一192+ax+b除以x+1,所得的余数是一24;除以x一3,所得的余数是8.求a、b的值. [提示:从f(-1)=一24、f(3)=8写出两个关于a、b的方程组,再解这个方程组即得.] 2。多项式f(x)能被x-4所整除的条件余数定理 ·142· ==========第148页========== 的一个重要应用,是用来直接判断:多项式()能不能被x一a所整除 事实上,从恒等式 f(c)=(花-a)f1(x)+r 直接可以看出: 只须f(a)=0,f(ax)就能够被x-a所整除;要使f(c)能被一a整除,必须f(a)=0.把这两句话合起来说,也就是: 余数定理的推论要使多项式f(x)能被x一a整除,只须并且必须f(a)=0. 注对于某一结论B来说,如果具备了条件A后,这个结论就能成 立,这时说:条件A是结论B成立的充分条件;如果不具备条件A,这 个结论就不成立(或者说,要使这个结论成立,必须具有条件A),这时说: 条件A是结论B成立的必要条件.条件A既是结论B成立的充分条 件,又是结论B成立的必要条件,那末称条件A是结论B成立的充分 必要条件,简称充要条件.应用这种说法,上面的推论就可以说成: 多项式f(x)能被x一a整除的充要条件是f(a)=0, 大 因为,当f(a)=0时,f()=(c-a)Q(),这正表示着心一a是f(x)的一个因式,因此上面的推论通常也叙述成: 因式定理·多项式f(心)含有因式心一a的充要条件是 2 f(a)=0. 例3.已知多项式 f(x)=aox"+a1x"-1+…+am(ao≠0,%≥2)在c=a或者心=b(a≠b)时的值都是零,求证f(w)有因式 (c-a)(-b). 分析要证明f(x)含有因式(x一a)(x一b),只需证明f()能表示成(x一a)、(x一b)与另一个多项式的积的形式.根据因式定理,从f(α)=0可知,f(x)可以表示成(x-a)与多项式f1(x)的积的形式,即 ·143· ==========第149页========== (x)=(c-a)f1(x). 因此只要运用另一条件(b)=0,且a≠b,证明f1()可以表示成x一b与另一多项式f2(x)的积的形式 [证] ∴.'f(a)=0, .∴.f(x)=(-a)f1(), (1) 式中f1(x)是%一1次(n≥2)多项式. 用b代替(1)式中的,得 f(b)=(b-a)f1(b), (2) .·f()=0,而Q≠b,从式(2)可得 f1(6)=0. (3) 由此,根据因式定理, fi(a)=(a-b)f2(a). (4) 从式(1)和式(4),得 f(心)=(c-a)(-b)f2(). 这就证实了f(x)有因式(w-a)(x一b). 习题51(2) 1.证明: (1)c3+32-x-6能被c+2整除; (2)x一w能被x一a整除; (3)当n为奇数时,xn+a能被x+a整除; (4)当n为偶数时,xm-a”能被x+a整除. [解法举例:第(2)题设f(x)=x一am:今f(a)=an一an=0,所以x”-am能被x一a整除.] 2.证明(a-b)8+(b-c)3+(c-a)3能被a一b、b-c、c一a整除. 3.设f(x)=aon十a1xn-1+…+an(n≥3),且f(a4)=0,f(ag)=0,f(a)=0(这里x1,a2,a互不相等).求证f(x)含有因式 (x-1)(-a2)(x-3). [提示:运用例3已证得的结果.] ●144▣ ==========第150页========== 上 4,已知f(x)=ox”十a1n1十…十an,求证: 上 (1)在a0+a1+…+a%=0时,f()能被x一1整除; (2)在a-a1+…+(-1)an=0时,f(x)能被x+1整除.[提示:对于(②),把”分成偶数和奇数两种情况来证.] 3。多项式∫(c)的标准分解式如果当c=a时,f()的值是0,那末a就称为是多项式f(c)的根.例如,对于二次三项式 f(x)=32-2x-1 米说,因为f()-0,f(~司)=0,所以1和-昌都是它的根.由此可知,这个二次三项式可以表示成二个一次因式的积,就是 f(m)-3e-1(e-君). 高等代数里将证明在复数范围里,任何一个正整数次的多项式,至少有一个根。这定理称为代数基本定理.从这定理出发,可以推导出下面这个定理: 定理2.任何一个%次多项式 f()=aoxn+1c"-1+…+am-1c+am(ao≠0)都可以表示成n个一次因式的积分析这就是要证明:()能表示成 ao(x-a1)(x-a…(G-an) 的形式.为此,仿照例3的办法,首先证明f(x)能表示成(一)f1(x)的形式,f1(x)能表示成(x一a2)f(x)的形式,这样依次递推下去.[证】根据代数基本定理,多项式f()至少有一个根设这个根是1.那末,根据根的定义,必有f(a1)=0,由此,从因式定理可知,(心)可以表示成 f()=(cーa1)f1() (1) 0145.0 ==========第151页========== 小 的形式.这里f1()是心的%一1次多项式,并且含2-1项的系数是ao,即 人 f1(c)=ao心m-1+b1"-2+…+bm-1(ao≠0). 如n=1,f1(c)=aow°=ao,这时f(x)=ao(c一a1).定理得证 如果肌>1,根据代数基本定理,f1(x)至少有一个根,设这个根是a2,那末根据同样的道理,可以推出 f1(x)=(e-ag)f2(w). (2) 式中 f(x)=ao"-2+c1M-8十…十cn-9(a0≠0).由(1)和(2)可知 f()=(心-a)(x-ag)f(x). (3) 如果n=2,f(c)=ao,这时f(c)=o(-a1)(x-ag).定理得证. 如果%>2,仿照上面的方法再推下去,这样到了第%步,总可得到 (x)=ao(x-a)(x-ag)…(c-an)(ao≠0).(4)定理得证 在上面的证明中,所设的%个数1,ag,…,并不要求各不相同.如果这n个数中有飞1个都等于a1,2个都等于cg,…,;个都等于a4,这里1,2,…,都是自然数,且1十2十…十,=%,那末这个多项式就可以表示成f(c)=ao(优一1)(c-a2)…(-4)(ao≠0).(6)上面这个式子称为多项式()的标准分解式,其中x一,x一Cg,…,-,分别称为f()的k1重,k2重,…,飞;重因式. 例如,f(x)(x+2)(2+2c+1)(x3一3ax2+3心-1)的标0146• ==========第152页========== 准分解式是 f(c)=(c+2)(+1)2(-1)8, 这个多项式含有一重因式(简称因式)心+2,二重因式c+1和三重因式心一1. 从f()的标准分解式可以看出: 推论f(a心)有k重因式x一a的充分必要条件是:f(x)能被(心一a)整除,但不能被(w-a)+1整除 例4.已知f(ax)=4-3-花十1有二重因式x-1,把f(x)写成标准分解式. [解]f(x)能被(一1)=2-2c+1整除,做除法: 4-3 -然十11心2-2c+1 4-23+c2 c2+x+1 G3-心2-0 c3-2c2+w 心2一2G+1 c2-2w+1 0 得商是心2+心+1, ..f()=(x-1)2(c2+心+1) 、 =e-1(。--1+y8e--1)2 从f(c)的标推分解式,容易看出,当c=;时,f()=0,所以a;是多项式f(c)的根;当c=B≠y时, f(B)=ao(B-1)(B-ag).…(B-a)+0,所以B不能是多项式f(c)的根.这就是说: 多项式f()的每一个一次因式的根,都是f()的根.除此之外,多项式f()不能有其他的根. ·1474 ==========第153页========== 话 通常,把多项式f(c)的飞重因式一a的根,称为f()的飞重根,当=1时,这个根就称为单根.在计算多项式f(心)根的个数时,飞重根算作是飞个根.这样,从 f(x)=ao(c-c1)(c-ag)…(G-ay),(ao≠0)和1十k2十…十,=n以及上面的结论,就容易推出: 定理3(多项式的根的个数定理).次多项式∫()有,并且只有m个根, 例5.已知一1是 f()=c5+3ax4+4x3+4ax2+3a+1 的一个三重根,求这个多项式所有的根. 分析因为一1是多项式f(x)的三重根,所以f(x)应能被(然+1)8整除.直接做除法,可以求出(c)的另一个二次因式,从而找到其他两个根.由此得下面的解法 [解]x5+34+43+4ax2+3x+1x3+3c2+3c+1 5+34+3心3+x2 c2十1 x3+3x2+3w+1 c3+3x2+3+1 0 于是 f()=(c+1)8(a2+1) =(+1)3(+i)(x-). ..f(c)的五个根是1,1,1,,-元. 例6.求作一个三次多项式(),使它满足下面三个条件:(④它的三个根是1,是,-受 (②)各项的系数都是整数,其最大公约数是1; (3)最高次项的系数是正数. ·14B· ==========第154页========== 头 [解]满足条件(①)的多项式,可以一般地表示成:f)=o(-1)(r-)(e+安) a@-10(e2-)-a(2-是+) 要使这多项式各项的系数满足上.面的条件(2)和(3),a必须是4. 所以,所求的多项式是 f(e)=43-42-w+1, 注1.满足条件(②)的多项式,通常简称最简整系数多项式.2。本题中如无条件(③),那末有两解,另一个解是 fx)=-4c3+4x2+x-1. 习题51(3) 1.把下列各多项式分解成一次因式的积: (1)c-1; (2)3+1; (3)x4+4; (4)4+x2+1. 2.求作一个最高次项系数是正数的最简整系数四次多项式(),使它的四个根是: (1)1,-2,-3,4; 、 3。求作一个次数最低的多项式(x),使它的最高次项系数是1,并且具有根: (1)√2和√3; (2)1,-1和1+, 太 4。多项式恒等于零的条件在多项式 f(x)=ao心+a1n-1+…+n-1十am 里,如果o=1=…=n-1=am=0,f(c)就表示一个零多项式.这时,不论心取什么值,∫()恒等于零.这就是说:多项式f(x)恒等于零的充分条件是 ◆1490 ==========第155页========== 乐 a0=1=…=a1-1=a=0 可以证明,这个条件也是使多项式f(x)恒等于零的必要条件. [证】用反证法.如果ao≠0.这时,f(c)是n次多项式.根据定理3,∫()只能有%个根.这就是说,最多只能有n个数1,ae,…,a,能使f(心)=0.这与f()恒等于零这 一已知事实矛盾.所以,ao≠0是不可能的.因此=0.同样可以证明 a1=0,…ya-1=0,4=0. 由此可知,要使f(x)恒等于零,必须 a0=a1=…=am-1=an=0. 综上面所述,就可以得出:定理4.多项式 ao"+a1x-1+…+an-1+an 恒等于零的充要条件是 a0==…=0n-1=an=0. 由此,还可以得出:推论两个多项式 f(@)=aoq"+ajm-1+...+an-12+am,p(心)=bo+b1c-1+…+bn-1c+bn 恒等的充要条件是 ao=bo,a1=b1,…,aa-1=b4-t,an=bn. [证]f(c)与(x)恒等的充要条件是f(a心)一p(ax)恒等于零,就是 (ao-bo)c+(-b1)xn-1+…+(an-1-bn-1)x+(an-b)恒等于零.因此,根据定理4,必须并且只须 a0-b0=a1-b1=…=au-1-b-1=am-bm=0, ·150· ==========第156页========== 装 就是,ao=60,a=b1,,am-1=6n-1,an=bn. 例7.已知二次三项式心2+pc+g的两个根是5和一3,求p、g的值. 【解]根据题中条件,可得恒等式 、 x2+px+q=(x-5)(c+3). 就是 c2+pc+g=x2-2c一15. 比较两边的系数,得 p=-2,g=-15 注意想一想:除上述解法外,本题还可以有哪些另外的解法?例8.已知多项式f(心)=4c4-4w3+13c2-6w+9, (1)求证:f(心)能表示成龙的二次三项式的平方; (2)求这个二次三项式. 分析只要找到一个二次三项式a2+bx+c,它的平方恰巧等于(x),那末问题就完全解决了.所以,这就是要从 4x4-43+13x2-6+9=(ax2+bx+c)2 这一恒等式(假定能够成立)中确定、b、C的值.容易看出,a只能是±2.为了方便起见,不妨假设题中的二次三项式是士(2x2+bx+c).应用两个多项式恒等的定理,即可求得其解 [解]设∫(x)能表示成一个二次三项式的平方.那末4a4-4w8+13x2-6e+9=[±(2x2+bx+c)]9 (1) 是一个恒等式.从式(1)即有 44-48+13x2-6x+9 =44+4bx3+(b2+4c)x2+2bcc+c2, (2) 比较两边同次项的系数,得方程组 46=-4, (3) b2+4c=13, (4) (I) 2bc=-6, (6) c2=9, (6) 。151● ==========第157页========== rb=-1 解式(3)和式(④)组成的方程组,得 C=3 代入式()和式 (⑥)都适合,所以它是方程组(I)的解。 由此可知, 4w4-4w8+13aw2-6x+9=[±(2x2-c+3)]2 是恒等式.所以, (1)()能表示成心的二次三项式的平方;(②)这个二次三项式是 22-x+3或-2x2+-3, 说明1.本题解法中,先假设这个二次三项式是士(2x+b+c),然后再根据已知条件来确定b、¢的值,这样的解法称为待定系数法,b、C称为待定系数.在应用待定系数法解题时,常常要用到两个多项式恒等的条件 2.本题解的过程中,待定系数只有b和c两个,但是根据两个多项式恒等的条件列出的方程组却有4个方程.对于含有两个未知数的 四个方程所组成的方程组,从其中两个方程中求出b和C的值以后,还必须代入其他两个方程中进行检验,如果不适合,那末它不是原方程组 的解;如果原方程组无解,那末原先假设的恒等式就不能成立,例如, b=-1b=-1 本题中在解方程组(I)时,如从(3)和(6)来解便有。一3和c=-3'显然后一解是不适合其余两式的.又如,本题如改成 f(x)=4x4-4x8+13x2+6x+9, 那末,从 生x4-4c8+13x2+6x+9=[士(2x2+bc+c)]2 (I) 可得出方程组 4b=一4, (3) b2+4c=13, (4) (I) 2bc=6, (5) c2=9 (6) 由(3)和(4)可解出b=一1,c=3,但是,代入(5)不适合,方程组(I)无解.这就是说,不能找到b,c的值,使(1')为恒等式。由此可知(x) ·1520 ==========第158页========== 不能表示成二次三项式,原来的题目是无解的,即 4x4-4x3+13x2+6x+9 不可能表成x的二次三项式的平方. 习题51(4) ,1.已知 c3+px2+gc+r=(x-1)(x+2)(x-3) 是一个恒等式,求P,9,"的值. 2.已知3+8x2+5+a能被x2+3x+b整除,求a、b的值,并求除得的商 3.用待定系数法求下列各式的平方根: (1)9x4-12ax3-14x2+12x+9; (2)43x4-12c8+29x2-30x+25 4.设f(x)=6+6x3+3ax4-28x8-9x2+54x-27, (1)证明f()能变形成(x2+px+g)8,其中p,q都是实数; (2)分解f(x)的因式. §5·2综合除法 是 在研究方程的理论和解法时,常常要计算多项式(c)除以心一α所得的商和余数.用普通的除法,计算比较麻烦.如果利用§51里的两个多项式恒等的定理,将有一种比较简便的方法. 设f(c)=ao心"+a1心”-1十ag心2+…十-1G十aw(ao≠0)除以x-a,所得的商是p(c),余数是?.那末,从除法的意义,可以得出一个恒等式 f(x)=(心-a)p(c)+T. (1) 这里p(心)是x的%一1次多项式,把它记作 p(c)=boM-1+b1m-8+b2xn-3+…+bn-2c十bn-1. ◆153· ==========第159页========== 美 这样,从式(1)可得 ao"+a1c-1+ae-2+…十an-1十am =(c-a)(boa"-コ+b1"-2+ bg"-3+.….+bn-+bn-1)+r,就是 aoxn+12c8-j+a2cn-8+…+a-1十an =bo2+(b1-abo)m-1+(bg-ab1)x-3+…+(b-1-cb-2)c+(x-ab-). (2) 等式(2)是一个恒等式.根据§5·1定理4的推论可知,式(②)的等号两边同次项的系数应该相等.由此可得: a0=b0 .∴.b0=ao a1=b1-b0, ,b1=a1十abo; a2=b2-ah1, 62=a2+a61; an-1=6n-1-a6n-2, b:-1=au-1十abn-2; an=r--abn-1, .∴.=an+cbm-1. 从上面右边得出的一列等式可以看到,商p(心)的各项系数bo,b1,b2,…,bm-1和余数T,可以从被除式f(x)的各项系数ao,a1,a2,…,a-1,am和除式化-a中的a顺次递推出来,方法是: (1)把ao就作为b: (2)把bo与a的积abo加在a1上,即得b1; (3)把b1与a的积ab1m在ag上,即得b2 (2)最后,把bn-1与a的积ab-1加在a上,即得r.为了方便,上面的计算可以列式来进行:·· o a1 01-1 an +abo +xb1… +ab-2 +06-1 ao a+abo a2+abi an:-1+a6n-2au十cbu-1 01 D2 bn-13 •154” ==========第160页========== 韦 例如,求f(c)=3x5-143+7除以一2所得的商和余数,先把f(x)看成是按心降幂排列的完全多项式 f(ac)=3c5+0x4-143+0c2+0z+7, 由此确定各项的系数顺次是3,0,一14,0,0,7,再由G一2 确定=2,这样,就可以按照上面的算式,列式来计算: 30-1400·+72 +6·+12-4-8-16 3+6-2-4-8,一9 .'商是p(c)=3a4+6a3-2ax2-4c-8, 余数是个=一9 象上面这样求多项式f()除以c一的商和余数的方法,称为综合除法 例1.求f(c)=3a一4w2+5.w4除以+1的商和余数.[解)这里f(c)=5x+0x3-42+3x+0, 花+1=筋-(-1). 60-4+30-1 -5+5-1-2 5-6+1+2,-2 ,·.商是5x3-5c2+心+2,余数是-2. 注1.一个完全四次多项式应该有5个项,所以第一行排列出的被除式的系数应该有5个.在计算时,用这种方法来检验一下,可以防止把应写出的系数漏掉(例如,本题中最后常数项是0,稍一疏忽是很 容易被漏掉的) 、” 2.一个四次多项式除以一次式,所得的商应该是三次多项式.在计算时,注意到这一点,商就容易写出. 例2.已知f()=3x5-4-43+6x2+x-2, 求f() ·155· ==========第161页========== 业 分析在本题中,直接用号代督了(x)中的“来计算比较森须,根据余数定理,(-号)就是f()除以+号所得的余数.于是可改用综合除法来计算 [解]3-4-.4+6+1-2 23 、-2+4+0-4+2 3 -6+0+6-30 f-)=0. 习题52(1) 1.用综合除法求商和余数: (1)(3x4-5x3-4x2+3c-2)÷(x一2); (2)(25-12x4+143-2362+17x-33)÷(x-5); (3)(x3+6x2+11x+6)÷(x+3); (4)(x4-1)÷(x+1). 2.已知f(x)=1-3c+5x-2ac,应用综合除法求: (1)(5); (2)f(-4). 3.已知f(x)=3a4-2x3+5x2-8心+4,求: 四信) ②-) 对于多项式f()除以一般的一次二项式a心一b的除法,也可以应用综合除法来做.为了得出法则,先来解答下面的问题: 问题求f(x)=3x3-112+18w一3除以 (1)3x-2; 四女-号 的商和余数,并比较求得的两个商和两个余数之间各有怎样的关系 ·1560 ==========第162页========== 对(1),做普通除法 3x3-11ac2+18ax-33x-23x3-202 x2-3x+4 9x2+18c 9a2+6c 12x-3 12x-8 5 .'.商是2一3ac+4,余数是5.对(②),运用综合除法. 3-11+18-3128 12 -6 +8 3-9 +12,+5 .'.商是3x2-9心十12,余数是5. 比较(1)和(2)中求得的商和余数,可见f(心)除以3一2 所得的商,恰巧是f()除以。一景的商的寻:而它们的余数相同. ,上面从比较中所得出的结论,可以直接根据除法的意义得出.事实上,设f(回)除以。一号所得的商是《倒而余数是,那末,根据除法的意义,有 f)-(e-号)q(@+r 号3x-2)Q(@)+r (3a-2)[공()]+r". ·157• ==========第163页========== 李 由此可知:f(e除以3-2所得的商应是日Q(m)面余数仍 是. 由此容易得出,求多项式f(x)除以ac一b的商Q(x)和余数?的法侧是: 1°先用综合除法求出f(a)除以心一。所得的商Q()和余数m'; 2°以a除Q'()即得所求的商Q(w); 3°即以1°中求得的余数'作为所求的余数?.例如,上面问题中的(①),可以列成如下的算式来解: 3-11+18-33 +2-6 +8 313-9 +12,+5 1-3 +4 ·.商是2-3心+4,余数是5. 例3.已知f(花)=25-34+23-22+1有二重根1 和单根一豆把f(✉)分解为一次因式. 分析根据已知条件,显然f(x)能被(x-1)2(2x+1)所整除,故可做除法而求得它的另一个二次因式,再分解因式即得. 【解12-3.+2-20+11 1 +2-1+1-1-1 2-1+1-1-1,0 1 ¥十2+1+2+1 1 2+1+2+1, 0 2 -1十0-1 22 0+2.0 10·+1 ·158· ==========第164页========== ∴.f(w)=(c-1)2(2c+1)(a2+1) =(-1)2(2c+1)(c+)(-). 习题52(2) 1,应用综合除法,求商和余数: (1)(2x3-3x2+8x-12)÷(2x-3); (2)(24+33+3x2+5x-1)÷(2c+1), 2.已知f(x)=6x4-心8-6x2--12, (1)求证f(x)能被2一3整除,也能被3x+4整除;(②)把f(x)分解成一次因式. 3.已知f(x=2x3+7x2+k有因式2x+3,决定k的值,并且把f(x)分解成一次因式, §5·3…元n次方程 把%次多项式 f(x)=aoc"+1+…十an 用等号与0联结起来,就得到方程 aoc"+a1x"-1+…+an=0, (1) 式中是自然数,a,a1,a2:…,:是任何复数,ao≠0.,这样的方程称为关于心的一元n次方程,如果%>2,通常也称之为高次方程. 现在,利用§51里讲过的关于多项式的性质,来研究一般的一元加次方程的重要性质.为了方便起见,将方程(①)简单地记作 f(x)=0. 年 1。一元n次方程的根根据方程的根的意义,我们知道:如果f()=0,那宋a是方程f(c)=0的根;反过来,如 ·159· ==========第165页========== 生 果a是方程f()=0的根,那末f(a)=0.这就是说,方程f(心)=0的根与多项式(c)的根是完全一样的.由此,从多项式∫(c)的根的性质,可以直接得出 多项式f(心)的每一个一次因式的根,都是方程()=0的根,除此之外,方程f(心)=0不再有其他的根 多项式f(心)的k重因式心一a的根,称为方程∫(x)=0的飞重根.由此,进一步可以得出: 方程f(w)=0有k重根α的充要条件是多项式f(a)能被(c一a)整除但不能被(w一a)+1整除; n次方程f()=0有且只有见个根(飞重根算作是k个根) 例1.已知方程 5-a2-a6+1=0 有二重根一1,求系数a的值. 分析既然一-1是方程的二重根,那末多项式f(x)=x-a2一ac+1必能被(c+1)整除,但不能被(+1)3整除.因此,f(x)除以 +1所得的余数要等于0,再除以”+1所得的余数也,要x+1 等于0,但织,再除以:+1所得的余数n不能等于0,根据以 上要求,求出"1,T2,r,就可决定a应取的值.[解] 1+0+0-a -a +1|-1 -1+1-1 +(a+1)-1 1-1+1-(a+1)+1, 0 .r1=0, -1+2-3 +(a+4) 1-2+3-(a+4),+(a+5) ..rg=a+5, -1+3-6 1-3+6,-(a+10) .y3=-(a+10). ·160e ==========第166页========== 令 fmg=a+5=0, (1) lr3=-(a+10)≠0. (2) 从(1)得=-5,代入(2)适合 由此可知,要使原方程有二重根一1,必须并且只须α= -5 习题53(1) 1,求证: (1)各项系数都是正数的一元”次方程没有正根; (2)偶次项的系数是正数,奇次项的系数是负数的一元次方程没有负根 [解法举例:(1)设这个一元%次方程是 f(c)=aox+a☑cn-1+…十an-1c+an=0, 、式中ao,,…,an-1,n都是正数. 再设:为任一正数,根据正数的积必为正数,正数的和必为正数,可知: f(a)=aoa"+an-1+…十an-1a+an>0 所以a不可能是方程f(w)=0的根.这就是说,方程f(x)=0没有正根.] 2.已知方程 f(x)=x+8x2+5x-50=0, (1)求证一5是这个方程的二重根; (2)求这个方程的另一个根. 3.已知方程 f(x)=x5-5a+7ax-2ac2+4c-8=0, (1)求证2是这个方程的三重根; (2)求这个方程的另外两个根. 4.已知方程 f(c)=2x4-11x3+18x2-ag-2a=0 有三重根2, ·181· ==========第167页========== 手 (1)求a的值; (2)解这个方程. 2。一元n次方程根与系数间的关系对于一元二次方程,我们已经知道,存在如下的性质:如果一元二次方程 a心2+bx十c=0 的两个根是1和,那末 G 1=a・ 现在把方程的这一性质推广到一般的一元肌次方程. 定理如果方程 f(c)=aoc"+a1m-1+…+au-1c+a.=0(a0≠0)的%个根是1,xg,…,x-1,化,那末 ◆ 1十x2十…十n-1十n=-4, ao 19+123+…十x-1= G1购-1a=(-1)n [证1因为1,x2,…,心a-1,m是方程f()=0的根,所以多项式f(x)必定含有%个一次因式: 化一01)比-t2,··)化一Ea-1y优一4. 于是 aoc"十a1x"-1+…十aa-1+am =an(G-1)(e-x2)…(x-化H-)(x-心n). 把上式的右边按照心的幂展开,得 ·162· ==========第168页========== a十ac"-1+…+at-1+am =ao"-ao(1十c2十…+cn-1+cn)"-1 十a0(c1cg+1c3+…+cn-1n)c"-2-…+(-1)aoc1cg…Un-1C. 这是-个恒等式,根据§51定理4的推论,可得 1=-Q0(w1+w2十…+c-1+n),ag=00(C12+c1c3十…+cu-1c#), an=(-1)"x1Ug…-1x. 1十+十n=一 a2 G1化2+U1化3+…十C-1Cn=a0’ (购…-1=(-1)na 例2.已知方程2a3+5x2-4x一12=0有重根,解这个方程. 分析既然这个方程有重根,它至少要有两个根相同.因此,可设这个方程的三个根是a,a,B,利用根与系数的关系,即可确定α和B的值. [解]设这个方程的根是&,a,B,那末 (atat8--5 (1) 、 a2+a8+a8=-2,' (2) 2B=6, (3) 从(1),得 6一2. (4) 代入(②),得 •163 ==========第169页========== a2+2a(-营-2a)=-2 江 就是 3a2+5a-2=0. ..a1=-2, 3 代入(4),得 8=-19 6 「a1=-2 把 形代入(3)是适合的,所以它是列出的方程组的β1-2 解。 1 02=3 把 代入(3)不适合,所以它不是列出的方程19 B2=6 组的 所以,原方程的三个根是一2,一2和之3 例3.已知方程 c4-4w3-24x2+56+52=0 的四个根成等差数列,解这个方程. 【解]设这个方程的四个根是 a-36,a-b,a+b,a+36, 那末,根据根与系数的关系,可得方程组:(a-3b)+(a-b)+(a+b)+(a+3b)=4, (a-3b)(a-b)+(a-3b)(a+b)+(a-3b)(a+3b)+(a-b)(a+b)+(a-b)(a+3b)十(a+b)(a+3b)=-24,(a-3b)(a-b)(a+b)+(a-3b)(a-b)(a+3b) +(a-3b)(a+b)(a+3b)+(a-b)(a+b)(a+3b)=-6,(a-3b)(a-b)(a+b)(a+3b)=52.•164· ==========第170页========== ¥ 就是 r4a=4, ) 3a2-562=-12, a(a2-5b2)=-14, (a2-9b2)(a2-b2)=25. 4) 从(1)得a=1.代入(2),得 3-562=-12,∴.b=3,.∴.b=±√3.fa=1 23或8、3代入③和④都适合,所以它 是列出的方程组的解.由此可知,原方程的四个根是 1±3√3,1士√3. 说明1.已知四个根成等差数列,虽也可设成这四个根是a,a+b,a+2b,a+3b,但这样列出的方程组在求解时就较繁.上面解法里采取这样假设,可以从一个等式中求出一个未知数,计算也就比较简单。解这类问题时,应该善于掌握这种技巧. 2.本题中列出的方程组只有2个未知数:和b,但是有四个方程.求α和b只需选取较易计算的两个方程(一般,可取次数较低的两个方程)即可,但这样求出的解还必须代入其他两个方程中进行检验,经验算后能适合的才是方程组的解 习题53(2) 1.已知方程2x3-5ax2-4c+12=0有二重根,解这个方程 2.已知方程12x3-8x2-3+2=0有两个根互为相反数,解这个方程。 3.已知方程c4-4x3+10x2-12x+9=0的根两两相等,解这个方程。4.已知方程x8-(2+3)x2-(1-5)x+2(1-)=0的三根成等 比数列,解这个方程. 5.k是什么数时,方程3+6x2+7x+飞=0的三个根成等差数列? ·165· ==========第171页========== 人 这些根是什么? 在学习一元二次方程时,曾利用根与系数的关系,不通过解一元二次方程 ax2+6x+c=0, 而直接计算它的两个根1与2的某些特殊代数式,如 1十1,好+喝 12 等等的值;以及作出另一个新的方程,使它的两个根与原方程的两个根间有某种指定的关系,如为原根的倒数,原根的平方,等等.这类问题也可推广到一元饥次方程 例4.已知方程3+p2十q十=0的三个根是,B, Y,求a2+B2+y2的值. 分析从(a+B+Y)2=a2+B2+y2+2(a3+By+ya)可以推得a2+B2+y2=(a+3+Y)2-2(ad+y+ya),所以只需先应用根与系数的关系求出a+B+y和a6+By+ya的值,代入即得 [解]·.'a2+B2+y2=(a+B+y)8-2(aB+By+Ya),根据根与系数的关系,有 a+B十Y=-p, a8+By+ya=9. ·.a2+B2+y2=(-p)2-2q=02-2g. 例5.已知方程3+pc2+9c+m=0的三个根是a,B, Y,求作一个以.a2,B2,y2为其根的新方程.分析设所求的方程是 3+lc2+mx+n=0. 那末,应有 -1=a2+62+y2, m=aB2+82y2+y2a, n=a232y2 因此,只需先仿照例4计算出2+B2+y2,a2+B2y+y2a2和a232y2 ·166● ==========第172页========== 年 的值,代入上式即得所求的方程,为了方便起见,不妨把所求的方程假设成 3c3-1x2+mx-h=0. 解1设所求的方程是 、x3-lx2+mw-%=0, 那末 7=a2+82+y2=(a+8+7)2-2(a8+8y+ya)=p2-2g, m=a282+82y3+ya =(aB+8y+ya)2-2aBy(a+8+y)=g2-2pr,n=a2B2y2=(a8y)2=r3.所以,所求的方程是 8-(p2-2q)2+(g2-2pm)x-m2=0. 习题53(3) 1.已知方程2x3-3-5=0的三个根是a,B,Y,求下列各式的值:(1)++分 (2)a2+B2+y2; (3) (4)B2y2+y2ax2+a2B2 2.已知方程x8+px2+qx+7=0的三个根是a,B,Y,求作三次方程(y)=0,使它的三个根是:, (1)-a,-B,-Y; (3)a,B,kY; ()-1 、 3.已知方程2x3-3x2+x+1=0的三个根是a,B,Y,求作一个三次方程,使它的根是: (1)a-3,B-3,y-3; 中 ()1,11 a-3’B-3’Y-3· [提示:(2)可以应用(1)求得的方程来作.] ·167· ==========第173页========== 于 §5·4实系数一元%次方程 在方程 f(x)=aox"+a"-1+…十am-1c+an=0(ao≠0)里,如果各项的系数都是实数,称这样的方程为实系数一元次方程.很明显,实系数一元次方程应该具有一般的一元%次方程的一切性质。 现在来研究实系数一元%次方程所特有的一个重要性质 我们知道,对于实系数一元二次方程 ac2+bx+c=0(a≠0) 来说,如果判别式b-4ac<0,那末它的两个根 a=b+4c和4-6-V0-4g2a 2a 是共轭虚数.这里可以看到,实系数一元二次方程的虚根是成对地出现的 下面来证明,对于一般的实系数一元%次方程,也都具有这样的性质. 定理如果实系数一元次方程f(如)=0有一个虚根a十bi,这里a,b都是实数,b≠0,那末它必定有另一个虚根a-bi. 通常称这个定理为实系数方程虚根成对定理, 分析要证明a一bi是方程f(x)=0的另一个根,只需证明多项式f()能被 [エ-(a+bi)][-(a-bi)] 整除.因为n次多项式(x)被x的二次式所除,得到的商是x的%一2次多项式,而余式最高只能是x的一次式,所以可先设 ●168· ==========第174页========== 美 f(x)=[-(a+bi)][然-(a-b)](x)+(px+g), 再应用已知条件4+bi是方程f()=0的根,来证明2,q都是0. [证1设用 g(x)=[心-(a+b)][c-(a-bi)]=x2-2a+(a2+b)除f(),所得的商是Q(c),余式是px+q.那末就有f(ax)=[-(a+b)门[c-(a-b)]Q(ac)+(pc+g).(1)因为被除式f()和除式g(c)的各项系数都是实数,而通过除法是不会得出虚数的(这在具体写出相除的除式中可看得很清楚),所以,商Q(心)和余式px+q的各项系数都是实数 因为a+bi是方程f(心)=0的根,所以f(a+b)=0.代入式(①),得 0=0+p(a+bi)+q. 即 (pa+g)+pbi=0. (②) 因为pa十g和pb都是实数,根据复数等于零的条件,从式(2)可得 fpa+g-0, (3) pb=0, (4) 因为≠0,所以从式(4)可知卫=0.代入式(3),得g= 0.因此,式(1)可以写成 f(x)=[-(a+bi)][-(a-bi)]Q(g), (5) 从而有 f(a-b).=0. 所以,a一bi是方程f(w)=0的根.定理得证 例1.已知方程2ax5一7x4+8ax3-2ac2+6c+5=0的两个根是2一元,,解这个方程 ◆169◆ ==========第175页========== 集 解]这是一个实系数一元五次方程,根据已知条件,2一,2是它的根,可知2+么,-i也是它的根. 设这个方程的第五个根是&,那末,根据根与系数关系,有 a+(2+)+(2-)++(-)-7. (1) 由此可知a=一受 所以,这个方程的五个根是-司,2士,士, 注意1.在应用虚根成对定理时,首先要判定原方程各项系数都是实数; 2.如果题设条件正确的话,应用式(1)求得的α必定适合,所以检验的步骤这里可略去 例2.、求作一个次数最低的实系数方程f(心)=0,使它 的一个根是-司+,另一个根是V习。 【解】因为-号+i是实系数方程f(知)=0的根,所以是-i也是方程子四)-0的报,要使方程f四)=0的次数最低,这个方程必须,并且只能有三个褪:-司+,一司-8和 √2.所以,所求的方程是 [e-(-3+)】[e-(2-小e-√2)-0. 即 [(+2》°-]ke-√)=0,(2+a+)水e-√)-0,+(1-)+(-√)ー-0 170 ==========第176页========== 1 注在解本题时,当确定(x)=0是三次方程以后,也可设这个方 程是 x8-1c2十mx一n=0, 再应用根与系数的关系,求出 (-+i)+(--)+√=-1+ゾzm=va[(ー+)+(ーー)+(ー+(--) =-V②,5 m=(-是+-豆-Vg-v2.从而写出所求的方程, 习题64 1.(1)已知方程3x8-4x2+x+88x0有一个根是2+√7i,解这个方程; (2)已知方程x3-(3+√3)x2+2(3+√3)x一4(1+V3)=0的一个根是1+√3i,解这个方程 2.求作一个次数最低的实系数方程,使它含有下列的根: (1)-2,2+i; (2)2+i,-1+; (3)1-V2i,i,0; (4)-1+W3i,1-√W3. 3.已知实系数方程f(x)=0的开头两项是3+心,并且它有一个根是-1+√2i, (1)写出这个方程; (2)求这个方程的另外两个根. 4.已知方程x4-4c3+11x2-14x+10=0的一个根是a+bi的形式,另一个根是a+2bi的形式(a,b都是实数,b+0),解这个方程. 5.根据实系数方程虚根成对定理,证明: (1)一个奇数次的实系数方程,至少有一个实数根; (2)如果a+bi是一元u次方程的k重虚数根,那末a一bi也是这个方程的k重虚数根。 •171• ==========第177页========== 大 §55有理系数一元%次方程 在方程 f(c)=o+a1"-1+…+an-1+an=0(ao≠0)里,如果各项的系数都是有理数,这样的方程就称为有理系数 一元次方程。特别,如果各项的系数都是整数,这样的方程就称为整系数一元儿次方程、很明显,任何一个有理系数的 一元%次方程,都可变形成为与它同解的整系数一元%次方程 有理系数一元次方程,除掉具有实系数一元%次方程所具有的一切性质以外,还具有一些特殊的性质 1。有理系数方程f(c)=0的有理根对于有理系数方程f(x)=O,最常碰到的问题是要找出它的有理根,或者确定它没有有理根.因为,任何一个有理系数方程都可以变形成为与它同解的整系数方程,所以这个问题也就可以归结为怎样来找出整系数方程 f(c)=b心n+b1a-1+…十bm-1心+b4=0(io≠0)的有理根. 首先,我们来看几个具有有理根的整系数方程,考察一下它们的根与系数之间有怎样特定的关系具有整数根2,一3,一5的三次方程是 (心-2)(w+3)(x+5)=0, 就是 3+6a2--30=0. 这里,可以看出,2,3,5正是这方程的常数项30的三个约数. 具有分数根 受-日,一号的三次方程是 。172· ==========第178页========== 生 (2c-1)(3e+1)(6x+1)=0, 就是 30x3+2-6a-1=0, 这里,可以看出,这三个根中的分母2,3,5恰巧是这方程的 三次项系数30的三个约数 具有分数根和号的二次方程是 (2c-3)(7c-5)=0. 就是 14a2-31x+15=0, 这里,又可以看出,这两个根的分子3和5是这方程的常数项15的约数,而分母2,7却是这方程的二次项系数14的约数.从上面的观察中,可以得到启发。如果既约分数?是方程f(心)=0的根,那末p应该是常数项的约数,q应该是最高次项的系数的约数 定理1.如果既约分数”是整系数方程f(c)=b"+b1m-1+…+bw-1+bm=0(bo≠0)的一个根,那术p-一定是bm的一个约数,g一定是bo的一个约数. *T证]因为卫是整系数方程f(x)=0的一个根,所以 bo(日”+(日+…+b-:(日)+,-0, 、 两边乘以q”,得 bop"+61pm-1g+...+on-ipg"-1+8ng"=0.∴.bng”=-2p(bopn-1+b1qpn-2+…+bn-19n-1). b.9=-(bopm-1+b1gpm-2+…+bm-19-1). p 这个等式的等号右边是一个整数(由于b0,b1,…,bn-1,y,q都是整 173· ==========第179页========== 数),所以b,0”应该也是一个整数,于是b9”应该能被p整除。但q与p互质,因此g”不能被p整除,从而知bn必须能被p整除.即p为bn的约数 类似地,从 bo2”=-(b1p"-1+…+b-1pgn-2+bn9m-1).2 可以推导出q必须是b,的约数. 从上面的定理,容易推出:推论1,如果整系数方程 f()=bo+b1-1+…+bn-1+bn=0 有整数根p,那末p是常数项bu的约数 推论2.如果最高次项的系数为1的整系数方程 f(c)=xm+b1c-1+…+b-1c+bw=0 有有理根,这种有理根只能是整数 我们知道,任何一个整数的约数只有有限个,因此,应用上面的定理,通过试探的方法,就可以把整系数方程的所有有理根逐个找出,或者证实它不具有有理根.但是必须注意:在应用定理时,首先要判定原方程是整系数方程 例1.求方程f(c)=2x3+3ac2-8c+3=0的有理根.分析这里,最高次项系数2的约数只可能是士1,士2,常数项3的约数只可能是士1,士3,所以,原方程的有理根只可能是士1,士3,±分,±》.容易看品,f①=0,所以1是原方程的根.这样,只要 用综合除法,求出f(x)除以x一1的商g(x),再解二次方程gx)=0即得. 【解]根据上面的定理,所给整系数方程f(心)=0的有 理根只能是士山,士3,士日,士》今用试除法米有下: ·1740 ==========第180页========== 年 2+3-.8+31 +2+5-3 2+5-3,0 可见=1是原方程的根.原方程其余的根就是方程 22+5x-3=0 的根.解这个方程,得x=-3或者如=2·1 因此,原方程的有理根是1,-8子 、注意1.因为任何一个整数总有约数士1,所以在求整系数方程f(x)=0的有理根时,总要考察1或者-1是不是它的根.对于这一步,只需直接计算[可以应用习题5.1(2)第4题的结论]f(1)和f(-1)是不是等于零而加以肯定. 2.当求出整系数方程f(x)=0的一个有理根x以后,用综合除法求出(x)除以x-a的商g(心),就会得到一个降次方程g(x)=0,所以,下一步只要再解这个方程就可以了.特别是,如果g(x)=0已是二次方程,就不必再用试探的方法来解。 例2,求方程 f(x)=2x5+3x4-15c3-26a2-27x-9=0 的有理根,然后再求出其余的根. 【解]原方程可能有的有理根是: ±1,士8±9,±号, ,+る,王? .·f(1)≠0,f(-1)≠0,.∴.c=1和=-1都不是它的根. 对于=3. 2+-3-1ǒ-26-27-9|3 +6+27+36+30+9 2+9+12+10+3,0 ◆175· ==========第181页========== 于 可知心二3是原方程的根,并得降次方程 f1(x)=2w4-+9x3+12x2+10w+3=0 这个方程的各项系数都是正数,故不可能有正数根.因此,它 可能有的有理根是-山一品,一子,一号。但是=-1不 是f()=0的根,它当然不可能是f1(x)=0的根. 对于x=-3. 2+9+12+10+3-3 ー6-9-9-3 2+3+3+1,0 可知心=一3是它的根,并得降次方程 f(w)=2x3+3ax2+3+1=0, 这个方程的可能有的有避根只能是一分,(想一想,这 是为什么?) 对于x=一1 2+3+3+1 2 ー1-1 -1 212+2+2, 0 1+1 +1 可知=一是它的根,并得降次方程 f3()=x2+c+1=0, 解这个方程,得xs-1±√3龙 2 答原方程的有型根是3,-3,一立,另两个根是二1生y e176· ==========第182页========== 注在实际解题时,可以略去上面解中所作的说明,而只列出如下的算式: 2+3-15-26-27 -91 3 +6+27+36+30+9 2+9+12+10+3, 0 3 -6-9-9-3 2+3+3+1,0 12 -1-1-1 212+2+2, 0 1+1+1 所以,原方程的有理根是3,-3,一分·解方程x2+c+1=0,得原方 程其余的根是一1±V3:2 例3.证明方程23+2c一1=0没有有理根.[证]这个方程可能有的有理根是士1,士:今 f(1)=3≠0,(-1)=-5≠0, f()-+0,f(-)=-2즉o.虹,±都不是它的根,所以原方程没有有理根、 习题55(1) 1.求下列方程的有理根: (1)3-5x2-2c+24=0; (2)2c3-x2-5c-2=0; (3)12ax4-20ax3-11x2+5x+2=0; (4)2c5-9+33+10x2+21c+9=0,2,求证下列各方程都没有有理根: (1)x3+G+3=0; (2)x5-5=0; (3)0-x+6=0; (4)x8-x2+3-2=0, ·177、 ==========第183页========== 3.求证下列方程都没有整数根: (1)16a4-12c3+4x2-3x+1=0; (2)21z+10x4+x3+.19x2+5x+6=0 [提示:(2)只需验证f(-1)≠0,f(-2)≠0,f(-3)+0.] 4.解下列方程: (1)5+2x4+2x3+42+5x+2=0; (2)4+5c+8x2+x-15=0; (3)2x4-5x3+3x2+4x一6=0; ()--2+1=0. [提示:先求出有理根,再解降次方程.] 5.分解下列各多项式的有理系数因式: (1)3+3x216x-48; (2)Gx4+19x3-7c2-26x+12 [提示:先求出各多项式的有理根,再用因式定理.] *2。有理系数方程∫()=0的二次不尽根我们知道,在方程 ac2+bx+c=0(a+0) 里,如果a,b,c都是有理数,b2-4ac>0,且b2-4ac不是一个完全平方数,那末它的两个根 =ーb+Vb2-ac和2=-bV√24ac 2a 2a 都是无理数.这样的根通常称为二次不尽根.这里可以看到,象二次方程的虚根一样,二次方程的二次不尽根也是成对地出现的,现在来证明,一般的有理系数方程f(x)=0,都具有这样的性质定理2.如果有理系数方程 f(x)=axn+a1n-1+…十an-1g+an=0(a÷0) 有无理根a+√石,这里a,b都是有理数,√石是无理数,那末它一定还有一个无理根a-V万. 这个定理的证明,可以仿照虚根成对定理的证法, 【证】设 ◆178• ==========第184页========== 2 f(x)=[-(a+Vb)][-(a-Vb)]e(x)+p心+g.·(1)这里,因为除式[x-(a+√6)][x-(a-Vb)]=x2-2ax+a2-b是的有理系数二次多项式,被除式(x)是x的有理系数”次多项式,而两个有理数的加减乘除仍是有理数,通过这里的除法是不会得出无理数或虚数的,所以商Q(x)是x的有理系数n一2次多项式,而余式中p和g也都是有理数. 因为&+V石是方程f(x)=0的根,所以f(a+√b)=0.代入式(1),得 0=0+(a+√6)+q. 就是 (pa+q)+pW/石=0, (2) 因为pa+q是有理数,而pv石是无理数,所以从式(2)可推出 [pa+g=0, (3) lpvb=0 (4) 由式(4),因为√石≠0,所以p=0.代入式(3),得q=0.因此,式 (1)可以写成 f(x)=[x-(a+Vb)[c-(a-√6)门g(x), (5) 从而有 f(a-W石)=0, 所以x=a-√石是方程f(c)=0的根.定理得证把上面的定理加以推广,还可以得出以下的定理①:定理3.如果有理系数方程 f(x)=aoxn+a1n-1+…十an-1c+an=0(a+0) (1)有一个无理根√a+√石,这里a,b是有理数,而√a,√b是无理数,那末它一定还有三个无理根: Va-Vb,-Va+Vi,-√a-√/石. (2)有一个虚根Va+√石i,这里a,b是有理数,而√a,√石是无理数,那末它一定还有三个虚根: Va-V石i,-VWa+√bi,-Wa-/bi.注意定理2不能推广到任意无理根的情形。例如,有理系数方 ①证明较繁,这里把它略去了。 ●179· ==========第185页========== 程f(x)=0如具有三次不尽根x=a+/石(a和b背有理数,/方为无理数),但推不出f(x)=0必具有根x=a-/万。例4.已知方程 f(x)=x6-2x3+8x2-32x+64=0, 有一个根√3+i,解这个方程 [解】这是一个有理系数方程,它既有一个虚根√③+i,必定还有另外三个虚根√3一i,一√3+i,一√3-i.可以推出,()能被 [x-(V√3+)][然-(V3-)][x-(-√/3+)][x-(-V3一)] =x4-42+16 整除.做除法,求出商是x2-2c+4,再解二次方程 x2-2x+4=0, 求得另两个根是心=1+√3i和=1-√③i.所以原方程的六个根是√3士i,-√③±i和1±W3i. 注本题在确定方程(x)=0有四个根√3士i,一√3±i后, 也可以先假设另两个根是α和B,根据根与系数关系列出方程组,从而 求得a:和B的值 例5.求作一个次数最低的有理系数方程(x)=0,·使它的一个 根是V3+2√乞. [解】既然√3+2W2(就是√3+√区)是有理系数方程f()=0的一个根,根据定理3,可知所求的方程至少还要有另外三个根: √3-2V√z,-V3+2y2,-√3-2W/2. 所以,所求的方程是 [x-(√3+2W2)门[x-(√3-2V/2)][x-(-√3+2V2)]×[c-(-√3-2V/2)]=0,就是 [(x-V3)2-8][(c+V3)2-8]-=0,[(x2-5)-2V3x][(2-5)+2V3x]=0, x4-22x2+25=0, 注对于这类问题,还可用另-·种解法.因为,具有根√3+2√②的最低次方程是⑧=√③+2√2.现在要使所求的方程各项系数都是有理数,可以用逐次平方的办法,把这个等式的系数有理化。 ·180· ==========第186页========== 3 ,x-√3=2V√2, 两边平方,得 2-2√3c+3=8, 即 x2-5=2V√3x. 两边再平方,得 4-10x2+25=12ax2, 所以 x4-22x2+25=0, 这就是所求的方程. 习题55(2) 1.已知1+√2是下面这两个方程公有的一个根: (1)3-4x2+3x+2=0, (2)3-2(√2+1)x2+(1+4V2)x-2=0,解这两个方程。 [提示:(①)可以应用本节中的定理2来解,(2)不能应用定理,应先求出一个降次方程后再解.] 2.已知有理系数方程x3-3a2+ax+b=0有一个根是1-V2,求a,b的值,并解这个方程, 3.求作含有根√③一2和1+i的次数最低的有理系数方程. 4.求证:1+V2是方程 f(x)=x-3ax4-2x3+6x2+5x+1=0 的…个二重根,并解这个方程。 5.已知方程x5+-2ax3-2ax2+25c+25=0有一个根是√3一√21,解这个方程.、 §5·6几种特殊类型的高次方程的解法 、 作为本章的结束,下面介绍几种特殊类型的高次方程的解法 1。二项方程形如 aoc"-卜an=0 ·181· ==========第187页========== + 的方程称为二项方程,这里n是自然数,ao和an是不等于零的复数 很明显,这类方程都可以化成 do 从而应用复数的开%次方的方法解出.例1.解方程(-2-2)4-8=0. [解]原方程可化为一-2”2 8 即 心4=-2+22 .g-2z(8+n8) 3i +2kπ.=/8 4 +2kπ +isin 4 4 (k=0,1,2,3). 就是 -8(cos+s如m蹈) 16 》 心阳=/8 +m〉 (c0916 c3=8 cos 19死+isin19 16 16 a4=/8co827m.+isin27r 16 16 当二项方程"+b=0的%是3,4或6,且.b是实数时,还可用因式分解的方法来解 例2.解方程: (1)4+16=0: (2)x+a=0. 。182· ==========第188页========== 单 解](1).·4+16=c4+8ax2+16-8x2 =(x2+4)2-(√8c)2 =(a2+2√2x+4(ac2-2/2x+4), 所以,原方程可化成 (2+2√2x+4)(x2-2√2x+4)=0.由x2+2√2G+4=0,得 2 1,2=-√2士√2i; 由x2-2√2x十4=0,得 ,4=√2±√/2i. (2)‘.·x6+a5=(c2)3+(a2)3=(x2+a2)(c4-a22+a4) =(c2+a2)(c4+2a2ax2+a4-3a2x2)=(x2+a2(x2-√/3a+a2(aw2+√/3aa+a2), 所以,原方程可以拆成3个方程: x2+a2=0, 从而得c1,9=士G; 1 2-/3ac+a2=0,从而得8,4=(W③±)a: 心 c2+√3ac十a2=0,从而得6,8=-(v8士)a, 习题56(1) 1.解下列二项方程: (1)x4-4i=0; (2)5-1=0; (3)x3-2+2i=0; (4)(1-V3)x6-2=0; 向(号+)-V- 2.用因式分解的方法,解下列各二项方程: (1)x3-64=0; (2)8ax3+27=0; (3)814-16=0; (4)x4+625=0; (5)6.-64=0; (6)64x6+1=0. ·183· ==========第189页========== 上 2。三项方程形如 a2n+6xm+C0 的方程称为三项方程,这里是自然数,a,b,c是不等于零的复数 当儿=2时,这个方程就是在代数第二册里学过的双二次方程 aw千b心2+c=0, 所以,三项方程也可以象双二次方程一样,令”=y而化成二次方程 a2+by+c=0. 这样,在这个方程里,求出y的两个根1,以后,只要再解下面的两个二项方程 心=1和x"=y3 即得原方程的解 例3.解方程12x8-8r3+1=0. 【解]令x3=思,得 12则-8y+1=0. 解这个方程,得 6 从3-1 从3-1 注从心=1的三个根是1,-1+√3i,一1-V3i,直接 直接可 2 以知道:方程-专的三个根分别是-1的三个根聚以经即 0184● ==========第190页========== 足 习题56(2) 1.解下列方程: (1)x8-5x3+6=0: (2)36x3-13x+1=0; (3)x6-2x3+2=0: (4)x3+8x4-9=0, 2.解下列方程: (1)(34+3)4-13(4+3)2+36=0; (2)(x2+3)4+12(2+3)2-64=0,[提示:分别令4+3=y,r2+3=y.]*3。倒数方程我们来解下面的方程: x4+33+2a2+3x+1=0, (I) 这个方程的各项系数有一个重要的特征:与首末两项等距的两项系数相同.根据这个特征,可把系数相同的两项结合起来,即得 (x4+1)+3(3+x)+2x2=0 (1) 因为x=0显然不是方程(1)的根,用x2除方程(1)的两边,可得 (2+)+3(+)+2=0. (2) 容易看出,这里 +是-(+》-2s是(+》'-2 所以,如果令x+上=,由式(②)可以得出一个关于y的二次方程 y2-2+3y+2=0. 就是 y2+3y=0. (3) 由此,从式(3)可得 1=0,2=-3. 把这两个值分别代入+是-就得到两个关于:的方程(i)G+1=0.由此,得2+1=0. .出i,=-。 (i)x+1=-3.由此,得2+3x+1=0. 0185· ==========第191页========== 4=-3+V万 -3-/5 2,x4=-2 这就解得了这个方程的4个根, 从求得的4个根中,还可以看出: 0·02=i(-2)=1, 这就是说,这个方程的每两个根互成倒数.象(T)这样的方程,把它称 为第一类型的偶次倒数方程 一般地说,对于2m次方程f(x)=0,如果距首末两项等距的每两项系数相同,这样的方程就称为第一类型的偶次倒数方程.这类方程,都可以仿照上面的方法,令x+是=,化成一个关于y的m次方程.因 此,如果得到的关于y的m次方程可解的话,那末原方程也就可以解出,并且求出的根总是每两个根互为倒数. 现在再来解下面的方程: x5+4x4+5x3+5x2+4+1=0 (II) 这个方程的各项系数也同样具有特点:与首末两项等距的两项的系数相同. 根据根的意义,如果令x=一1,方程(II)里左边的多项式的值是 0,所以方程(II)有一个根心1=一1,做综合除法: 1+4+5+5+4+1|-1 -1-3-2-3-1 1+3+2+3+1,0 得降次方程 x4+3x3+2x2+3x+1=0 这是一个第一类型的偶次倒数方程,它已在上面解出,从而求得这个方 程的解是: 6=-1,8=士i,46=一3去V52 象上面这种类型(II)的方程称为第一类型的奇次倒数方程 一般地说,对于2+1次的方程f(x)=0;如果距首末两项等距的每两项的系数都相同,这样的方程就叫做第一类型的奇次倒数方程.这 ·186· ==========第192页========== 类方程总有1个根是花=一1,并且把f(x)除以x+1以后,总可得出 一个次数是2m的第一类型偶次倒数方程.例4.解方程 f(c)=-2ax4-5x8-5c2-2+1=0. 【解] 1-2-5-5-2+1-1 1+3+2+3-1 1-3-2-3+1,0 ∴.f(x)=(+1)(x-3x2-2x2-3x+1)=0.由x+1=0,得1=-1;由0-3x3-2ax2-3+1=0,得 P (e2+)3(e+》-2-0. 令+ッ,期+=-3,代入得 y2-3y-4=0,∴。1=ー1,y2=4. 从ェー-,得 x2+8+1=0, 6=-1生V③; 2 从x+1=4,得 2-4x+1=0, x4,5=2士√3 由此,得原方程的5个根是-1,2士V3,-1±③立2 *习题5·6(3) t 解下列方程: 1.x4+x3+2x2+c+1=0 从 2.2c4+78+7c+2=0, 33x4-7x3-7x+3=0 4.2x5+5c4-13x3.-13x2+5x+2=0, 0187· ==========第193页========== 5.x5-34+x8+x2-3x+1=0 6.2x6-x5-10.x4+13x3-10x2-x+2=0 [提示:先令+是=,则+子=-名,+是-(e+》 -3(红+》=8-3别,由此号出-个关于y的三次方程.小我们再来解下面这两个方程: x6+3+04-x2-3-1=0, (III) x5+2x4-x3+2-2x-1=0 (IV) 这两个方程都有这样的特点:距首末两项等距的每两项系数互为相反的数。 这样的方程,分别称为第二类型的偶次倒数方程和第二类型的奇次倒数方程. 容易看出,把心=1和x=一1代入方程(III)的左边,都能使这个多项式的值是零,.所以x=±1是方程(II)的根.为了求方程(III)的其他4个根,先进行综合除法: 1+3+1.+0-1-3-111 +1+4+5+5+4+1 1+4+5+5+4+1,0-1 -1-3-2-3-1 1+3+2+3+1,0 这样,从方程x+3x3+2x2+3x+1=0,就可求得这四个根是士i, ~3±V5 2 也容易看出,x=1是方程(IV)的一个根.·做除法: 1+2-1+1-2-11 +1+3+2+3+1 1+3+2+3+1,0 这样,从方程 x4+3x3+2x2+3x+1=0 同样可求得方程(IV)的另外4个根是士i,一3去Y5.2 上面的例子表明:第二类型的偶次倒数方程总有两个根土1,第二 ·188◆ ==========第194页========== 文 类型的奇次倒数方程总有一个根1,其他根可由第一类型的偶次倒数 人 方程求出 正因为这样,通常也把第一类型倒数方程称为标准型的倒数方程.例5.解方程15x+34x4+15x3-15x2-34:c-15=0. 【解】显然,这个方程有1个根是1.做除法: 15+34+15-15-34-151 +15·+49+64+49+15 15+49+64+49+15,· 令15x4+49x3+64x2+49c+15=0,两边除以2,得 15(。+)+4(e+)+64=0.设+=ッ,则+子ー2代入,得 15y2+49y+34=0. (15y+34)(y+1)=0. 1-,%-34 由x+1=-1,得2+x+1=0, .=-1士V3: 2 。 由+是-器得152+3+150,就是34 (5x+3)(3x+5)=0, =-号,或-3 5 由此,得原方程的根是:1,一号,-多,二1生y,2 注这方程有3个有理根.这三个有理根也可用试探方法求出.然后再解二次方程求出两个虚根。但,这样做一般来说不及上面这种解法简单。 r *习题56(4) 解下列方程: ·189· ==========第195页========== 1.25++x3-2x2-x-2=0. 2.3ax5-10x4+78-7x2+10x-3=0 3.6ax6+5x5-44ax4+44x2-5x-6=0 应用解倒数方程的方法,还可以解根是互为负倒数的一种特殊方程。例如,解方程 3c4-4x8+3x2+4x+1=0, (1) 这个方程距首末两项等距的偶次项的系数相同,而奇次项的系数互为相反数.根据这个特点,把这个方程的各项重行结合起来,得 (x4+1)-4(3-x)+x2=0. (2) 因为就+0,用x2除方程的两边,得 (+)4(x-)+1-0. (3) 令。-子-%那末就有 2+-(e-》+2是-r+2 代入式(3),得 y2-4y+3=0, (4) .1=1,y=3. 从s-2-1,得2ーェー1=0, y5; 从8-1=3,得2-3x-1=0, ,4=3±√13 2 例6.解方程2x8-115+12x+13ax-12x2-11x-2=0. 【解】首先,把与首末两项等距的两项结合在一起,得 2(6-1)-11(x3+x)+12(4-x2)+13a3=0. 因为x≠0,用x除方程的两边,得 (-)-(+)+(-2)+13-0. ·190 ==========第196页========== に,則+3-+。而サ=ー-3+ 五“所以 x8-1 =y+3则.由此可得 2(y3+3y)-11(y+2)+12y+13=0, 即 2y3-11y+18y-9=0 应用确定有理数根的方法,可求得上述方程的根 h=-1,-3,0-是 从而得 공-고,~-3或금-를 解上面三个方程,可得原方程的六个根是 ,25,4=3生V丽 1±W5 1 2,=2,=ー 习题56(5) 解下列方程: y 1.44-123十x2+12x+4=0. 2.4-3x3+3x+1=0 3.2x4+78+2x2-7t+2=0 4.2a8+9x5+7-12a3-7x2+9x-2=0. 本章提要 1.多项式f(x)=aox”十41xn-1十…+a的一些量要性质 (1)余数定理:f(x)除以x一a的余数是f(). 余数定理的推论:f(c)能被x~a整除的充要条件是f(a)=O.(②)标准分解式 f(x)=ao(c一a41)(x-a2)…(-ag (这里1,2,…,伦是自然数,1+2十…+!=外). (3)两个多项式()与g(x)恒等的充要条件是:同次项系数相等。 ·191· ==========第197页========== 2.一元n次方程 (1)标准形式:f(x)=Qox”+1-1+·+an=0(2是自然数,ao,41,…,an是复数,且4卡0). (2)根的个数定理:有、并且只有n个根 (3)重根的判别:f(c)能被(x一a)整除,而不能被(一a)+1整除,则a是k重根. (4)根与系数的关系: 1十2十…十n=-生西+g十…+xn-1必n=2 0●●0。t小000小0t10◆e◆e0·040 心2xn=(-1)nn 8.实系数一元n次方程 (1)标准形式:f()=0(ao,a1,…,an都是实数,ao卡0)(②)虚根成对定理:如果a+bi是其根,则u-bi必也是其根(a,b是实数,b≠0) 4.有理系数一元n次方程 (1)标准形式:f(r)=0(ao,a,…,a都是有理数,ao≠O).特例:整系数一元%次方程g(g)=bo”+b1x”-1+…十bn=0(bob1,…,bn都是整数,bo≠0). (2)有理根的求法: (i)先化成整系数方程g(x)=0; (i)如果f()=0有有理根亚=号卫必为.的约数,g必为6如的约数。由此,可用试探方法,把有理根逐一找出。, (③)关于无理根和虚根的定理: (i)如果a+√石是它的根,则a-√石必也是它的根;(近)如果√a十√b是它的根,则 Va-√i,-Va+vb,-√a-√万 必都是它的根; (ii)如果√a+√石i是它的根,则 .Va-V6i,-√a+Vbi,-Va-√ii ·192· ==========第198页========== 天 必都是它的根. 上 (a,b是有理数,√,√6是无理数.) 6.几种特殊类型的高次方程的解法 (1)二项方程:aoxm+am=0(ac≠0). (1)一般方法一80,-名 (i)因式分解法:适用于n等于3,4,6时. (2)三项方程:ax2n+bx"十c=0(a+0). 解法:先令x”=y,得y2+by+c=0;求出根1,2,再解二项方程x”=1和x”=y2.“(3)倒数方程(1)标准型倒数方程 ao(x2m+1)+a1(x2m-1+x)+…+am-1(xm+1+x-1)=0.解法:两边除以,再令三+子-,导出一个关于y的m次方 程,解得y后再求出. ((ⅱ)其他类型的倒数方程 解法:必定具有根1或一1;把根1或一1除去后,就可导出一个标准型倒数方程,再用解标准型倒数方程的方法来解. 复习题五 1.用综合除法求商和余数 (1)(3x3-4x2+7-14)÷(3x-1); 之 (2)(2+33+42+11)÷(2x+3). 2.已知f(x)=5-124+15x-7, (1)求f()除以x一1的余数; (2)求f(6). 3.(1)已知多项式6ax9-19x2+ac+b能被3x+1整除,也能被2x+3整除,求a和b的值; (2)已知多项式ax3+b2-47x-15能被6x2-7x-3整除,求a和b的值; (3)已知多项式ax4+bc3+1能被(x一1)2整除,求a和b的值; ·193· ==========第199页========== (4)已知多项式x4+48+6ax2+4bx+c能被3+3x2+9x+3整除,求a,b和c的值. 4.(1)已知n次(n>2)多项式f(x)除以x一1的余数是-1,除以x+2的余数是2,求f()除以(x一1)(然+2)的余式.[提示:令f(x)=(x-1)(x+2)Q(x)+x+q.] (②)求证:当a+b时,多项式f(x)除以(x一a)(x一b),所得的余 、《 式是fKa)-fb)r+af(b)-bf(@2 a-b a-6 5.(1)求证:如果使多项式 f(c)=aoxn+a1xn-1+…十an 的值等于零的x的值多于n个,那末()必恒等于零; (2)求证 (o-b)m-c)+(x-c)x-a2+x-a(x-b)=1(a-8)(a-c)(b-c)(6-a)(c-a)(c-B)是一个恒等式. 6.(1)已知方程x3+9x2++21=0的三个根成等差数列,求k的值,并解这个方程; (2)已知方程x3一7x+k=0的一根为另一根的2倍,求k的值,并解这个方程; (3)已知方程x4-13x3+56x2+kx+48=0的两个根的比是2:3,余两个根的差是1,求k的值,并解这个方程. 7.(1)求证:如果方程aw2+bx十c=0的两个根互为倒数,这个方程可变形成2+px+1=0的形式; (2)已知方程2ac4-13x8+26x2-21c+4=0有两个根互为倒数,解这个方程. [提示:(2)可设 2x4-133+26.x2-21x+4=(x2+px+1)(2x+92+4), 确定”,.9的值后,再解两个降次方程.] 8.已知方程x3-2x2+5x-3=0的三个根是a,B,Y,求: (1)a2+B2+Y2; (2)a3+B8+y3. 9.已知方程x8-2x2+3x一1=0的三个根是a,B,Y,求作一个三次方程,使它的三个根是: ·194· ==========第200页========== 生 (1)22 a’7 (2)&,E,Y 7’2ya’a 10.(1)已知方程x4+4x3+62+4r+5=0有一个根是一i,解这个方程; (2)已知方程x3一(1一)x2+(1-)x+i=0有-一个根是-i,解这个方程. 11.(1)已知方程6ac4-13x8-35x2-x+3=0有一个根是2-√3,解这个方程; (2)已知方程3-(4+√3)x2+(5+4V3)x一5W3=0有一个根是√3,解这个方程. 12.已知2+3,√2和-3都是方程f(x)=0的根,(①)如果只要求这个方程的次数最低,这个方程是什么? (②)如果既要求次数最低,且要求各项系数是实数,这个方程是什么? (3)如果既要求次数最低,且要求各项系数是有理数,这个方程是什么? 13.求下列方程的有理根: (1)4ax3-4x2-25x+25=0; (2)6ac4+19x3-7x2-26+12=0; (3)24x5+10x4-8-19x2-5x+6=0; (4)10x4-138+15x2-18c-24-0. 14.已知f(x)=x6+2-4x4-5x8-6x2, (1)在有理数范围内分解f()的因式;(②)在复数范围内分解f(c)的因式. 15.在方程x3+px2+qc+r=0中, (1)已知一根为另一根的相反数,求证pq+?=0; ¥ (2)已知三根成等差数列,求证2p8-9pq+27rx0, 16.解下列方程: (1)8-7x3-8=0; (2)(x4+1)2-15(x4+1)+50=0; (3)x5-7x4+x8-x2+7x-1=0; (4)x6-(2+3)c-2(1-2)=0. 。185· ==========第201页========== 丛 第六章二阶和三阶行列式 《 行列式是代数里重要基础知识之一,它是研究方程组的重要工具。读者在今后自学解析几何的时侯,也要用到关于行列式的知识 本章将联系一次方程组的研究,学习关于行列式的一些初步知识. §6·1二阶行列式与二元一次方程组 在本丛书代数第二册里,曾经讲过:任何一个二元一次方程组,经过变形以后,都可以化成下面的一般形式: (I) a1+b1y=C1, (1) a2w+b则=c2. (2) 当a1b2一b1≠0时,方程组有唯一的解: 化、C162-C261 (HI) 1b-aeb1’ =@1C2-a2C1 abg-aeb1· 这组解可以作为公式来应用.为了便于记忆,现在引进 一个新的符号来表示它. 把四个数a,b,c,d排成 b d 用它来表示左上角和右下角的两个数的积ad减去左下角和 ·188· ==========第202页========== 3 右上角的两个数的积bc的差,就是 =ad-bc, d a c 式子b d称为二阶行列式.它含有两个行(横排称作行)和两个列(竖排称作列).a,b,c,d这四个数,称为这个行列式的元素,ad-bc称为这个二阶行列式的展开式①例1.展开下面的行列式: (1) b \ax (2) (3) asGa a1 C解](1) bi=ab2-a:61. Aab2 b3 (2) =c1b2-C2b1. C b2 a C1 (3) =a1C2-a2C1. a3 C2 从例1求出的结果可以看出,应用二阶行列式,方程组 (I)的解就可以写成 Gb1 心= bal as agb2 1b中0。 alC1 42 C2 a1az b2 ①在高等代数里,也直接把ad-bc称作是a,b,c,d这四个元素所组成 的二阶行列式。本书中为了避免混淆,把“行列式”与“行列式的展开式”这两词 区分了开来。 、197· ==========第203页========== 为了简便起见,用D,D,D,分别表示例1中(1),(2), (3)的行列式,于是方程组(I)的解就可以写成 =D’ (D≠0). Dy 仔细考察一下,行列式D,D,D,里的元素,与方程组(I) 里:,y的系数以及常数项间的关系,可以发现一个重要的规律: 1.把方程(1),(2)里心和则的系数顺次写成 ar bias ba 就得到行列式D;这个行列式称为方程组()的系数行列式. 2.把方程(1)和(2)里等号右边的常数项cG和cg,顺次代替行列式D里心的系数1和2,就得到行列式Dx;代替行列式D里y的系数b1和b,就得到行列式D. 应用这个规律,就可以把二元一次方程组的解直接写出.例2.解方程组 2ac-y-11=0, x+3y-18=0. [解]原方程组就是 2c-y=11, 心+3y=18. 2-1 今 D- =6+1=7, 1 3 11-1 D:- =33+18=61,. 183 1980 ==========第204页========== 211 D- =36-11=25 1.18 所以,所求的解是 D=1 D772 25 y=D 4 73 注意1.应用上面的规律时,首先要把方程组写成一般形式(T), 特别要注意常数项要放在等号右边 2.因为方程组(I)解的公式中要求D+0,所以解题时宜于先依次 求出D,Dx,Du,再代公式 习题61(1) 1.计算下列行列式的值: 1| 2 (1) 13-5-7 (2) 5 1 6一 sina -cosa a-b a2-a6+82 (3) (4) cosa sina atb a2+ab+82 2.证明: by 1 b虹 b1a1 (1) (2) 三一 a2 a2 02 b1 b1 kara bi (3) bi =0; (4) = b2 kaz a1十61 1 a bi (5) a2+ag0 a2 2 a 从上面证明的结果,你发现二阶行列式有哪些重要性质? [解法举例:(1)左边=a1b2一b12,右边=ab2-a2b1,∴.左边=右边 这说明:二阶行列式中,如把行改为列、列改为行,而不改变它们原 199· ==========第205页========== 来的次序,行列式的值不变.] 3.利用二阶行列式解下列关于、y的方程组: 2ac+3y+4=0, (1) (2) 4(x+2)=1-5y, 5x+6y+7=0: 3(y+2)=3-2; 3x+4y=7a-b, aa+b则=a2+b2, (3) (4) 4x+3y=7a+b; (ba+ay=2ab (a!#6). 上面研究的二元一次方程组是系数行列式D不等于琴 的.现在进一步来研究,当系数行列式D=0时,方程组 (1) () a1+b1y=C1, a2x+b2则=c9 (2) 的解的情况 首先,考察a1,a2,b1,b这四个数中至少有一个不是苓的情况.不失去一般性,不妨设1≠0,这时从 (as D =a162-b1ag=0 可以推得 ba42 b1. a1 命 d2-k, as 就有a2=ka1(:为常数),那末 b2=kb1. 这样,程组()就可以变形成: a1心+b1y=C1, kaix+kb1y=ca. 由此可知, 当cg=kC时,方程组(I)有无穷多组解当cg≠C1时,方程组(I)没有解. ·200· ==========第206页========== 因为,当cg=C时, D,= Bi Cy C1 g kctkb1 C16二0, ar a C1 1 C1 =k Az ha 0, ci 1 所以,上面的结论可以说成: 对方程组(I),当D=D=D,=0时有无穷多组解;当 D=O,且D和D,中至少有一个不等于零时没有解 其次,再来看1,ag,b1,b2这四价数都是零的情况、这 时,方程组(I)就是 0w+0y=c, 0心+0y=cg. 很明显,当1=c2=0时,它有无穷多组解;当c1和cg中至少有一个不是零时,它没有解. 注意在这种情况下,不管c1和c2是否等于零,D,和D,都等于零.所以,仅仅知道D=D=Dy=0而没有1,a2,b1,b,这四个 数中至少有一数不是零的条件,不能贸然判定方程组(I)有无穷多组 解 例3.解方程组 ∫(2k-1)x-(+1)y=3k,(46-1)x-(3k+1)y=6k+4, 【解】D=2k-1-(k+1) 46-1-(3k+1) =-(2k-1)(3+1)+(k+1)(4k-1)=-6k2+k+1+42+3k-1 =-2k(k-2). ·201· ==========第207页========== 3k D -(k+1) 5k+4-(3k+1) =ー3k(3+1)+(k+1)(5k+4)ー(462-6k—4) =-2(2k+1)(G-2). 一人 2h-13k D,-46-15k+4 =(2k-1)(5k+4)-3k(4-1)=-2k2+6k-4 =-2(k-1)(k-2). 如果D≠0,也就是≠0,≠2.这时,方程组有唯一的 解: 化= D=-2(2k+1)(k-2)-2k+1 -2k(k-2) ==-2(后-1)(肠-2)=飞-1 D -2k(k-2) 如果飞=0.原方程组就是 「--y=0, -x-y=4. 显然,这个方程组无解 如果飞=2.原方程组就是 3x-3y=6, L7c-7y=14. 它可以化成 c-y=2, x-y=2. 因此,方程组有无穷多组解. ◆202· ==========第208页========== 牛 注当D=0时,如果已知方程组里未知数的4个系数中至少有1 d 个不是零,那末也可以直接从Dz和D,是否都等于0而确定方程组有无穷多组解或者无解。例如,本题中,当=0或2时,未知数的系数都不是0;而当飞=0时,D+0,所以这时方程组无解;当k=2时, D=D,=O,所以这时方程组有无穷多组解. 习题61(2) 1,不通过解出方程组,而直接判定下列方程组里哪些有唯一的解?哪些无解?哪些有无穷多组解? x+2y=3, x+2y=1, (1) (2) 2x一y=4; 2ax+4y=2; x-2y=1, (3) x-2y=1, (4) 2x-4y=3; 人y-号1-0. 2.解下列关于x、y的方程组,并进行讨论: x+y=k+1, (1)l a+ky =2k; (k-1)c+(+1)y=2(k2-1),(ん2-1)+(2+1)y=2(8-1). §6·2三阶行列式 从§61可以看出,用二阶行列式可以简便地写出二元一次方程组的解.类似地,可以应用三阶行列式来简便地写出 三元一次方程组 戈 a1c+b1y+c12=k1, (I) a2心十b则十c2然=k2,3十b则十c3x=g 的解.本节将先来说明一下三阶行列式的意义. 把九个数a1,a,3,b1,b,b3,C,Cg,C排成下面的形 ·203· ==========第209页========== 式: ab1c A2 b2g (1) asba C3 并且用它来表示式子① (aibaC3+a263C1+a3b1C2)-(a3b2C1+a261C3+a1b3c2)(2)式(1)称为三阶行列式,式(②)称为这个三阶行列式的展开式. 要写出三阶行列式(1)的展开式(2),只需在原行列式第 三列的旁边顺次把第一列、第二列的元素写出,添成5列,如下图所示: 2 3 03 bs 然后,把每一条实线经过的三个元素的积的和,减去每一条虚线经过的三个元素的积的和。这种写出三阶行列式的展开式的方法,称为对角线法 例1.计算行列式 11 2 3 D4 5 6 7 8 9 的值. ①象二元一次方程组一样,式(1)事实上就是方程组(I)的系数行列式;而 式(2)的值不等于零就是方程组(I)有唯一解的条件。读者有兴趣的话,可以用 普通的方法解方程组(T)来验证, ●2D4· ==========第210页========== 平 【解]D=(1×5×9+2×6×7+3×4×8) -(7×5×3+8×6×1+9×2×4)=(45+84+96)-(105+48+72)=225-225=0. 注行列式的展开式中,每一个项的元素是由不在同一行又不在同 一列的元素构成的.例如,在项☑b中a,b2,cg分别是第一行第一列,第二行第二列,第三行第三列的元素,它们不处在相同的行或相同的列。写出展开式时应该注意这个特点。 清 习题62(1) 1.计算下列行列式的值: l 2 31 -3 4 (1) 3 1 2 (2) 2 -5 2 3 1 3 -1 2.展开下列行列式: a b人 0 x (1) b C (2) 0 3 b 0 3.证明 a1biCs \b2C2 bC1 b1 a2 b2 C2=W1 c.0% as a as bs9 c 4.'证明: ab1 C1 a2 (1) 0%C2 b102be; asDs C1Ca Cs (b1 a 01 (2)oa2 2b2 ag Co bs ◆205· ==========第211页========== ¥ a1 C1 (3)a2a2C2=0; a3QgCa ar b1 kci a1b1 (4) a2b2hc2二水9D2C2 agDakcs asDa 1b1+ C1 1bi 1bl (5)a262+b2 a2bC2 a2bC2 ag08+b3 s ba [提示:只需把行列式展开,证明等号两边的式子恒等.] 根据习题6.2(1)第4题证得的结果,可以看出:三阶行列式象二阶行列式一样,具有下面这些重要性质: 1°把行列式的行改为列,列改为行,而不改变它们原来的次序,则行列式的值不变; 2°把行列式的相邻两列对调,则行列式的绝对值不变而符号相反; 3°如果行列式中两列元素完全相同,那末这个行列式的值等于零; 4°把行列式的某一列的元素同乘以飞,相当于这个行列式乘以; 5°如果行列式的一列的元素都是二项式,那末这个行列式等于把这些二项式各取一项作成相应列而其余各列不变的两个行列式的和 根据上面提出的性质1°,容易看出:上面2°、3°、4°、5°中所指的关于列的性质,对于行也同样成立. 应用上述性质,有时可使行列式的计算比较简便。例2.计算: ·206· ==========第212页========== ·L0z· eq Ep 0 lT9刚m唑=0 0 q ED 职半·8卧 I- ×8×× I- 乙 8- 工 (亿) 8 - 03五=(T+I+I+I+T+I-)×0zI= I- I- ×五X8×9×Z= I- 五 8 8 9×る=v(T) [趣] 8 9I98 8口 I- 98z =&(亿) 不8 I- 五 8 9忍8 (T) 木 ==========第213页========== a G 【证] D=龙0)b1 C1 (性质4) as baC3 =kx0 (性质3) =0, 从例3可以看出,由行列式的性质4°和3°,可以推出行列式还具有下面的性质: 6°如果行列式的两行(或两列)的对应元素成比例,那末这个行列式的值等于零. 习题62(2) 1.利用行列式的性质5°和6°,验证行列式还具有下面的性质7°:把行列式的某一列(或行)的所有元素乘以同一个数后,加到另一列(或行)的对应元素上,则行列式的值不变 2.利用行列式的性质,计算下述各行列式的值: 2312 6 4227 6 (1) 8 -28 36 (2) 12 1 3 20 3590 1632是 3.不通过展开行列式,证明下列各式的值都等于零: 110 8 2 25 -12 (1) 15 3 12; (2 13 35 2 21 417 323 143 [提示:(1)可应用性质7°和6;(2)可应用性质1°和2°.]4,计算下列行列式的值: 65 5 -12 13 11 (1) 5 6 5 (2) 131213 5 5 6 -111112 ·208· ==========第214页========== §6·3子行列式和代数余子式 为了简化行列式的计算,也为了下面研究三元一次方程组的需要,本节将引进关于行列式的两个重要概念一子行列式和代数余子式. 在习题6•2(1)的第3题里,读者曾证明过下面的恒等式: a1 C1 b2Cg bG1 bC1 agbsCa=1+a3 (1) ba -02 C3 Ca as baC3 这里可以看到:一个三阶行列式,可以其第一列的元素分别乘以一个相应的二阶行列式,用所得的积的代数和来表示.仔细考察一下这些二阶行列式与三阶行列式间的关系,还可以看出:a1,a,a所乘上的二阶行列式,正好是三阶行列式里把这个元素所在的行和列的元素全部划去以后所留下来的四个元素(不改变它们的相互位置)所组成的,如下图所示: i… 1…b1C1- C1 1 01 C1 A2 b2 C2 Ca Qa b3 b3 b3-…3 把行列式里某一元素所在的行和列划去后留下来的行列式,称为这个行列式对应于这个元素的子行列式(简称子式).如果用。和分别表示某一元素所在的行数和列数,这个元素的子式乘以(一1)+所得的式子,称为这个行列式里对应于这个元素的代数余子式。 ·209· ==========第215页========== 代数余子式,通常用这个元素的大写字母附以相同的下标来表示.例如,在行列式 a C1 D-a2 b2eo as b3c3 里,对应于元素a1,a2,ag的代数余子式分别是: b2C2 b2C2 A1=(-1)1+1bs G 1b1 A2=(-1)2+1D3Ca b3 Ca b 1b1 A3=(-1)3+1ba bC2 应用代数余子式的记号,式(1)就可以简单地表示成 D-a1A1+a2A2+a3A3. (1) [这里D表示式(1)等号右边的三阶行列式.] 读者可自行验证:三阶行列式还可以表示成以下各种不同的形式: D=6:B1+62B2+03Be, (2) D=C101+C202+CO3, (3) D=a1A1+61B1+CC1, (4) D=a2A2十b2B2+c2Cg, (5) D=a3A3十b3B3+cgC3 (6) 上面的等式(1')、(2)、(3),分别称作是把行列式D按照 它的第1、2、3列元素展开,等式(4)、(6)、(6)分别称作是把 行列式D按照它的第1、2、3行元素展开. 把上面这6个等式概括起来说,就是: 定理1.行列式的值,等于任意一行(或一列)的各元素 ·210· ==========第216页========== 乘以对应于它们的代数余子式的积的和 应用这个定理,有时可简化行列式的计算。例计算下列行列式的值: 1308 3 08이 (1) 5217; (2) 0 -11 7026 4 60이 、[解](1)按第二列元素展开,得 38 原式=2 =2(76-56)=38, 725 (②)先把第3列元素加到第2列的对应元素上,这样,第2行就有两个元素是零,再按这一行元素展开,得 386 138 原式=001 =-(15-32)=17. 45 450 对于行列式的代数余子式,还有下面所述的性质:定理2.行列式某一列(或行)的各元素与另一列(或行)对应元素的代数余子式的积之和恒等于零. 这也就是说,对于行列式 C1 D-Ag b2 Ca,as b3 C3 以下的等式都成立: aB1+a2B2+a3B3=0; @1A2+61B2+C1O2=0; a1C1+aεC2+agC3=0; a1A3+b1B3+c1C3=0 b1A1+b2A2+b3A3=0; agA1+bsB1十cC1=0 6:O1+b202+0sOs-0; @2A3+62B3+0:O3=0; C1A1+C2A2+C343=0; agA1十b3B1+cgC1=0 c1B1+c2B2十cgB3=0 a3A2+b3B2+CgO2=O. ●211· ==========第217页========== 这些等式的证法完全一样。例如,下面来证明其中第一个等式 [证]在行列式D中,把第二列的元素b1,b2,b3换成第 一列的元素a1,a2,%,得行列式 a1 C1 D'=N2 Cg a3a3 Cs 把D'按第二列的元素展开,因为这列元素的代数余子式 就是行列式D中第二列的各元素b1,b2,b的代数余子式 B1,B2,B3,所以,根据定理1,有 D'=a1B1+a2B2+agB3. (7) 今D'中有两列元素完全相同,根据行列式性质3, D'=0. (8) 由式(7)和式(8),即得 a1Bi+a2B3+a3B3=0. 注对这些等式,也可以把各代数余子式展开后代入左边,再进行化简而得到证明,但不如上面的证法简单 习题63 1.在行列式 C1 D=a2 b2 C2 as be ca 中,元素a4,b,C的代数余子式分别是A,B,C(i=1,2,3),求证: (1)a2A1+b2B1+c2C1=0; (2)G1A1+c2A2+cgAg=0. 2.利用代数余子式,计算下列各行列式的值: ·212- ==========第218页========== 牛 7 0 -3 81 12 13 (1) 0 02 (2) 13 0 45 y 12 5 36 7 3.先应用行列式的性质,把某一列(或行)的元素化成有两个为0,再应用代数余子式来进行计算。用此方法,试计算下列各行列式的值: -1 2 2 3 4 2 (1) 1 2 (2) 7 -5 2 2 一1 3 2-4 4,计算下列各行列式的值: 15 25 5 1216 12 (1) 25 15 5 (2) 1 0 5-2515 -3-4 112 §64三元一次方程组 有了上面的基础,现在就可以利用三阶行列式来研究三元一次方程组 a1+b1y+C12=k, (1) (I) @x+629+C2z=12, (2) a3x+b3则十ca名=k3 (3) 的解法. 首先,应用代数余子式和它的性质,设法从这三个方程里消去y和名,导出一个只含有未知数的方程 行列式 D- D2 Ca D3 ·213。 ==========第219页========== 中,分别记元素au,b,c的代数余子式为 A,B,C4(元=1,2,3) 把A1,A2,A3分别乘式(1),(2),(3)的两边,得 a1A+61Ay+CA1z=k1A1, (4④ a2A+6Ay+CA:z=k2A2, (5) a3A30463A3y+-C3A3z-%3A3. (6) 把式(4),(5),(6)的等号两边分别相加,并应用上节的定理1和2,得 Do=%1A1+%2Ag+k34.3. (7) 类似地,从式(1),(2),(3)中消去心和,或者心和y,可以导出只含有未知数则,或者公的方程: Dy=k1B1+k2B2十kBg, (8) Dz=k1C1+kC2十kC3. (9) 为了简便起见,分别用记号Dx,D,D来表示(7),(⑧),(⑨)中等号右边的式子,就是 11b1 D=1A1十k2A2+3Ag= ,bgCa, IceDa C3 1 7ox C1 D,=k1B1+kB2十gB3=a2ka Cg a3 3 Cs a 61 无1 D=%101+kaC2+%63Cs=a2 D212 asba 这就导出了一个方程组 Da-Da, (II) Dy=Dy Da=Da. ·214 ==========第220页========== 工 下面将分成三种情况来研究. 情况一D≠O.这时,方程组(II)有唯一的解 = y- D. 我们来看一下,这组解是否是方程组(①)的解.为此,通过检验①,例如,把这组解代入式(1)的左边,得a号+b品+费 D(aD+bD+GD,) 공a(+k4+h4)+ (a2+B,+bB) +c1(%101+%C2+%gCg)] (+6В+)+(+B+6C +k3(aA3+61B3+C103)] =方D=k, 所以,这组解适合方程(①);类似地,读者可以自己验证这组解也适合方程(2)和(3).所以,这组解也就是方程组(①)的解.情况二D=0,但D,Dy,D。中至少有一个不等于零 这时,方程组(II)无解,方程组(I)当然也无解 ①因为,从方程组()导出方程组(I)时,没有附加A中0,B,中0,C+0 的条件,只能肯定方程组(T)的解都是方程组(II)的解,但不能保证方程组(II) 的解都是方程组(I)的解,所以还必须通过检验这个步骤, ◆215· ==========第221页========== 情况三D=Dx=D,=D。=O.这时,方程组(I)有无穷 多组解,但方程组(I)可能有无穷多解,也可能无解①. 例如,在方程组 x-y+2=2, x-y+22=2, 2x-2则+4x=3, 和 2x-2则+4=4, 3x-3y+6x=6 3x-3y十6=6 里,都有D=Dz=D,=D=0,但是,前面这个程组无解,而后面这个方程组有无穷多解. 例1.解方程组 w+2y-4z=11, 2c-3y =2, y-4名=1, 1 2 -4 【解] D 2 -3 0=12-8+16=20. 0 1 -4 11 2 ー4 De- 2 -3 0=132-8-12+16=128, 1 -4 11 -4 Dy-2 2 0=-8-8+88=72, 0 1 -4 1 2 11 D-2 -3 2=-3+22-4-2=13, 0 1 1 所以,所求的解是 ①这种情况的详细研究比较复杂,本书中把它略去了。 ·216· ==========第222页========== 1282D 20 =6 D 72 y=D 20 33 D 13 D 20 习题64(1) 1.应用三阶行列式,解下列方程组: 2ac-3y+2+1=0, x+2y+32=6, (1)8+y十8=6, (2) 2m十4y-7=名, 3ac+y-2z=-1; 3+2y+9z-14=0; 2x+y=5, 0一y十2=4, (3)x-3z=-14, (4)x+y一2=b, 5y-8=10; -c+y十2=C, (a,b,c是已知常数.) 2.应用三阶行列式,确定下列方程组的解的情况: x+y-名=5, 2c十y-2=1, (1)3x+y+a=2, (2) 3x+y+28=-3, 5x+5y-5a-=25; 10x+4y+22=4; 2x-y+3z=2, 2-y+3z=1, (3)4-2y+62=6, (4) 4c-2+6a=3, 6x-3y+9z=6; 7x-y+名=0; x-y一2=0, -y+2=1, (5) 2x+y-32=3, (6) 2x-2y+2%=2, 3x-42=3; -3x+31-32=-3. 最后,再来研究一类特殊的三元一次方程组 a1X+619+Ciz-O, (I) a2c十b则+co2=0, a3t+b3y十c32=0. ●217◆ ==========第223页========== 在这个方程组里,各个方程的常数项都等于零,含有未知数的项都是次项.这个方程组通常称为三元齐次线性方程组.很明显,三元齐次线性方程组都有一组解 x=0, y=0, 公=0. 这组解称为三元齐次线性方程组的零解. 现在的问题是三元齐次线性方程组是不是还可能有其他不是零的解(即非零解)? 因为方程组(III)就是方程组(①)当k1=k2=k?=0时的一个特例,所以上面得到的结论应该成立.这样,就可以知道: 当系数行列式D≠0时,方程组(III)有唯一的解,这组解 就是心=则=公=0. 当系数行列式D=0时,方程组(II)或是无解,或是有无 穷多解:但是,既已知方程组(III)有一组解=则=公=0,所 以这时方程组(III)有无穷多组解. 由此,可以得到结论: 定理1,三元齐次线性方程组有非零解的充要条件是它 的系数行列式D=0 例2.已知方程组(II)有非零解,求证 x=kA1, y=%B1, (飞是任意常数) 名=C1 是这个方程组的解 【证】因为方程组(II)有非零解,所以D=0.因此有 a1(kA1)+b1(kB1)+c(C1)=k(a1A1+b1B1+c1C1) =kD=0, 0218· ==========第224页========== 上 a(kA3)+6(kB)+c2(kC)=k(asAi+62B1+CC1)=0,a(kA1)+b3(kB1)+c3(C1)=k(a3A1+b3B1十cC)=0,所以G=A1,y=kB1,名=C1(飞是任意常数)是方程组(III)的解。 注同样,可以证明: 鸵=KA2, 优=e”A8, y=kB2, 和 y=k"Bs, 8=k'C2 a=k"Cg 也是方程组(I11)的解.在A,B,C(i=1,2,3)不全为零时,方程组 (I工)的非零解就可以用上面的一组解一般地表示出来. 例3,解下列方程组: 4c-3则+52=0, -g+=0, (1) 3c+y-2%=0, (2) x十y-公=0, G-4划-3%=0 5x+y-%=0. 烁] 4-3 5 (1).·D=3 1-2 1-4-3 1-2 -3 4 3 .-4 -3 -4 + -3 =-44-87+1 =一130≠0, 所以,原方程组有唯一的一组解: G=0,y=0,=0 11.-1 110 11 (2).·D=11-1=10 -1=0 51-1 0-1 ·219· ==========第225页========== 1 A1-1 {1 1 Ci- ー4 5 1 所以,这个方程组有无穷多组解: x=0, y=一4E, (这里飞为任意常数), 2=ー4k 注本题的解的过程中,如果计算代数余子式A,B2,C,可得 A2=0,B2=-6,C2=-6. 由此得方程组的解是 〔就=0, y=-6k, 名=一6 很明显,这与上面所求得的解事实上是一样的、如果在前面这组解里命-4=m,后面这组解里命-6=,那末它们都变成了 「x=0, y=m, (m为任意常数) 之=m 的形式,因此,便可把前后任一组解作为方程组的解的一般形式. 习题64(2) 1.解下列各方程组: 「x+y+2=0, [x+5y-102=0, (1) 2+3则÷22=0, (2) 2x-3y+6z=0, 4x+5yーz=0; 3x+2y-4z=0; 「x-2y+名=0, [6x-y-78=0, (3) x+2y-z=0, (4)5x+10y+53=0, 5x+2y-2=0; 4x-3y-7z=0. ◆220· … ==========第226页========== 2.证明下面这些方程组都没有非零解: [+y+22=0 -x+2y+22=0, (1) 2x+y+名=0 (2) 2x-y+22=0, x+2y+2=0; 2x+2y-z=0; 「2a+z=0, 4x-2y+4a=0, f (3) x-4划=名=0, (4)10x+2y+128=0, -x+8y+32x0; x+2y+2=0, 本章提要 1.二阶行列式与三阶行列式的展开法则 a =a1b2-ab1. a01M 42b2 C2=(a1b2C3+a208C1+agbic2)-(ag62C1+a261Cg+ai6gc2) aabsCs 2.行列式的性质 (1)把行列式的行与列对调后,不改变它的值. (2)把行列式的相邻两列(或两行)对调后,它的绝对值不变而符号 2 相反. (③)如果行列式中有两列(或两行)的元素依次相同,那末这行列式的值等于零, (4)如果用数乘行列式的某一列(或某一行)中的所有元素,所得行列式的值即等于原行列式的值乘以k, (5)含有两个成比例的列(或行)的行列式,其值等于零. (6)如果行列式的某一列(或某一行)可以表为两项的和的形式,那末这行列式可变形为两个行列式的和 (7)如果在行列式中把某一列(或行)的元素加上另一列(或行)的对应元素的倍,那末此行列式的值不改变 (⑧)行列式的值,等于它的任一列(或行)中所有的元素和它对应的代数余子式的乘积的和。 ·221· ==========第227页========== (9)行列式的某一列(或行)的各元素与另一列(或行)对应元素的代数余子式的积的和恒等于零. 3.一次方程组的解 (1)二元一次方程组 aix+b1y=c1, 是a2t+b2y=C2. D= - a1 设 a2 1°如果D≠0,方程组有唯一组解: D 2°如果D=0,而未知数的系数不全为琴,那末: 当D≠0(Dy≠O)时,方程组无解; 当D。=D,=0时,方程组有无穷多组解. (2)三元一次方程组 aix+biy+cz=k1, agx+b2y+C28=ka, agc十bgy+C%&=kg. y01 C1 ki b1 设 D=ag62C2, Dc= b2/ dabsCa gba d41k C1 a b1k D.=da C2, D.=gb2 ds ag b8有 1°如果D≠0,方程组有唯一组解: D 2°如果D=0,而D,Dg,D,至少有一个不等于零,方程组无解.3°如果D-0,且D。=Dv=D,=0时,方程组有无穷多组解,或无解。 (3)三元齐次线性方程组 ◆222· 乙 ==========第228页========== 1 41x+b1y+G12=0, a2x+b2g+c28=0, agx+b3y+c8=0 ar bi1 设 D=a2 62C2 asb3 Cs 1°如果D+0,方程组有唯一组解: =0,y=0,=0(称为零解). 2°如果D=0,方程组有无穷多组解: g=k44, y=kB, a=kCs, 这里飞为任意常数,44,B,C不全为零(i=1,2,3)。' 复习题六 1.解下列方程组: 3x+5y=13, 3x+4划-2=0, (1) (2) 2c+7y=81; 2c+3y-7=0; x+2y+32=6, 2x+8=5, (3) 2+4y+名=7, (4)G+3y=10, 3x+2y+9z=14; 5y-z=10. 2,不通过解方程组的方法,试确定下列方程组是无解还是有无穷多组解: 3w+5y=4, 3x-5y=4, (1) (2) 6c+10y=2; 9xc-15y=12; 3x+4划=1, 3x+49=1, (3) (4) 6x+8y=3; 6ac+8U=2; 2c-3y+z=-2, 2x+4y-22=8, (5)x-2y+3z=-1, (6)3c-y+2g=-1 x-y-22=3; 11x+y+4z=0; ·223· ==========第229页========== í3x-6y+3z= 3 Tx+2y-2=-3, (7)2-y+3z=3, (8) x-2y+2=1 8x+y+72=3; -2x+4y-28=-1. 3.计算下列行列式: 2 0 1 又 一2 4 (1) 1 -4-1; (2)10 212 -1 8-3 1 22 14 23 a (3) 3 42; (4) b 2 3 4 a b c 1 a6+c 11 1 (5) 1 bc+a (6) a b 1 a+b a2 飞2c2 4.计算下列行列式: la+b 2 b 1a+2ba+4)a+6b (1) a c+a c (2)a+3ba+5b a+7b b c+b1 a+46a+6ba+8b 8+c aーC c%-b| (3)6-c b-a c-bC一ua+b 5.证明下列各式: 1 a &2-bc (1) 1 b b2-ca=0 1 c c2-a6 6+c c+a a+b Ja b (2) b'+c" c'+a'a'+b'=2d6' c b”+c” c”+a”a”+b db” a a-b a+6+c (3)2a3a+2b 4a+3b-2c=a8+3a2b; 3a6a+3b10a+9b+3c ·224· ==========第230页========== 1 1 1 (4) axc ay ae =a(x-y)(yーz)(a-). a2-+x22+2y2a2+z26,解下列方程中的x: x2-12 31 2x 이 (1) 2 3 2=0; (2) 3 510=0; ↓1 1 1 13 8 |15-2x1110 (3)11-3x1716=0. 7-x1413 7.解下列方程组: c-3y+42=0, 「10x+8y+22=0, (1)2x-52=0, (2) 15c+3y+122=0, 3x-y+7z=0; 21+4y+172=0; + 3x+3y+4之=0, 23 y一2=0, 1 1 (3) 2c+5y+4a=0, (4) +之g+a=0, 4 2x+3y+7z=0; 1 (5y+2a=0 [x+y+(b+c)z=0, (5)x+by+(c+a)2=0, (a,b,c为已知常数); B+cy+(a+b)8=0 g+22=0, (6){y+22=0, 是”一y=0 8.飞为何值时,下列方程组有非零解: 4x+3y+名=x, 3x-4划+7z=ky, x+7y-6z=飞a, ·225· ==========第231页========== 、 总复习题 1.(1)怎样判断一个问题里所指的是相异元素的排列,还是组合? (2)在一条铁路线上,从起点站到终点站,共有10个大站.铁路局要为这些车站总共准备多少种客票?这些客票里票价不同的最多有几种? 2.(1)个不同元素中取”个不同元素的排列种数与组合种数间,存在怎样的关系? (2)在是什么值时有Am=C? (3)在什么时候有An=24C? (4)有没有这样的%,能使等式A%=10Cm成立?为什么? 3.求下列各式中的: (1)C+1=당3; (2)xP3=4C-2; *(3)C1:C5z-)=132:35提示应用0g=mm] m “4.(1)解方程组 C=C2型, 3C%+1=11Cg-1; (②)已知在m个不同元素中取%一1个、”个、”+1个不同元素所有组合的种数,顺次成比2:3:4,求m和n. 5.证明: (1)non=mCm; (2)A+1=A编+nAa1. 并且说明这两个等式的具体意义, 6.把a,b,c,d,e这五个字母排成一排,问 (I)a被排在第一个位置,或b被排在末一个位置的排法共有几种? *(2)在上面的排列中,α被排在第一个位置且b被排在末一个位置的概率是多少? ·226• ==========第232页========== 7.某厂生产一批五个数字的号码锁(每档数字都可以是0,1,2,…,9这十个数字的任一个), (1)问产品中总共可有多少不同的锁? *(②)在上面这批锁中取出一个,随便拨一个号码恰把锁打开的概率是多少” (3)在上面这批锁中,奇数号码的锁有几个?偶数号码的锁有几个?*(4)在上面这批锁中,要求五个数字都不相同且号码被25整除的锁出现的概率是多少? 8.有8部机器,分配给甲、乙、丙三个工人管理 (1)如果甲管4部、乙管3部、丙管1部,有几种分配方法?(②)如果甲管4部,其余的二人是一个管3部、一个管1部,有几种分配方法? 9.某青年突击队有8个男队员和7个女队员,现考虑从中选出6人,组成一个试点小组: (1)如果男女各占一半,有多少种选法? (2)如果至少有3个女队员,有多少种选法? (3)如果最多只能有3个女队员,有多少种选法? (4)如果至少有3个女队员并且至少有2个男队员,有多少种选法?[提示:对(②)、(3)、(4)小题,宜用加法原则来解,这样不易出错.] (5)如果要选出4个男队员和2个女队员,分别在试点小组里担任6种不同的工作,问有多少种选法? 10.(1)今有1元币、5元币、10元币各一张,可以组成多少种不 同的币值? (2)今有1元币5张、5元币2张、10元币1张,可以组成多少种 不同的币值? [提示:只须注意,能组成的最小、最大币值是多少?这中间的币值情况怎样.] (3)从1、2、3、4、5、6这六个数字中,任取两个相减,可以得到多 少个不同的差? [提示:注意一下,能组成的最小、最大的差是什么?这中间的差值的情况怎样] ·227◆ ==========第233页========== X 11.设%是自然数,用数学归纳法证明:(1)②n)1=21.35…(2m-1 n! (2)(13+23+33+…+n3)+3(15+25+35+…+n) =4(1+23十…十)3 12.在(+)”的展开式里,已知第六项的值是112、第七项的值 是7、第八项的值是全求心,,ym为然数,女y为实数. 13.求证: 四在(e-)的展开式里,2"的系数是 (-1)n-r(22)! (0-r)1(%+r)1 (2)在(1+x)p+9的展开式里,”的系数与x的系数相等; (3)在(1十x)”+1的展开式里,x*+1与x的系数的差,与(1+)”的展开式里,x+1与x-1的系数的差相等. 14.求证 C头+2C2+3C+…+物Ch=0.2n-1 15.已知 1=(x+y)+(x2-x-2y)i,2=(2c-y)-(y-y)这里心,y都是实数,试问x和y取什么值时, (1)1和2都是实数? (2)1和2都是纯虚数? (3)1和2互为共轭复数?(4)1和2互为共轭虚数?16,设x,y为实数,解方程: a)(e-20)-(2y-6xi)=(3x+2)-(号-2)月 (2)6x-yi 155+2i8x+3则i 17.解下列方程或方程组: (1){z-z=1+2; (2-)u+(2+)v=2, (2)(uU=2. ·228· ==========第234页========== 5 18.设w=-1+√3i,求证: 2 (1)(1+w-w2)8-(1-w+2)8=0: (2)(1-w+w2)(1-w2+w4)(1-w4+w8)…(1-w2n+wn)=22m (2是自然数), [提示:利用习题4.8(1)第4题的结论.] 10.(少已知f)-2x+b2+m+d的振是是、一2、3,术这个 多项式; (2)已知f(x)=x4+a3+c2+b+1能被x2-2c+1整除,求这个多项式; (3)已知f(x)是一个三次多项式,且 f(2)=f(3)=0,f(1)=6,f(4)=18, 求这个多项式 20.解下列方程: (1)x4+43-3ax2-10x+8=0; (2)4+2m3-17x2-18x+72=0; (3)2x5+5x4-13x3-13x2+5x+2=0; (4)x5-114+368-36x2+11x-1=0 21.(1)已知方程 x4一23-22x2+62x-15=0 有一个根是2+√③,求其余各根; (2)已知方程 x4+(V3+1)x8-(V3+1)x+(√3+1)-(2+V3)=0 1 有一个根是一(2+√3),求其余各根. *22,(1)已知方程c3+ax2+bx+c=0的三个根成等比数列,求证 a8c=b8; (2)已知方程x3-52+t+8=0有一个根是2-31,求证t+8=4; (3)已知方程4x一5x+k=0的两个根是直角三角形的两个锐角的正弦,求k的值. [提示:设一个根是sina,那末另一个根是sin(90°一a).] 23.解下列不等式: .229· ==========第235页========== 3-3(1) 2\z0: 3x (2) 2x <14.1 24.不展开行列式,证明 m61+nC1V1C1 mb2+nc2 b2 =0 mba+nceDaCs 25.已知w是方程x3-1=0的一个虚根,计算下行列式的值: 1 w2 1 w9 (1)u w2 1; (2)w2 1 lw21 いw? 1 11 w21 2 51 (3) 3 ¥ (4) w? 3 6 6 7 b tw8 4 26,.已知e=cos5w+isin (1)求证e是方程x-1x0的一个虚根; (2)求证方程x一1=0的5个根可以分别用,e2,e,e,e来表示; (3)求证e+e2+es+e+c=0 ·280· ==========第236页========== 习题答案 第一章 习题11 1.25,20;2.15. 习题12 1.(2)60,(3)120,(4)120;2,(1)16,12, (2)64,24,(3)256,24;3.(1)504,(2)729.习题1.3(1) 1.(1)120,(2)5040,(3)5,(4)3; 2.の)②提(3)4,()2 习题1.3(2) 1.(1)120,(2)360;2.60;3.720;4.20. 习题141.(1)1,(2)210;3.120.习题1·6(1) 1.325;2.(1)24,(2)96,(3)48: 3.(1)48,(2)672,(3)240,(4)480;4.30,15,15.习题15(2)1.(1)100,(2)36,(3)52,(4)48; 2.(1)48,(2)360. 习题16 1.2401;2.810000:3.900,648,9,243; 4.(1)60,(2)243. 习题171.(1)10,(2)60;3.(1)20,10,(2)10,20.习题18(1) 3.(1)8,(2)7:4.(1)28,(2)56; 5.(1)120, (2)210, 习题1.8(2) 1.(1)4,(②)4,(3)12,(4)16,(5)16; 2.(1)10,(2)26,(3)20,(4)164.116;5.(1)217, (2)494 习题1.91.(1)999,(2)198;2.(1)2,(2)4.习题1.101,(1)60,(2)360,(3)90;2.(1)60,(2)15, (3)15;3.(1)2520,(2)576,(3)216. ◆231· 美 ==========第237页========== 太 习题1.11(1) 1.(1)95 990,(2)97 ;2.(1) 17 990 25’(2)2 奥 3.(1).1 (2) 29 188 96 216, 2164.(1)295’ (2)295· 习题111(2) 1. の(2) 1 3 7 16’ (3)16’ (4)1 2.≈97.0%;3.(1)≈13.5%,(2)≈99,3%;4.(1) 32 243? (2)11 243· 复习题一 2.(1)4,(2)46,(3)56;3.(1)1630,(2)288,(3)21, (4)479; 4..(1)10080,(2)30240,(3)1152;5.(1)360, (2)3888;6.(1)64 (2) 48 125’125’ ()2 125· 第二章 习题21 1. (2) a =+2; 2. (1) a,=,(2)a=が+ 2n 习题23 2.(2)%=1,%≥10. 复习题二 3.%≥3. 7 第三章 习题31 3.(1)-560,(2)160. 习题33(1)1.(1)T4=220abV万,Tg=495a44, ②108680rw,1s09s0,g.)Ca2,2),70 习题3.3(2).1.(1)Cx18,(2)Cxa1;2.(1)T7=C96=5005,(②没有;·3.252;4.%=8. 习题3.41.(1)Coa65,…(2).C,,C1x;4.Ci6a68. ·232。 ==========第238页========== 正 习题35 2.(1)90/2,(2)-C1ox2VE;·3.10x6,-10x9. 习题3.6 1.127;2.511. 习题3.71.1.12;2.0.984:3.0.99;4.1.009;5.1.03;6,0.9964;7.731.921;8.1.030;9.3128;10.255.5. 复习题三 4.3003;5.-84;6,(1)1+2c-262,(2)135a,(3)-168, (4)-51;8。(1)2",(2)2mm+12;10.1;11.(1)1,26, (2)31.84, 第四章 习题42(1) 1.1,,-i,32.(1)-1+,·(2)-1 (3)为奇数时:乞,偶数时:1. 习题42(2)2.-V+5②)-Y,)3,④1- 2 4. (8)0, (4) (2v2 -10); 6. (1)ま,(2) (3)±2,±2,(4)士2,±V√5. 习题484.(=手=至,()=4,g=3,(8)=-8,4 y=3或x=3,=-8,(4)x=1,y=3或x=-1,y=-3; 5.(1)m=-1,m=6,(2)m=4,(3)m=-1. 习题44(1)3.(2)2,(4)是;4.以原点为中心,1和2为半径的两个同心圆所构成的圆环(边界在内). 习题44(2)1.(1)0,②),(3)受,(④受;2.(等, 3 ②3,(8号,国青a;3.21220,②6723, 、 习题44(3)1.(1)5(cos3652+isin36°52'), (2)13(c0s247°23'+isin24723),(3)6(cos330°+iin330),. (4)5V√/2(cos135°+isin135),(5)5(cosπ+isinπ), (⑥13(os0+isim0,()6(os受+iim), 233· ==========第239页========== 以 图4如是+in号r片.2+2V362)-4,③)-线(④)2W3-2;3.r[cos(-6)+isin(-)]或 s(2-9)+:sn(2-9)]: 4. () 3,, (2)3,용, (3)√3,15,(④1,3元.4 习振46)1.山名-名, (2)-2√2i, (3)2b+2ai, (④g-)+5(g-x),(⑤252.①)-V3+1+V3-1, 2 2 (2)2+2W√3i;3.(1)x=4,y=2,(2)x=-1,y=-2, 9 3 (3)x=豆,y=18;5.(1)2=2+36,(2)g=-2i. 习题4.5(2)2.(2)3+i,(3)V10,4V;3.2W+i背 4.60.83公斤. 习题46(1)1.(1)18+15i,(2)6-17i,(3)0.11-0.07, (4)1;2.(1)(a2+b)2,(2)(a-b)+(a-b); 3.(2)(i)(+22)(x-2),(i)(x+a)(x-a)(x+a2)(x-ai), (a++)%+ー)4.(1)6,-4,(2)1,11. 习题46(2)1.(1)V√3+V3,(2)16, (3)(co)(d) 22 习471。(の)-1+()-넣-중,(32 (4)cos(-75°)+iin(-75),(⑤)-6+V;2.(1) 4 2)-:3。(0)-6+2 ②g=-合+号;u=1+,8-5.1.习题481.④)-19-120,②)-17+44,(3)寻国2+2V5.)82,(2②)3g(44-53.(-】, (2)-i. ·234.· メ ==========第240页========== S 习题4.8(2) 1.(1)243,(2)729i,(3)4096,(4)1; 2.(1)2,(2)一16,(3)0,(4)-64.习题48(3)2.n=6,-1;3.%=4;4.%=4; 6.cosd=cos”9-C只cosn-20gin29+C4cosn-40sin49-…,最后一项,当n为偶数时为(-1)sin日,奇数时为(一1)宁mcos日in-19;8inn0=C%cosn-10sin0-C?cosm-3esin30+C%cosn-50sin3日-',最后-项,当m为偶数时为(-1)学ncos0in-10,奇数时为(-1)学in”日,习惠4.9(1)1.(④2es专m+i告x小k=0,2, (2)√/2[c0s(6-1)10°+(6k-1)10](k=0,1,2,3,4,5); 2.四() ②z[ws4315+44315(飞=0,1,2,3),4 (3)/2[cos(3k+1)20°+i8in(3k+1)20](k=0,1,2,3,4,5), (④⑧cos360°+135°,.k360°+135° 5+isin 135]k=0,1,83,4好 3.四-1,言±,②21,20-1t9: 4.(1)-a, Wn@s,a0,aw⑧)±(竖±竖水 (4)士a,士ai. 复习题四 1.(1)0,(2)0;5.(1)=2,y=1,(2)x=4,y=3, ③(侵,-2),(会,1(2-2),(2,1,(④=-1,=5; 6. () (ms24+ia20), (2 ( s+is 용);7。 4)013 (2)1;8.(1)x+2x2+9,(2)-2x2+9;·11.(1)±(1+2), ±(v+,±(Vi-,④±(1-V):2 2 2.1)影+瑞,e)瑞91.2-+1-0 ·235· ==========第241页========== 人 (2)22-2+5=0; 第五章 习题5.1(1)1.(四丝,(②45,(8)8,(国2号 2.(1)-11,(2)-8;3.a=4,b=5. 习题6-1y1.a四e-1e++y儿+1,@a+1e2+¥e-1-Y (3)(+1+i)(+1-)(エ-1+i)(c-1-i), (+)(-=Y-)6-+프)(+=y3) 2.(1)x4-15x2-10x+24,(2)36x4-13x2+1; 3.(1)2-(√2+√3)x+√/6,(2)x3-(1+)x2-x+(1+i).习题5.1(4)1.p=-2,9=-5,Y=6;2.a=-50,b=-10,商是x+5;3.(1)土(3ax2-2x-3),(2)±(2ax2-3+5); 4.(x-1)3(x+3)3. 习题5.2(1)1.(1)商3x3+2-2c-1,余-4, (2)商2ax4-2x3+4x2-3+2,余-23,(3)商2+3c+2,余0, (4)商x3.-x2+x-1,余0;2.(1)-5639,(2)1741; 3. (1)1용()1 习题5.2(2)1.(1)商x2+4,余0,(2)商8+x2+x+2,余-3; 2.(2)(2c-3)3ax+4(x+(x一);3.飞=-9,(2c+3(c-1(+3).习题5.3(1)2.(2) 2; 3. -그共34;4。(1)4, 2 ②2,2,2- 习题58(2)1.22,-:2。 ·236· ==========第242页========== 3.1士V2i,1士√2i4.1,1+,2;5.飞=-2,根是-2, -2±W/5. 习题5.3(3)1. aの),(2)33)g,(④星 2.(1)2y9-py2+9y-r=0,(2)rg3+g2+py+1=0, (3)y8+p22+2qy+3r=0,(4)yy8一gy2+2py-1=0; 3.(1)2y3+15y+37y+31=0,(2)31y3+37y2+15y+2=0, 习题41.④2士V7,-8,②)1士V,1+V; 2.(1)(x+2)(x-2+i)(c-2-)=0,即x3-2x2-3c+10=0, (2)(ェー2+i)(·-2-)(z+1+)(+1-)=0, (3)c(+i)(ーi)(-1+√2i)(~1-V2)=0, (4)(エ+1-√3)(エ+1+√3i)(c-1+√3i)(x-1-3i)=0; 3.(1)23+x2+x-3=0,(2)1,-1-V24.1±i,1±2. 习题66山)1.-2,34,②-1,2,-会 (9)4,공(4)33-,y32 4.Q)-1,-,-1,1y76,②1,-3,3m 2 2 3-1,是,1,④,±,±2 5.(1)(x+3)(x+4)(-4),(2)(c+3)(2c-1)(3a2+2x-4).习题i.6(2)1.(1)1±V2,2,(2)V2±1,2; 2.a=b=1,根是1,1±V公;3.x4+28-52+6c+2=0; 4.1±√2,-1;5.V3=V2,-V3±V2i,-1. 习题56(d)1.④国(osgx+nm5gre=01,23动,8 (3)98(s8%+720 +isin8k+720r} (k=0,1,2,3,4), (6 v(12:+프。+in12".)(-,1,2,3);24 24 2.②-是,二3y③,④2Va(-1±,营V1生,2’ 4 ·237· ==========第243页========== 心 @±培,原,g起4 4· 习题56(2)1.a,(-1,3, 2 (),(8), 3 @)zes8吉2x+法2a)=-01,2,8k+1 (m吉7+n84吉7+)传=0,1,2》,8k+7 ) ,i,V6专6,-V6V6;2 2@四o,00,0,(-+号》竖±)2 2 2 2 号±竖)四tV5,%,-生a2 习题663)1.,-131;2.-2士V③,1生Y5; 2 4 34y5,二그2V2;4.-1,2,,-2ł3; 3. 2 3 5. -1, 1,3*5,6.2。끓보3,-약2 2 2 习题.6(4)1.1,±i,-3±V7,4 2. 1,35,-그/34,3.1,2,풍,-3,- 2一 3 3・ 习题665)1.22,-号,宝;81土V,1±y5,2 3.-1生,-2,骨-1,1特5,2,资2 复习题五 1.(1)商x2-x+2,余-12,(2)商x8+2x-3,余20;2.(1)-3, ·288· ==========第244页========== (2)-7693;3(1)a=-52,b=-15,(2)a=24,b=2, (3)a=3,b=-4,(4)a=2,b=3,c=3;4.(1)-x; 6.(1)=25,根是-3,-3±√②,(2)=士6,根是±1,±2,干3, (8)k=-92,根是1,2,4,6;7.(②)2士V3,5±y7i; 4 8.(1)-6,(2)-13;9.(1)y3-6gy+8y-8=0, (②)2+2y+5y-1=010.(1)±6,-2士,(2)-6,1±y3;2 .④2士V,-3-图)V3,2士 12.(1)《x-2-3)(x-V2)(x+3)=0, (2)(x-2-3)(x-2+3)(x-√2)(c+3)=0, (3)(-2—3i)(ェ2+3i)(ー√2)(x+2) (r+3)=0; 18.④,±营(@)-3,京,(国子号是(倒设有有蜜限; 14.(1)x2(x-2)(x+3)(x2+x+1), ②:-2+a(-1+-Y为 16.)-1,1±⑤i,2,-1tV, 2 (②)±V,土V2,±√3,±V6,(3)1,-1±,7±v52 2 (4)Vi,/2+2. 第六章 习题6.1(1)1.(a17,(2)-子(3)1,(④-2 3.(1)x=1,y=-2,(2)-3,1,(3)a+b,a-b,(4)4,b.习题6.1(2)1.(1)有唯一解,(2)有无限多组解,(3).无解, k。 (4)有唯一解;2.(1)*士1时,x=车,y=龙十1)k=1时,有无限多组解,k=一1时,无解,(2)k=0或k=1时,有无穷多组解,飞*0或+1时,有唯一解父=无+1,y=尼一1.习题8.2(1)1.(1)18,(2)74; ·239· ==========第245页========== 2.(1)a9+b8+c8-3abc, (2)2cy2. 习题6.22)2.(1)0,(2119 108; 4.(1)16,(2)36. 习题632.(1)-94,(2)5;3.(1)27,(2)92; 4.(1)-20,(2)12. 习题641)1.1)-,9=2-3,(2-1只,y=1,=与 (3)x=1,y=3,名=5,(4)x=a+b, ,v=,:-은2 2.(1)无穷多组解,(2)无解,(3)无解,(4)无解, (5)无穷多组解,(6)无穷多组解. 习题6.4(2)1.(1)x=k,y=0,2=-无,(2)x=0,y=2k,2=k, (3)x=0,y=k,名=2k,(4)c=k,y=一k,2=k. 复习题六 1.(3)x=1,y=1,名=1;(4)x=-5,y=5,8=15; 2.(1)无解,(2)无穷多组解,(3)无解,(4)无穷多组解,(⑤)无解,(6)无解,(7)无穷多组解,(8)无穷多组解; 3.(1)44,(2)8,(3)27,(4)a(a-b)(b-c),(5)0, (6)a2(c-b)+b2(a-c)+c2(b-a);4.(1)4abc,(2)0,(3)8abc; 6.(1)x=2,x=-3,(2)2,(3)x=4;7.(1)x=y=z=0, (2)x=,y=-k,8=-k,(3)2=y=2=0,(4)x=y==0, (5)x=k(a+b+c),y=-k,2=-k,(6)x=-2k,y=-2k,2=; 8.k=0,-3士2V2I. 总复习题 1.(2)90,45;2.(2)1,(3)4,(4)没有;3.(1)1或4,(2)4, (3)6;4.(1)x=15,y=5,(2)m=34,2-14;6.(1)42, (2)号;7.(1)100000,(2)0.00001,(3)50000,50000, (4)0.01008;8.(1)280,(2)560;9.(1)1960,(2)3115, (3)3850,(4)2940,(5)1058400;10.(1)7,(2)25,()10; 。240… ==========第246页========== 八 宋 4 12.%=8,=4,刻=是 3 [x=0[x= x-0a= 32fx- 15.(1) 1,(2)无解,(3) (4) ly=0 ly=3 y=0 3 ly= ly=4 1 x=1[x=-…1 3 3 [x= 16.(1) ,(2) y=0 y=3’y=-3y=4【y=-4 -1+72 是-2,(2②) 517.(1) fu=1-2 -1-72’{u=1+83 5 t 19. (1) 2-3-1l+6,(2)을+-를+1, (3)2x3-9.x2+7x+6;20.(1)1,1,-2,-4,(2)2,3,-3,-4; 3-12,安,-2±V3,(④1,2士W3,3吐2V2; 21.(1)3,-5,2-V,(2)1,士2.38;23.(1)x>3, (2)-1